分からない問題はここに書いてね139

このエントリーをはてなブックマークに追加
1132人目の素数さん
さあ、今日も1日頑張ろう★☆

前スレ
分からない問題はここに書いてね138
http://science.2ch.net/test/read.cgi/math/1067753996/
2:03/11/17 09:36
余裕の2。しかし、人いねー。
3hansin:03/11/17 09:36
d(∫[a→x] f(t) dt)/dx=f(x)を証明せよ。
この問題をお願いします。 
4132人目の素数さん:03/11/17 09:40
>963  132人目の素数さん sage Date:03/11/17 08:13
>x-y平面に正三角形をおくと、どんな場所におこうとも、
>頂点のx-y座標は少なくとも一つは無理数になることの証明方法を教えてください

A(r cosθ, r sinθ) に対し、

cos(θ+60°)= (1/2)cosθ - ((√3)/2) sinθ
sin(θ+60°)=(1/2)sinθ+((√3)/2) cosθ
なので

A(a0, a1)を60°回転させると ((1/2)a0 - ((√3)/2) a1, (1/2)a1 +((√3)/2) a0)になる。

正三角形の座標を
A(a0, a1), B(b0, b1), C(c0,c1)と置く
全て有理数とする。
x方向に-c0, y方向に-c1の平行移動(有理数分の平行移動)を考えることにより
A(a0, a1), B(b0, b1), O(0,0)と置いてよい。
BはAを60°回転させたものとすると
a0, a1は同時には0にならない(Oと一致しない)ので
Bのx座標かy座標は無理数になり矛盾
5132人目の素数さん:03/11/17 09:44
>>3
f(x) の不定積分を F(x) とすると、
∫[a→x] f(t) dt = F(x) - F(a)
よって
d(∫[a→x] f(t) dt)/dx = d { F(x) - F(a) }/ dx = dF(x)/dx = f(x)
6hansin:03/11/17 09:50
132人目の素数さん ありがとうございました。
7hansin:03/11/17 09:56
もう1問お願いします。
G(x)=∫[0→x^2] (t-1)e^t dt dG/dt>0となるxの範囲を求めよ。
よろしくお願いします。
8132人目の素数さん:03/11/17 10:01
>>7
Gはxだけの関数でtは関係ないから
dG/dt = 0だよ。
9132人目の素数さん:03/11/17 10:06
>>7
>dG/dt>0となるxの範囲を求めよ。
dG/dx>0となるxの範囲を求めよ。
と考えて解く。
dG/dx = (x^2)'*(x^2-1)e^(x^2) = 2x (x^2-1)e^(x^2) > 0 より
x(x^2-1) >0 ⇔ -1 < x < 0, 1 < x
10hansin:03/11/17 12:28
132人目の素数さん
問題の間違いまで訂正していただいて、ありがとうございました。
11132人目の素数さん:03/11/17 12:55
いつもセブンブリッジをベースにしたトランプゲームをやってるんですが、
ずっと疑問に思ってたことが 質問します。
(板違いだったら、誘導をお願いします(^ ^;)

ジョーカー4枚入りのトランプがあります。(計56枚)
4人に7枚ずつ配ります。
このとき、配った時点で 7枚の合計点数が
10点以下になる 確率とその方程式が知りたいんです…。
どなたかお願いします。
同時に80点以上になる確率と方程式も、できればお願いします。

数え方は
ジョーカー…0点
A…1点
J…11点
Q…12点
K…13点
なにとぞ…よろしくお願いします。
12132人目の素数さん:03/11/17 14:27
宿題50題を解いていたら3問わからないものがありました。
@
y=x^2,y軸,y=n(nは自然数)で囲まれる領域にふくまれっる格子点の
総数をSnとするときlim[n→∞]Sn/{n^(3/2)}を求めよ。ただし、格子点とは
x座標,y座標がともに整数である点のことで、領域境界にある格子点も
数えるものとする。
A
C:y=x^4-6x^3+13x^2-6x+6と二点で接する直線はただ1つ存在する。
その直線をlとし、lとCで囲まれる面積Sを求めよ。
B
楕円C:x^2/a^2+y^2/b^2=1上の二点P,Qにおける接線が直交していて、
その交点をRとする。P,Qがその点における接線を直交させながら動くとき、
Rの軌跡を求め図示せよ。ただしa>0,b>0である。

以上3問おねがいします。途中過程も書いていただければありがたいです。
13132人目の素数さん:03/11/17 14:52
f(x)=x^3-xとする。a=s+t,b=f(s)+f(t),c=st(s,tはすべての実数)のとき、
1. bをaとcで表せ。
2. (a,c)の表す領域はどこか。
3. (a,b)の表す領域はどこか。
という問題で、3番が解けません。
ちなみに、1番の答えはb=a^3-3ca-aになり、
2番の(a,c)の示す領域については、c<=(1/4)a^2になりました。
これもあっているのかどうか不安ですが・・・。

よろしくお願いします。
14132人目の素数さん:03/11/17 15:03
>>12 どこまでできたかもカキコしてごらん。
A
l の方程式を y=px+q とおく。2つの接点のx座標を a, b (a < b)とすると
x^4-6x^3+13x^2-(p+6)x-q+6=(x-a)^2 (x-b)^2 と表せる。これより
a=1, b=2, p=2, q=2 。よって
S = ∫[1, 2] (x-1)^2 (x-2)^2 dx
=∫[1, 2] {(x-1)^4 - 2(x-1)^3 + (x-1)^2} dx
=[(x-1)^5/5 - (x-1)^4/2 + (x-1)^3/3] [1, 2]
=1/5 - 1/2 + 1/3
=1/30
1512:03/11/17 15:15
>>14
Aありがとうございます。

どこまでできたかといいますと、
@はとりあえず、図を描いて、x=kかy=kという直線ごとに
格子点の数Skをkであらわしてそれをk=1からnまでΣをとろうと
方針を立てたのですが、いまいち格子点のあらわし方が
わかりませんでした。
BはP(Xp,Yp)とQ(Xq,Yq)における接線の式を求めて、
二直線が直交しているということで、
-Xpb^2/Ypa^2×-Xqb^2/Yqa^2=-1となり、
式が複雑になってしまい、これでいいのだろうか・・・。

という感じです。
16132人目の素数さん:03/11/17 15:19
>>12
(1)
y=0上に(0,0)の1個
y=1上に(0,1), (1,1)の2個


y=k上に(0,k),(1,k),…,([√k], k)の [√k] + 1個


[ ]はガウス記号。

m^2 ≦ k < (m+1)^2のとき y=k上に m+1個ある。
そして

y=k (m^2 ≦ k < (m+1)^2)上に (2m+1)(m+1)個ある。
n=p^2 の時
Sn={Σ[m=0 to p-1] (2m+1)(m+1) } + p+1
= (p+1){4p^2 -p+6}

n=p^2 +rの時 (ただし p=[√n]とする)
0≦r< 2p+1

Sn= Sp^2 + (r-1)(p+1)

lim[n→∞]Sn/{n^(3/2)}=4
17132人目の素数さん:03/11/17 15:33
>>12
B
R の座標を (X , Y) とする。R から楕円に引いた接線の方程式をパラメーターtを用いて
(x , y) = t (cosθ , sinθ) + (X , Y) と置き、楕円の式に代入して整理すると
{a^2(sinθ)^2+b^2(cosθ)^2}t^2+2(b^2Xcosθ+a^2Ysinθ)t+b^2X^2+a^2Y^2-a^2b^2=0
tの2次方程式と見たとき、重解を持つの
(b^2Xcosθ+a^2Ysinθ)^2-{a^2(sinθ)^2+b^2(cosθ)^2}(b^2X^2+a^2Y^2-a^2b^2)=0
整理すると
X^2(sinθ)^2+Y^2(cosθ)^2-2XYsinθcosθ=a^2(sinθ)^2+b^2(cosθ)^2・・・@
もうひとつの接線は90度回転したものなので、上の式でcosθ→-sinθ , sinθ→cosθ
とそれぞれ置き換えた式も成り立つ。
X^2(cosθ)^2+Y^2(sinθ)^2+2XYcosθsinθ=a^2(cosθ)^2+b^2(sinθ)^2・・・A
@とAを加え合わせて
X^2+Y^2=a^2+b^2
18132人目の素数さん:03/11/17 16:05
Rank[A,B]=Rank[A,B,A+B]
(行列A,Bについての条件は特になし)
はどうやって証明すればいいのでしょうか。


>>18
マルチ
20132人目の素数さん:03/11/17 16:21
>>18
Rank[A,B]の定義は?
Rank[A,B,A+B] の定義は?
21132人目の素数さん:03/11/17 16:25
>>20
Rank[A,B]=Rank[A,B,A+B]
を証明しろ、とだけ書かれていた問題で
定義などにはふれていませんでした。
定義も知らずに
>どうやって証明すればいいのでしょうか。
などと質問するのか。
2313:03/11/17 16:43
自力解決できた気がします。
お騒がせしました。

多分、a>0のとき、b>=a^3/4-a
a=0のとき、b=0
a<0のとき、b<=a^3/4-a
でいいんじゃないかなと思いました。
24132人目の素数さん:03/11/17 17:05
>>23
確かにその通りになった。
25hansin:03/11/17 17:51
log c / log a + log c / log b =0 のとき
abc+1=ab+c であることを証明せよ。
よろしくお願いします。
26132人目の素数さん:03/11/17 18:15
>>25
c=1のときabc+1=ab+cは常に成り立つ.
c=1でないときは,条件式を変形するとab=1になるのでabc+1=ab+cは成立するでしょ,多分.
27あほです:03/11/17 18:18
数Uの問題です。
((32)^1/2)^1/2×(8)^1/2÷(-16)^1/3
この計算をお願いします。m(_ _)m
^は○乗の意味です。
=(32/2)^1/2×8/2÷(-16)/3
=(32/2)/2×8/2÷(-16)/3
わざわざ32とか8に括弧つけてるのに
1/2とか1/3に付け忘れたってことはないだろうから・・・
(32)^1/2
=((32)^1)/2 etc.
30132人目の素数さん:03/11/17 18:25
>>27
数Uって複素数とかあるんだっけ。
(√(4√2))*(2√2)/(-4)

-√2+√√2

・・・・でいいのかな。。。。。なんかおかしい気もする。
31132人目の素数さん:03/11/17 18:29
32132人目の素数さん:03/11/17 18:30
>>28,29

うわ・・・・俺、見事に騙された。。。。。_「|○
33132人目の素数さん:03/11/17 18:34
>>25
logc(1/loga+1/logb)=0となるから
条件を満たすのはlogc=0
または(1/loga+logb)=0が条件
logc=0となるのは
C=1の時

(1/loga+logb)=0となるのは
loga=-logbとなるのが条件
つまりa=1/bとなるのが条件
それぞれの条件を代入すると
abc+1=ab+cが成立
34132人目の素数さん:03/11/17 18:43
       ,,..-''''":::::::::::::::::``''''-、
     ,.r''"::''" ̄``''''-、‐''''ー-、::::`>‐''"":ヽ、
    /::::::::::::::::::::::::::::::::::::\::::::::`Y/::i::!::::r'"::',
  ,r':::::::::::::::::::::::::::::::、:::::::::::::::\::::::|:i::::!:;::/:::::::::! logc(1/loga+1/logb)=0となるから
 ./:::;::ヽ:::::\::::ヾ;::::::::\::::\:::::`、::ゞ、;::::i'::::;;:ノ 条件を満たすのはlogc=0
 i:::::i::::|''\`、\::\_;;;;;;;;\:::`;:::::::i;:::::::`'''''「、::::', または(1/loga+1/logb)=0が条件
 |:::::|::::|.  ヽ;\''"\`ヾ、;|_\::ト、::|;::::::;:-、:! 'i::::', logc=0となるのは
 i::::::';:::|,r'''' `ヾ\   r''i;;;;;;`ヾ、 ';| '、/ ,. ! |:::::! C=1の時
 .|::;::::'、|. r'i;;ヽ,      .r';;;;;;;;;;! i   ! iソ,ノ .!:::::i (1/loga+1/logb)=0となるのは  
  i;::';:::::'、.| 'q;;;ヽ.      '、;;;o;リ '   ,.ィ.   !:::::| loga=-logb
  !:/、::::'、'、ヾ;;;リ ,     ''"゙     ,リ'    |::::::! つまりa=1/bとなるのが条件  
  "  \!、` ''" ヽ           !     |::::::i それぞれの条件を代入すると 
      |::'、    -‐''フ    ,    i    |:::::::! abc+1=ab+cが成立   
       |:::::`i''-、、,_ .`"  .,.r'"    ',   |:::::::|  
      |:::::::i    ``''、‐'"        .',,,_  |::::::::!
       .!:::::::',    ,,,..ゝ     ,,.-‐''" i. |::::::::|
      .';::::::::'、 ,,.r''7i、__,,,,..-─''"    _,,`'''ァ::::::|
        '、:;;:-'/          _,.-‐''"   !:::::::|
     ,.-''7 /         ,.r'"   _,,.-'''"`、:::|
    /  / /'`''ー''-、、,,、,,_,,/  ,,.r''"     ';::|
35あほです:03/11/17 18:55
皆様、本当にありがとうございます。m(_ _)m
>>29
>わざわざ32とか8に括弧つけてるのに
>1/2とか1/3に付け忘れたってことはないだろうから・・・
忘れてました。本当にごめんなさい。
正しくは
((32)^(1/2))^(1/2)×(8)^(1/2)÷(-16)^(1/3)
だと思うのです。

本当は日本語で表すと
(32にルートが2重に付いている)×(ルート8)÷(-16に3重根が付いている)
ってのを表したかったのです…

(√√32)×(√8)÷(3√-16)
           ↑
         3は左上に小さく

と書けばよかったのでしょうか?
みなさまお願いします。
36132人目の素数さん:03/11/17 19:02
>>35
じゃ、たぶん
>>30 で合ってると思う。。。。
37132人目の素数さん:03/11/17 19:04
>>36
まじ??
(-16)^1/3 = -4 ???
3811 :03/11/17 19:32
11です。

やっぱり 俺の質問は板違いだったんでしょうか...。
どなたか お願いします。
39132人目の素数さん:03/11/17 19:34
>>38
                _,.. ---- .._
              ,. '"       `丶、
              /            ` 、
            ,..-‐/    ...:  ,ィ  ,.i .∧ ,   ヽ.
.         ,:'  .l .::;',. :::;/..://:: /,':/  ', l、 .i  ヽ
.          ,'  ..::| .::;',' :;:','フ'7フ''7/   ',.ト',_|, , ',.',
       ,'   .::::::!'''l/!:;'/ /'゙  /     '! ゙;:|:、.|、| 'l
.         ,'.  .:::::::{ l'.l/  、_  _,.      'l/',|.';|
       l  :::::::::::';、ヾ      ̄     `‐-‐'/! ';. '
.         ! :::::::::::/ `‐、        ゝ   |'゙ |
       | ::::::::/   \    、_, _.,.,_ ノ::: !  
       |::::/.     _rl`': 、_     ///;ト,゙;:::::./
..      `´      /\\  `i;┬:////゙l゙l ヾ/   
                ,.:く::::::::`:、\ 〉l゙:l  / !.|        <マジレスすると問題が簡単でなおかつあなたの質問が
                                 的外れだからです
40あほです:03/11/17 19:37
皆様、本当にありがとうございます。
わたくしも
(-16)^(1/3)=-4
に疑問を持ちました。

(-16)^(1/3)
=-(2)^(4/3)

となるのかな〜?
41132人目の素数さん:03/11/17 19:38
>>40
                _,.. ---- .._
              ,. '"       `丶、
              /            ` 、
            ,..-‐/    ...:  ,ィ  ,.i .∧ ,   ヽ.
.         ,:'  .l .::;',. :::;/..://:: /,':/  ', l、 .i  ヽ
.          ,'  ..::| .::;',' :;:','フ'7フ''7/   ',.ト',_|, , ',.',
       ,'   .::::::!'''l/!:;'/ /'゙  /     '! ゙;:|:、.|、| 'l
.         ,'.  .:::::::{ l'.l/  、_  _,.      'l/',|.';|
       l  :::::::::::';、ヾ      ̄     `‐-‐'/! ';. '
.         ! :::::::::::/ `‐、        ゝ   |'゙ |
       | ::::::::/   \    、_, _.,.,_ ノ::: !  
       |::::/.     _rl`': 、_     ///;ト,゙;:::::./
..      `´      /\\  `i;┬:////゙l゙l ヾ/   
                ,.:く::::::::`:、\ 〉l゙:l  / !.|        <本当にアホですね
>>38
問題が不完全だから。
完全な問題だったらやってみようかと思ったのにね。
4311:03/11/17 19:46
>>42 >>39
> >>38
> 問題が不完全だから。

どのあたりが 不完全なのか
教えてもらえませんか?
44132人目の素数さん:03/11/17 19:52
S=Gγ+η(dγ/dt)を解くとγ=(S/G)[1-exp(-tG/η)]になるそうですが
私には解けません。。。数学板で聞くには初歩的過ぎるかもしれませんが
どなたか親切な方いらしたら教えていただきたいです
45hansin:03/11/17 19:55
次の指数方程式を解け。
6^(x+1) - 3・2^(x+2) + 9・3^(x+1) - 54 =0

よろしくお願いします。
46132人目の素数さん:03/11/17 20:00
>>43
                _,.. ---- .._
              ,. '"       `丶、
              /            ` 、
            ,..-‐/    ...:  ,ィ  ,.i .∧ ,   ヽ.
.         ,:'  .l .::;',. :::;/..://:: /,':/  ', l、 .i  ヽ
.          ,'  ..::| .::;',' :;:','フ'7フ''7/   ',.ト',_|, , ',.',
       ,'   .::::::!'''l/!:;'/ /'゙  /     '! ゙;:|:、.|、| 'l
.         ,'.  .:::::::{ l'.l/  、_  _,.      'l/',|.';|
       l  :::::::::::';、ヾ      ̄     `‐-‐'/! ';. '
.         ! :::::::::::/ `‐、        ゝ   |'゙ |
       | ::::::::/   \    、_, _.,.,_ ノ::: !  
       |::::/.     _rl`': 、_     ///;ト,゙;:::::./
..      `´      /\\  `i;┬:////゙l゙l ヾ/     いくら年を重ねても
                ,.:く::::::::`:、\ 〉l゙:l  / !.|        <頭の弱さは直らない
>>45
とりあえずlogでも取ってみたら?
2ちゃんねるはlog 取ってないらしいけど。
4811:03/11/17 20:06
>>46
そこを なんとか 教えて♪
49132人目の素数さん:03/11/17 20:12
>>44
γ(0)=0という条件がないと解答通りにはならない
50132人目の素数さん:03/11/17 20:18
>>49
ありがとうございます
ではそのような条件の下でどのように解けるのでしょう?
51132人目の素数さん:03/11/17 20:20
>>44
まず、dγ/dt= の形に変形してみそ
52132人目の素数さん:03/11/17 20:22
>>44
S=Gγ+η(dγ/dt) を変形すると
(d/dt)(γ-S/G) = - G/η(γ-S/G) となる。これより
γ-S/G = (γ(0)-S/G) * exp(- tG/η)
γ(0)=0 なら確かに γ=(S/G)[1-exp(-tG/η)] になる。
53132人目の素数さん:03/11/17 20:24
>>38
やってみようかと思ったけど
激しく面倒だったため
どうしようかなと思う今日この頃です。
5411:03/11/17 20:27
>>53
なんとか お願いします。

>10点以下になる 確率とその方程式が知りたいんです…。
1人以上が 10点以下&80点以上 になる確率と方程式が
知りたいんです。
5544:03/11/17 20:29
>>51,52
ありがとうございます!
変形の仕方がすごいですね。私にはとても思いつかなかったです
数学はホント苦手。。。がんばります
56132人目の素数さん:03/11/17 20:30
>>54
          ,,..- 、
 ,・、‐-,,,、_   〃 ,...__,,,,-,,,‐‐,
 | ヽ =`-‐‐/⊂O⊃==./.}
 |  }/´_,,`ヽ'二、`ヽ、/  |
 .| ./ ./   , 、 `ヽ、ヽ  /
  | / ./,/-/‐| .| |‐|-、_i | /
  .〉| |(./-、 V|. ノ,-レ、レ| }〈
  .| |. |´|:'::j:}  レ' {:'::j:|`| ! .|   ./ ̄ ̄ ̄ ̄ ̄ ̄ ̄ ̄ ̄ ̄ ̄ ̄
  | 人lゝ` "  . ` " レ' .| < 偏微分方程式で >>11 解くもん
  { / />‐.,-、-‐< \ヽ .}.  \____________
__○○‐'´ /ヽ-/ \ ` ‐-○○_
`レM.V  /l ̄レ'ヽ) ̄lヽ  .レMヽ!
.     |_|__    __|_|  ,,,-、
       V / ̄| ̄!. レ/`/
     〈___ヽ__、 /___〉'''"
       |~~l l~~l
        ./~~V~~ヽ
      〈__,、_|、_,__}        
57132人目の素数さん:03/11/17 20:30
>>45
6^(x+1) - 3・2^(x+2) + 9・3^(x+1) - 54 =0

a=2^x
b=3^xと置くと

6ab-12a+27b-54=0
2ab-4a+9b-18=0
(2a+9)(b-2)=0

b=2

x= log(2)/log(3)

少し不安があるが答え出たからいいか。
58132人目の素数さん:03/11/17 20:33
>>54
時間があるときにね。
でも、方程式って何?
5911:03/11/17 20:36
>>56
だから...
>偏微分方程式で >>11 解くもん
これが わかんないんだもん(苦笑)

ギャンブル板で訊いてもスルーされそうだったので
ここに 来たんだけど...。(泣)
どなたか お願いします。
あらためて 問題を再掲します

いつもセブンブリッジをベースにしたトランプゲームをやってるんですが、
ずっと疑問に思ってたことが 質問します。
(板違いだったら、誘導をお願いします(^ ^;)

ジョーカー4枚入りのトランプがあります。(計56枚)
4人に7枚ずつ配ります。
このとき、配った時点で 7枚の合計点数が
一人以上が 10点以下になる 確率とその方程式が知りたいんです…。
どなたかお願いします。
同時に80点以上になる確率と方程式も、できればお願いします。

数え方は
ジョーカー…0点
A…1点
J…11点
Q…12点
K…13点
なにとぞ…よろしくお願いします。
60132人目の素数さん:03/11/17 20:40

          ,,..- 、
 ,・、‐-,,,、_   〃 ,...__,,,,-,,,‐‐,
 | ヽ =`-‐‐/⊂O⊃==./.}
 |  }/´_,,`ヽ'二、`ヽ、/  |
 .| ./ ./   , 、 `ヽ、ヽ  /
  | / ./,/-/‐| .| |‐|-、_i | /
  .〉| |(./-、 V|. ノ,-レ、レ| }〈
  .| |. |´|:'::j:}  レ' {:'::j:|`| ! .|   ./ ̄ ̄ ̄ ̄ ̄ ̄ ̄ ̄ ̄ ̄ ̄ ̄
  | 人lゝ` "  . ` " レ' .| < 偏微分方程式で >>59 解くもん
  { / />‐.,-、-‐< \ヽ .}.  \____________
__○○‐'´ /ヽ-/ \ ` ‐-○○_
`レM.V  /l ̄レ'ヽ) ̄lヽ  .レMヽ!
.     |_|__    __|_|  ,,,-、
       V / ̄| ̄!. レ/`/
     〈___ヽ__、 /___〉'''"
       |~~l l~~l
        ./~~V~~ヽ
      〈__,、_|、_,__}        
6111:03/11/17 20:43
>>60
すいません...

こっちは ビセキも イマイチなんで...
わかんないっす
62132人目の素数さん:03/11/17 20:46
>>56
偏微分方程式とどう関係するの?
63132人目の素数さん:03/11/17 20:51
偏微分方程式でラグランジュ方程式解くもん

       ,. -r '´ ̄ ̄`゙ヽ、 ,-- 、     ,. ------ 、
      / ミ彡ィへ、\、彡ヽ_ノヽ!   / ,.ィ   |   `゙ヽ、
     / 彡'`   |ヽ\__ヽ、彡 ヽ<. レ'´/ / | | | | | | |ヽ ヽ
   r‐ |  |     ヽヽ´\\V/ V | ///- | | | | | | | |  ヽ
 .rゝ‐| || | , -    ヽ\ヾ\ヽリ |// 〃  ヽ !ヾ |ヽ「|「i |  ト、
 / ヽ_ノ || |′         /,ニミ、 レ|///|! _  ヽ ヽ、l|ヽ || |   |〉ヽ
 | /| ヽ|| |   __     {::;;;:::}   }|| | | !/,--、    r'ニヽ. | l |i ||
 | | | || ! | / r;;;ヽ    `ー'´  ',ィ!-' | {::;;;:j    |::;;;::} ! |j || | !
 | | | |!ヽr‐ | {::::::ノ   ,      j {入| ` ´     `ー' °rヽリヽ{|
 | | |   ヽ、ヽ ̄        /| リiーi      '      / ノ
 | | |    `ーヘ____ー ' ,ィ'‐┴くュ___ヽ-、  。    /´
 | | .!       //  _r‐二ユ->─<` ` ⌒l-ミ--r─‐r'ヽ┐
 | | l     /  ! '´  /     ヽ`     |_   ヽ  ヽ ┴,
 .', ! !    |  |        //ヽ、   |ー   ヽ  |‐' ト、
  ', l  ヽ  /   | -一′      / !   /ト    |  /  l_|
   ヽ!   |    \    _,.-rイ|〈⊥/ |──‐┴イ |  L|
64thubasa:03/11/17 20:58
一番の難題です。
簡単そうで難しい。
しかも答えは無い。
計算はしないのに難しい。

  問題
   0≦x≦10
これは、「xは、0以上、10以下」を、表す、変域です。
では、
   0<x<10
「xは、0より大きく、10より小さい。」
「より小さい」は、未満。
では、
「より大きい」は、なんと表現するでしょう。

ちなみに、未満の反対語。
分かったら即座に教えてください。
65132人目の素数さん:03/11/17 21:00
               __
           , -‐ '' ~   ~" '' ‐-、,_
        ., -'",,-、,-‐'"~'、__,,!"''7_,,-、,.''‐、
       ./ /ト‐"‐ '"    ,  ~、 ^、人_ト',、
      ,/ // /, '"  .,-‐'", 、 '、 ヾ、-、_]'、
     , ',_iニ!/ // ,,;‐'"  ノ  人ヽ;  '、、|_1ヽ、
    ,' .{ j | // ,/ ,,- //ノ! i,!、ヽ ! T トヽ, 、
   .//!"  |/ / / ///./  iト、 '、! | ゞ, ,,;i
   / 7    |/ // /// / ____|  、 |   |,;:'''
  /,  |    |/" ,/∠,,ノ / ::'" __,!''‐ ! |   |方程式という
  "':;;;;;| .|   | ///_/     .,/";;;''、 ,|/,   |言葉を使うのがいけないんだよ
     | |,   ',/ //''";;;~!     ヽ_;;;/~/;'   .|
     | | 、  | / {,ヽ;;;;;ノ     ....‐‐' /;;  , |
     .| '、ヽ  |ト、  '-‐';;;;;  、  '''''''' /,;;' .ノ! |
     .|  !;;'、  |-ヽ、      ⊂ニ;;、 /,;;' ./ | |  
     |  ヽ;;,、 |"'‐-'- 、,, 、-‐''‐-,i !,;;;' ./;;/ | 
     |  '、;;;;、| _/''^", '‐ ,ゝニ''‐、"' !;:' /;;;'  /‐--、
     人 、 '、;;;;| |  .< 、/⊂~"'''  / /;;;/ . |    !.
     !;;;! ヽ 、;;;| |   'フ ,i 7    /__/;;;;/ / / ,,;;;   !
    /!;;;、 |;、 、| i、  ム/ <-v-"~''" !;;;;/ / /!;;;;;''   .!
    | !;;;;,、 |;;;、y匚'‐- 、___ \,-‐--ト'"!; / /./;;''    .!
   ./| .!;;;;;,、|;;;,、|、_,-、_/__i ~/ 〉==‐/^、ヽ、/ ノ'' ,,;    .j
6611:03/11/17 21:02
>>65
ごめんなさい。
「方程式」って言うのは 取り消します...。

せめて 確率だけでも お願いします。
67132人目の素数さん:03/11/17 21:29
>>64
超過
68132人目の素数さん:03/11/17 21:29
3人の旅行者が土産屋に入り、3000円の土産を割り勘で買うことにし、
ひとり1000円ずつ出して土産物屋の小僧にそれを払い、買った。
小僧は奥の店主のところにその3000円を持っていくと、店主は、
「それは2500円でいいや」と言い、500円を3人に返すよう小僧に言った。
が、小僧はそれをいいことに200円パクり、300円だけ3人に返した。

さて、この旅行者は結局一人あたり900円払ったことになる。
900円×3=2700円、そこに小僧がパクった200円を足して2900円。
あれ、100円はどこに行った?

これマジわからん。教えてエロイ人!
6911:03/11/17 21:30
70132人目の素数さん:03/11/17 21:54
正方行列F,GがFG=GFを満たすならば自然数nについて
F^nG=GF^nがなりたつことをを証明せよ。
成分を比べてやろうと思ったのですが、項がぐちゃぐちゃになってお手上げです。
どなたかお願いします。
71132人目の素数さん:03/11/17 22:02
>>70
普通に
F^nG = F^(n-1) {FG}=F^(n-1) {GF} = { F^(n-1) G} F
=…以下繰り返しで = G F^n
>>70
アフォか。結合法則を順番に使うだけだ。
73132人目の素数さん:03/11/17 22:06
ある有名小学校の入学試験問題です。
次の数字はどんな規則で並んでいるのでしょう。
0 , 10 , 1110 , 3110 , 132110 , 13123110 , 12234110
※なんと、実際のテストでこれができた子が1人いたそうです。


答え教えて下さい
74132人目の素数さん:03/11/17 22:06
>>71,72
ああ、そうか。どうにもセンスが無い上、行列は習いたてなもので。
有難うございます。
75132人目の素数さん:03/11/17 22:15
>>72
いや、結合法則じゃなくて
交換法則…
>>75
おいおい、行列の積に結合法則が成り立つことを使わないとだめだろ。
77132人目の素数さん:03/11/17 22:21
>>73
>※なんと、実際のテストでこれができた子が1人いたそうです。

出題者と趣味の合った子が1人いたってことだな。
その子にとっていいことか悪いことかは悩むところであるが
78132人目の素数さん:03/11/17 22:27
初項が正、第二項と第四項の和が20、第四項と第六項の和は80である等比数列の一般項を求めよ。

この問題を教えてください。
>>78
初項を a, 公比を r とでもおいて条件どおり書けヴァカ。
80132人目の素数さん:03/11/17 22:32
すみません 教えてください
今日学校で
sin(θ+90°)=cosθ が成りたち
θ=θ−90°の時
sinθ=cos(θ−90°)が成り立つと言われたのですが
θ=θ−90°ってところが理解できません。
何故θとθ−90°がイコールになるのでしょうか?

8111:03/11/17 22:32
皆様からの お答えを じっと待ってます........♪
>>78
趣味の問題ではなく、類題で訓練をしていたのでしょう。
>>73
ただでさえひねくれているのに、数字の列を右から読むあたりは
変態としか言いようがない。
>>78
初項a(>0),公比rとおくとー、一般項a(n)=ar^(n−1)。
ar+ar^3=20,ar^3+ar^5=80 
⇔ ar+ar^3=20,20r^2=80
⇔ (a,r)=(2,2) (∵a>0)
∴a(n)=2*2^(n−1)=2^n。
8478:03/11/17 22:38
>>83
20r^2=80
ってのはどうやって出てきたのでしょうか?
85132人目の素数さん:03/11/17 22:39
>80
>θ=θ−90°の時

これは同じ変数を使っている。確かに変。

多分説明の上ではθ= a -90°のとき
sin a = cos(a-90°)
というような事が言いたかったのでは?

θ=θ−90°ではなくて
θのところにθ-90°を入れると
と表現しただけだろう。多分。
>>84
20r^2=(ar+ar^3)r^2=ar^3+ar^5=80だーよ。 
87132人目の素数さん:03/11/17 22:40
>>84
ar^3+ar^5 = r^2 ( ar + ar^3)
8878:03/11/17 22:42
>>86
理解できました。
ありがとうございました。
>>11は何の変哲もないただのマンドクサい計算問題。
数学板の回答者が一番嫌うタイプ。
90roo:03/11/17 22:44
f(x)=x^3+ax^2+bx+eがx=3で極大値56をとりx=5
で極小値をもつときa,b,cの値を求めよ
という問題なのですがわかる方教えてください。
お願いします。
>>90
f'を求めて極大、極小となる条件および該当する極値を当てはめ、出てきた方程式を解く。
9211:03/11/17 22:48
>>89
すんまそん...。
この板では 板違いかと思ったんですが...
ここしか 思い浮かばなくて、
どっかに いい人がいるスレはあるんでしょうか?

なかったら ここの住人さんに 引き続きお願いしたいと
思います。
マルチは したくないんで…。

93132人目の素数さん:03/11/17 22:50
>>90
f(x)を微分すると
f'(x)=3x^2 +2ax+b

極値の位置からして
これは
f'(x)=3 (x-3)(x-5) = 3x^2 -24x+45
になる。

a=-12
b=45

f(3)= 27 + 9a+3b+c=56
よりcが求まる
94132人目の素数さん:03/11/17 22:51
>>92
きっとどこへ行っても同じ。
計算にえらく時間がかかる。
このスレにずっと張り付いて貰っても
今夜中に出るかどうかすら定かではない。
95132人目の素数さん:03/11/17 22:53
>>85
θのところにθ-90°を入れると
と表現しただけだろう。

とあるのですがこれは「θにθ−90°を代入する」という
解釈でいいのでしょうか?
それとも「θの代わりにθ−90°と置き換えると」という解釈
なのでしょうか?
96132人目の素数さん:03/11/17 22:55
>>95
下の解釈だろうな。
9711:03/11/17 22:57
>>94
そうなんですか…。
マージャン牌に比べれば ずっと少ないし
ここの住人さんだったら すぐだと思ったんですけど、
かなり面倒みたいですね…。

気長に待つことにします。

ちなみに 3点以下が できる確率は
ざっと 14兆3000億分の1だと思うんですが
間違いないですよね?
98132人目の素数さん:03/11/17 23:11
ちょっと大学の過去問題を解いているのですが、全く分かりません。
参考書などを見てみたのですが、同時密度関数の類題があまりになく解法の手段に手を焼いています。

確率変数X,Yは以下の同時密度関数を持つものとする。
fx,y(x,y) = 2 0≦x≦y≦1
      = 0 それ以外
このとき、E[X](平均),V[X](分散),及びCov(X,Y)(共分散)を求めよ。

どなたか分かる方お願いします。
99132人目の素数さん:03/11/17 23:18
>>97
数学ができる人間は
暗算や四則演算が苦手なのが多い(w

算盤の達人で 数学ができるなんて話も殆ど無いし
100132人目の素数さん:03/11/17 23:18
>>98
周辺分布でもだせば?
10111:03/11/17 23:20
>>99
なるほど…

だったら この疑問は どなたに
尋ねれば いいんでしょ?
102132人目の素数さん:03/11/17 23:22
>>101
やり方は単純なんだから自分でやる。
>>101
式そのものは簡単。得点がNになる確率は
納0≦ai≦4,琶a_i=N]C[4,ai]/C[56,7]
よって10以下になる確率は
納N=1.10]納0≦ai≦4,琶a_i=N]C[4,ai]/C[56,7]
計算機がつかえるなら↑これ計算すればいい。
10480、95:03/11/17 23:29
>>85>>96のことを友達に言ったのですが
代入も置き換えることも解釈はいっしょだと言われてしまいました。
友達が言うには、「例えばθを90°に置き換えるとき、θに90°を代入すると言うだろう
そのときθ=90°って書くだろう? だからθをθ-90°に置き換えるときだって
θにθ-90°を代入するからθ=θ-90°書いて何が悪い」と言われてしまいました。
みなさまこの意見はどうなのでしょうか?

10511:03/11/17 23:33
>>103

> 式そのものは簡単。得点がNになる確率は
> 納0≦ai≦4,琶a_i=N]C[4,ai]/C[56,7]
> よって10以下になる確率は
> 納N=1.10]納0≦ai≦4,琶a_i=N]C[4,ai]/C[56,7]
> 計算機がつかえるなら↑これ計算すればいい。

ありがとうございます。
よくわかんないけど

[56.7]ってなんなんですか?
あと aiとか…。

すいません 文系で…(泣)
数学ではイコールの左右では等しくなくてはいけないのでθ=θ-90°と書けば問題あります。
あなたの友達は論点はずれています。
107106:03/11/17 23:35
プログラムを組む上では

A = A+1

と書くことはできます。
108132人目の素数さん:03/11/17 23:36
>>104
同じ文字を使うのが悪い。θ′とかを使ってほしい
109132人目の素数さん:03/11/17 23:38
>>104
知能障害者を友人に持つと誰しも苦労しますね
>>98
E[X]=∫_[0≦x≦y≦1]2xdxdy=∫_[0≦x≦1](∫_[x≦y≦1]dy)2xdx=∫_[0≦x≦1](1−x)2xdx=[x^2−2x^3/3]_[x=1,0]=1/3
E[X^2]=∫_[0≦x≦y≦1]2x^2dxdy=∫_[0≦x≦1](∫_[x≦y≦1]dy)2x^2dx=∫_[0≦x≦1](1−x)2x^2dx=2[x^3/3−x^4/4]_[x=1,0]=1/6
V[X]=E[X^2]−E[X]^2=1/6−(1/3)^2=1/18

Cov[X,Y]は、E[Y]を求めてから、同様に積分して求める。一つぐらい自力で解いてみましょう。
111132人目の素数さん:03/11/17 23:43
>>107
A = A+1と書ける言語って

等号は == のように2つ重ねたり別の表現を使うと
思うのだけど
11280、95、104:03/11/17 23:43
すみません
置き換えるということと代入するということは意味は同じことなのでしょうか?
113106:03/11/17 23:46
>>111
インタプリンタの BASIC は = でいいんでない?
>>112
同じ。元々「代入」は、英語の「substitute A for B」の訳語。substituteは置き換える、という意味。
>>105
ゴメン。ちょっとまちがった。
得点がNになる確率は
納0≦ai≦4,琶a_i=N]Π[i=0,13]C[4,ai]/C[56,7]


>[56.7]ってなんなんですか?
C[m,n]はm!/(n!(m-n)!)のこと。2項係数。全事象の数は56枚のカードから7枚のカード
をえらぶのでC[56,7]。
 
>あと aiとか…。
0=ジョーカーをa0枚、1をa1枚、2をa2枚・・・13をa13枚えらぶ組み合わせの数は
C[4,a0]×C[4,a1]×C[4,a2]×・・・×C[4,a13]
なので0=ジョーカーをa0枚、1をa1枚、2をa2枚・・・13をa13枚えらぶ確率は
C[4,a0]×C[4,a1]×C[4,a2]×・・・×C[4,a13]/C[56,7]
選んだカードの総数は7枚なので
a0+a1+・・・+a13=7
その場合の得点は
a0・0+a1・1+a2・2+・・・×a13・13
つまりaiがみたすべき条件は
a0+a1+・・・+a13=7&a0・0+a1・1+a2・2+・・・×a13・13
この2つの条件をみたす数列を全部列挙してその確率を全部たせばいい。
いま手計算でやってみようとしたらちょっと大変。でも計算機がつかえるなら十分可能な範囲みたい。
しばらくまってたら誰かやってくれるかも。
11611:03/11/17 23:58
>>115
ありがとうございます…。
丁寧な説明をされても、
アフォな俺には 何がなんやら…(泣)

でも これで 解決への 大きな一歩です…。
かなり前進しました…。
ホントに ありがとうございます。
ビバ♪ 数学版♪
>>112
言葉の上では同じだが、変項に変項を代入することと値を代入することとは
意味が異なるので、同じ記号は使えないのだ。
君はきちんと理解して、ついでに友達を哀れんでやればそれでいい。
11880、95、104、112:03/11/18 00:04
すみませんもう一回書き直します

sin(θ+90°)=cosθ が成りたち
θ=θ−90°の時
sinθ=cos(θ−90°)が成り立つと言われたのですが
θ=θ−90°ってところが理解できません。
何故θとθ−90°がイコールになるのでしょうか?



θ=θ−90°ではなくて
θのところにθ-90°を入れると
と表現しただけだろう。

このようなレスを頂いたのですが
「θ=θ−90°」と
「θのところにθ-90°を入れる」
の違いがよく分かりません。
どなたかご教授ねがいます。
>>116
だったら、喪舞自身は一歩も前進しとらんじゃないか。
>>118
基礎論・記号論理関係の本読んでくれ。
121132人目の素数さん:03/11/18 00:10
>>118
「θのところにθ-90°を入れる」
というのは、
θのところに a-90°を入れるが
誤解は無いと思われるのでこのaを
改めて、これまでと同じθという記号を
用いるという意味。

「θ=θ−90°」
というのは
両辺が等しいという非常に嫌な(w 意味があり
同じ記号を持ちいるのは好ましくない
122106:03/11/18 00:12
類友ってやつで、アホはアホと群れるんだな。
123132人目の素数さん:03/11/18 00:12
>>110
ありがとうございます。
今この解法をもとに考えているのですが、
E[X]=∫_[0≦x≦y≦1]2xdxdy
の2xはどこから出てきたのですか?
>>122
あれだろ、「友達が言った」=「実は自分が言った」の法則だよ。
12511:03/11/18 00:14
>>119
そのとおりです…。

でも 俺に取っては わけのわからんことでも
>>115様の カキコを 友達に見せて
なんとか 確率を見つけたいと思うんです…。
そんなわけで 115様には 再感謝♪

レスをつけてくれた 119様にも感謝です。
>>123
略記しただけ。詳しく書くと、
 E[X]=∫_[−∞<x,y<∞]xf(x,y)dxdy=∫_[0≦x≦y≦1]x・2dxdy+∫_[0≦x≦y≦1以外]x・0dxdy=∫_[0≦x≦y≦1]2xdxdy
ただし、fは>>98の同時密度関数。
127132人目の素数さん:03/11/18 00:16
>>118
決定的な違いは

=を使っているかどうかじゃないの?
12811:03/11/18 00:17
>>125への 追記
友達に見せても わかるとは 限らないンどけどね(泣)
129132人目の素数さん :03/11/18 00:27
f(x)=lim_[n→∞]x^(n+1)/(1+x^n)の連続性について調べよ

学校でならっているのですが全然わかりません。答えはわかっている(x=±1で非連続、それ以外で連続)のですが、計算過程を誰か教えて頂けませんでしょうか?
130132人目の素数さん:03/11/18 00:32
ほんとしつこくてごめんなさい 確認させてください

「θのところにθ-90°を入れる」
というのは、
θのところに a-90°を入れるが
誤解は無いと思われるのでこのaを
改めて、これまでと同じθという記号を
用いるという意味。

この意味は
例えば
sin(90°-θ)=cosθのとき
θ=-aとすると
sin(90°+a)=cos(-a)
と表せるので
aを今までと同じ記号θを使って
sin(90°+θ)=cos(-θ)
と表すことという意味でよろしいのでしょうか?
>>129
x^(n+1)/(1+x^n)=x/(1+x^(−n))。
|x|>1 ⇒ f(x)=x。
x=1 ⇒ f(x)=2。
x=−1 ⇒ f(x)は定義されない。
|x|<1 ⇒ f(x)=0。
>>130
いいよー。
133132人目の素数さん:03/11/18 00:39
>>126
何度もすみません。どうしてもクリアしたいもので。

例えば、確率密度f(x)=2x(0≦x≦1),0(x<0、1<x)で与えられている確率分布を考えるときは、
E[X]=∫[-∞,∞]x・f(x)dxより
E[X]=∫[-∞,0]x・0dx + ∫[0,1]x・2xdx + ∫[1,∞]x・0dx
=∫[0,1]x・2xdx
として計算すればよいと思うのですが、>>126さんがおっしゃる
∫_[0≦x≦y≦1]
とは、上端、下端の下端を示す「区間」という意図でしょうか?
範囲分けの考えが納得できない、というか理解できません。

重積分の理解がなければ解けないでしょうか?
>>133
平面上で積分してるのに重積分がわからんってどういうことよ。
135132人目の素数さん:03/11/18 00:43
>>133
xで0〜yまで積分して
yで0〜1まで積分すりゃいいんじゃねぇの?
ごめんー。>>131訂正ー。
>>129
x^(n+1)/(1+x^n)=x/(1+x^(−n))。
|x|>1 ⇒ f(x)=x。
x=1 ⇒ f(x)=1/2。
x=−1 ⇒ f(x)は定義されない。
|x|<1 ⇒ f(x)=0。

137129:03/11/18 00:46
>>131
スイマセン
x^(n+1)/(1+x^n)=x/(1+x^(−n))。
がちょっとわからない(なんでこうなるか)ので詳しい説明をお願いできませんか?
>>136
ASCII で書ける文字を全角にするのは気持ち悪いんだが。
13980、95、104、112、118、130:03/11/18 00:48
ふ〜
ほんとにみなさんありがとうございました。私は今まで数学は計算問題
だけをやってればいいと思ってましたが今日いろいろ教わって大変勉強になりました。
みなさんは私が質問したところはもうご承知なのでしょうね?
ほんとにすごいと思うし尊敬します。これからはアホと呼ばれないように頑張りたいと思います。
ほんとにありがとうございました。
>>137
x^n で分母・分子割っただけだろ。高校以下のレヴェルだ。
141132人目の素数さん:03/11/18 00:50
数学は論理の学。計算だけしてて良いのは算数。
>>133
まず、平均・分散の問題を解く前に、積分をもう少し深く学習すべきだ。

>範囲分けの考えが納得できない、というか理解できません。
もしそうであれば、この問題はキミには解けない。

>重積分の理解がなければ解けないでしょうか?
そのとおり。ただし、このレベルの問題であれば、そう深い知識は必要ない。
143132人目の素数さん:03/11/18 00:53
>>137
分母分子をx^nで割っただけ
144132人目の素数さん:03/11/18 00:55
>>134
xyの確率をz軸に取ったと考えると、重積分になりませんか?
145129:03/11/18 00:56
>>140
すいません、今気づきましたx^nが1に、1がx^(−n)になってたんですね。

あともう1問でつまずいたので、どなたかお願いします。
x-sinx=K(Kは正の定数)が少なくとも一つの正の解を持つことを示せ。

とりあえずf(x)=x-sinx-kとして、f(0)=-k<0,lim_[x→∞]f(x)=∞まで考えたのですが、ここから先が・・・
146132人目の素数さん:03/11/18 00:59
>>142
そうですね。積分を勉強してみます!
だいぶ理解が深まったところでまた質問します。

ありがとうございました。
147132人目の素数さん:03/11/18 01:00
ある品物が、1パック6個入り500円で売られている。
パックではなくバラで買う場合は、
 1〜20個のときは1個100円
 21〜40個のときは1個95円
 41〜60個のときは1個85円
で売っている。
いまこの品物を53個必要であるとき、
最も安く済ませるにはパック入りを何パック買えばよいか。

この問題、わたしは
0パック買う場合、1パック買う場合、・・・
と具体的に調べていった(答えは「2パック買えばよい」になりました)のですが、
もっとすっきりした解き方はないでしょうか。
>>145
解を求める必要がないんだったら…

左辺=f(x)とおくと、fは連続。
 f(0)=0<K<K+1≦(K+2)−sin(K+2)=f(K+2)
中間値の定理により、f(x)=K,0<x<K+2となるKが存在する。
149129:03/11/18 01:09
>>148
そこで、中間値の定理を使うんですか・・・ありがとうございます!
>>148
× f(x)=K,0<x<K+2となるKが存在する。
○ f(x)=K,0<x<K+2となるxが存在する。
>>147
問題自体があまりすっきりしていないので、
それが最も手っ取り早い方法だと思う。
152132人目の素数さん:03/11/18 01:21
>>147
aパック、バラでb個買うとする。

6a+b=53

500a+ f(b)bを最小にするものを求める。
ただし、
f(b)=100 or 95 or 85 (bの値に応じた値段)

500a + f(b)(53-6a)
=(500-6f(b))a +53f(b)

500-6f(b)<0なので f(b)が定数なら、aが大きいほど
式の値は小さくなる。

0≦b<21のとき
aの最大は8
-100a+5300 の最小値4500

21≦b<41のとき
aの最大は5
-70a+ 5035の最小値4685
41≦b≦53のとき
aの最大は2
-10a+4505 の最小値4485
で2ぱっくが最小
153132人目の素数さん:03/11/18 01:36
難しい問題は、お祭りのあとで・・・
自作自演がバレタ瞬間。
http://human.2ch.net/test/read.cgi/4649/1066741054/387-388

10分の時間を空けましたが名前を一緒にしてしまい
自演がばれましたwそのあとの必死ぶりが凄いですw
154217:03/11/18 01:45
先日、防衛大の文系数学に出題された問題です。

z=cosθ+isinθとする。
(1) z^2+az+1の実数部と虚数部をθで表せ。
(2) z^2+az+1=0 が成り立つ時のθの値を求めよ。

(1)は変形するだけでよいなら、一応答えは出せましたが、
(2)について、自信がありません。
問題の記憶があいまいになってしまっているので、
もしかしたら問題そのものに欠陥がある可能性もあります・・・。

どなたか正解をだしていただけますか?
よろしくお願いします。
>>154
a実数の条件でもあるのかな?

156154:03/11/18 02:52
多分、aは実数という条件はあったと思いますが、
それ以外の条件は無かったと思います。
157リアル中3:03/11/18 02:53
関数においてaが傾きである。

y=2x
傾きは2
(1,2) (2,4) できちんと2倍になってる。

y=x^2
a=1 傾きは1のはず。
なのに、
(1,1) (2,4) (3,9)
1倍になっていない。

なんでですか教えてください( * ゚ ー ゚ )
>>157
本当に中三なら、諦めろとしか言いようがない。
確か高校になってから習う事のハズ。
今は諦めて、一年先を待て。
159155:03/11/18 03:05
a実数は当たり前だった欝・・・

条件が足りない、というかそれだけならすべての角度で成立する予感
(ただしa=-2cosΘで)

>>157

 二次係数は傾きじゃないよ。
160リアル中3:03/11/18 03:11
>>158
わかりました。

>>159
じゃあ y=ax^2 の傾きはなになんですか?
>>157

y=2xの場合、xにかける数はxの値に関わらず2。
つまり変化の割合は一定。
y=x^2の場合、xにかける数もxなので、
xが変化すると傾きにあたる数も一緒に変化しちゃうのよ。
だから一定の変化にならない。

これでどう?

ちなみに、二次関数はy=ax^2で表せるものだけ中3でも習うよね。
162155:03/11/18 03:14
>>160

関数のグラフを考えてみな。
いわゆる傾きという奴はどんどん変化するよ
傾き=接線の傾きのことで話してるけど

 変化の割合の話は習っただろうな。
「傾きはxの値によって変わる」が答えかな。
具体的な傾きの出し方は、進学校で理系に進めば
高2の夏休み前後にならうと思うよ〜。
164リアル中3:03/11/18 03:19
>>161
!!!二次関数においては傾きはその座標座標によって変わるってことっすね!!
一次関数だったら傾きはつねに一定(直線)だけど、曲線(二次)だとそのときによってちがう!!
ありがとうございました

>>162
わかりました。
変化の割合は yの増えた数÷xの増えた数 ですね。
THX!!!
165155:03/11/18 03:22
>>164
もう寝るんで最後ね

> 変化の割合は yの増えた数÷xの増えた数 ですね。

 これが直線の傾きになってるんだよ。
 絵で考えると2点を結んだ直線の傾きになっている。
覚えておくといいことがあるかもよ。
166154:03/11/18 03:37
>>155

やっぱり条件が足りないんでしょうか・・・。
もう一度問題をよく思い出してみます。
ありがとうございました。
167132人目の素数さん:03/11/18 03:48
問題とはちょっと違うのですが、
x^2+y^3=2
を満たす整数x,yが存在しないことを証明した
数学者の名前を調べています。
わかる方、もしくはこういう検索ができるサイトを
知ってる方、よかったら教えてもらえませんか。
1^2 + 1^3 = 2
169ヒッキー中2:03/11/18 04:27
>164

あんた俺より馬鹿だな 。゚(゚^∀^゚)゚°ゲラゲラ
170167:03/11/18 04:28
>>168
なら2以上なのかな。
171167:03/11/18 04:30
すいません、
x^3+y^2=2
を満たす整数(1と-1以外)かもしれません。
172132人目の素数さん:03/11/18 04:33
±√1
173132人目の素数さん:03/11/18 08:12
x軸を横軸、yを縦軸とし、x≧0、y≧0で

原点から始まって
上に凸なピークが1つだけあって、
xが大きくなると、x軸を漸近線とするかのような
グラフを y=f(x) としてあらわしたいのですが、
そんな関数とグラフの例を調べるにはどうしたらいいですか?
>>173
君は何がしたいんだ?
y=x2^(-x)
176132人目の素数さん:03/11/18 09:31
>>171
かもしれませんでは話にならん
突然すいませんが、
 [(x÷20)-y]÷(x÷20)×10=z
これをy=  になおしたいのですが・・・
暇なときにお願い致します。
>>177
{(x/20)−y}*200/x=z
(x/20)−y=xz/200
y=(x/20)−(xz/200)=(10−z)x/200
179177です:03/11/18 10:06
>>178
ありが盗!!
すいません。カテキョしてる生徒の兄貴からの質問なんですが、
俺にはさっぱり分かりません。受験生なんで頑張っては欲しいのですが。
どなたかお力貸してください。

いずれもlim(n→∞)の下で、
{cos(π/2n)+sin(π/2n)}の‐n乗
={cos(π/2n)+sin(π/2n)}の(2n/π)乗の(-π/2)乗
=eの(-π/2)乗

{cos(π/2n)+sin(π/2n)}のn乗
={1+sin(π/n)}の(n/2)乗
={1+sin(π/n)}の {1/sin(π/n)}乗の{(π/2)*sin(π/n)}乗
=eの{(n/2)*sin(π/n)}乗
=eの{(n/π)*sin(π/n)}乗の(π/2)乗
=eの(π/2)乗

という式変形は大学受験の2次の数学として正しいのでしょうか。
>>180
{cos(π/2n)+isin(π/2n)}^(-n)
の間違いじゃねーか?
>>181
早速のお返事ありがとうございます。
預かってきた紙には、虚数は書かれてないです。

つまりこのままでは間違い、極形式(?)なら
OKという理解でよろしいでしょうか。
183132人目の素数さん:03/11/18 10:51
>>180
iが抜けているとしても
下の式は変形も変
184132人目の素数さん:03/11/18 10:56
あぁそういうことか。
すいません。>>180の下の式は3行目に私の誤植がありました。
私にはさっぱりなので、とりあえずiが抜けてると言ってみます。
京大目指してるみたいなので、そっからは自力で辿り着けないかな・・・

{cos(π/2n)+sin(π/2n)}のn乗
={1+sin(π/n)}の(n/2)乗
={1+sin(π/n)}の {1/sin(π/n)}乗の{(n/2)*sin(π/n)}乗
=eの{(n/2)*sin(π/n)}乗
=eの{(n/π)*sin(π/n)}乗の(π/2)乗
=eの(π/2)乗

186132人目の素数さん:03/11/18 11:03
>>185
{cos(π/2n)+sin(π/2n)}^n
={1+sin(π/n)}^(n/2) ← 中身2乗したね。
={1+sin(π/n)}^[{1/sin(π/n)}{(n/2)*sin(π/n)}]
=e^{(n/2)*sin(π/n)}  ←部分的に極限取るのはまずいかも
=e^[{(n/π)*sin(π/n)}(π/2)]
=e^(π/2)
187132人目の素数さん:03/11/18 11:08
>>180
{cos(π/2n)+sin(π/2n)}^(-n)
={cos(π/2n)+sin(π/2n)}^{(2n/π)(-π/2)}

={1+sin(π/n)}^{(n/π)(-π/2)} ←下の式と同じ変形
= ここらへんにも下の式と同じような変形をいれるんだろう

=e^(-π/2)


  ちなみに愛は関係ありません。
>>186-187
おかげで、私の頭にも理解できそうな感じになってきました。

={1+sin(π/n)}^[{1/sin(π/n)}{(n/2)*sin(π/n)}]
=e^{(n/2)*sin(π/n)}  ←部分的に極限取るのはまずいかも

この部分の適切な説明が双方抜けている、という様に伝えてみます。
自分の衰えぶりと、みなさんの知識とに驚かされました。
189132人目の素数さん:03/11/18 11:32
何やってんだか!
{cos(π/2n)+sin(π/2n)}^(-n)={1+sin(π/n)}^(-n/2)=[{1+sin(π/n)}^{1/sin(π/n)}]^{-(π/2)sin(π/n)/(π/n)]→e^{-(π/2)*1}=e^(-π/2) (n→∞)
{cos(π/2n)+sin(π/2n)}^n={1+sin(π/n)}^(n/2)=[{1+sin(π/n)}^{1/sin(π/n)}]^{(π/2)sin(π/n)/(π/n)]→e^{(π/2)*1}=e^(π/2) (n→∞)
>>180
Tn=cos(π/2n)+sin(π/2n)

Tn^2=1+2sin(π/2n)cos(π/2n)=1+sin(π/n)

Tn^(1/sin(π/n))->e

Tn^n=Tn^2^(n/2)
={Tn^(1/sin(π/n))}^(nsin(π/n)/2)

nsin(π/n)/2=(π/2)sin(π/n)/(π/n)->π/2
より
Tn^n->e^(π/2)

同様に考えて
Tn^(-n)={Tn^(2/sin(π/n)}^(-nsin(π/n)/2)
よりTn^(-n)->e^(-π/2)
ごめん
Tn^(1/sin(π/n))->e
じゃなくて
Tn^(2/sin(π/n))->e
ね。
{Tn^(1/sin(π/n))}^(nsin(π/n)/2)
とかも
{Tn^(2/sin(π/n))}^(nsin(π/n)/2)
に直して。
192132人目の素数さん:03/11/18 12:00
皆さんこんな馬鹿馬鹿しい問題でよくこんなに盛り上がれますね。
193132人目の素数さん:03/11/18 12:01
そんなことより
なんで終わった問題を清書したがる奴が
こうもいるのかがわからん
清書っつーか何つーか・・・
超のつく馬鹿だな
別解
Tn=sin(π/2n)+cos(π/2n)
=cos(π/2n){1+tan(π/2n)}(n>1として考える)

cos(π/2n)^n
={exp(iπ/2n)+exp(-iπ/2n)}^n/2^n
=(t+1/t)^n/2^n(t=exp(iπ/2n)) (t=exp(iπ/2n))
={t^n+nC1t^(n-2)+....t^(-n)}/2^n
t->1より
{}->2^n
∴cos(π/2n)^n->1

{1+tan(π/2n)}^n={1+tan(π/2n)}^(1/tan(π/2n))^(n(tan(π/2n))
(n(tan(π/2n))->π/2
{1+tan(π/2n)}^(1/tan(π/2n))->e
∴Tn^n->e^(π/2),Tn^(-n)->e^(-π/2)
196132人目の素数さん:03/11/18 12:19
別解っつーか、いかに回り道をするかという話になってきますた
教訓:
問題は一度解かれただけでは終わったとは限らない。
多くの人に多数の解法で確認され、清書されて始めて
果実となる。
198132人目の素数さん:03/11/18 12:23
教訓も何も、同じ方法で進めて清書してるのは
ただの馬鹿だろう
教訓
すでに話が終わった同じ問題を何度も解くのは
荒らしと区別がつかない
Bourbakiなんて清書の塊だけど何か?
∴Bourbaki=荒し
202132人目の素数さん:03/11/18 12:27
>>200
強引だな
>>189 で終わってんだからもういいじゃん!
次行こう 次 次!
204132人目の素数さん:03/11/18 12:28
>>197
確認は自分のノートでやってください

自慰をわざわざ他人に見せつけるのは悪趣味以外のなにものでもありません
205132人目の素数さん:03/11/18 12:30
f(x)=(x+1)/(x^2-2x+2)の最大値、最小値を求めよ。

お願いします。
206132人目の素数さん:03/11/18 12:30
     __      ___      __
.      {``ヽ`` ‐"´       ``- "´, r´}
     ,..ゝ、/ヽ./.  ,.  、 .\./ヽ ノ..、
    {:::::::::':::::::V´./ il  li ヽ`V::::::::::`'::::}
    ヽ::::::;;::::: ;{ l. l l  l l l }ヽ:::::;;:::::;ノ
      {::::::ノt‐' i ,.l.-l、l   l,.l-l.、l {::::::ノ::::!
     `‐i |  l ,.--、  ,.--、 !.`「´ト‐'    / ̄ ̄ ̄ ̄ ̄ ̄ ̄ ̄ ̄ ̄ ̄ ̄
      ノ l.  ト '''' ┌┐ '''' イ. ! !  _l
    / . |  lニ`_‐-`_´-‐_´ニ!. | l  \  仲良くしましょうよ〜♪
    i.  / !  |`‐{::::::ヽ./´:::::}‐'|  ! ゝ.   l
    ゝ. i. ,l  l 、t:::::水:::::r'  l  } 、 !   \____________
     jノレ'-ヽN ヽゝ::人::ノ  jハノュl_.ノ
     {_.. -‐|     |:::ハ:::|.   ト、  7
     K ,.-‐l    l´,、`!   l. ``〈-,
     `Lt  ,.|    「i 7   ト、,..-ニ.!
      ヽ`く._l -‐ l !l ヽ. レ' ,┘
        `~ t.L____」‐L__l.7~
         V , , , ,. }=! ,. , , ,V
           ト!JJJ~ , ~iJJJ'|
            l      !l    l
          .l─ --‐l l‐-- ─!
          .l:::::::::::::::l. l::::::::::::::l
         .l::::::::::::::l l::::::::::::::!
         ,!_/二ニ.} {.ニ二ヽ!、
           i ゝ-─-! !-─-ノ i
         .{_  ̄ ̄} { ̄ ̄ _}
           ̄ ̄´  ` ̄ ̄
>>204
それは数学に携わっている人々への侮辱ですか?
はいそうです。
209132人目の素数さん:03/11/18 12:42
>205

f(x)=(x+1)/(x^2-2x+2)
=(x+1)/{(x-1)^2 +1}
t=x-1と置いて
=(t+2)/(t^2 +1)
           t=tanθと置いて
=(tanθ+1) (cosθ)^2
=cosθsinθ+(cosθ)^2
=(1/2){sin(2θ)+cos(2θ) +1}
=(1/2){(√2)sin(2θ+(π/4)) +1}

最大値は θ=π/8の時
(√2 +1)/2
最小値は θ=-3π/8の時
(-√2 +1)/2
210132人目の素数さん:03/11/18 12:43
>>207
時と場所によるでしょう。
211132人目の素数さん:03/11/18 12:48
>>209
     __      ___      __
.      {``ヽ`` ‐"´       ``- "´, r´}
     ,..ゝ、/ヽ./.  ,.  、 .\./ヽ ノ..、
    {:::::::::':::::::V´./ il  li ヽ`V::::::::::`'::::}
    ヽ::::::;;::::: ;{ l. l l  l l l }ヽ:::::;;:::::;ノ
      {::::::ノt‐' i ,.l.-l、l   l,.l-l.、l {::::::ノ::::!
     `‐i |  l ,.--、  ,.--、 !.`「´ト‐'    / ̄ ̄ ̄ ̄ ̄ ̄ ̄ ̄ ̄ ̄ ̄ ̄
      ノ l.  ト '''' ┌┐ '''' イ. ! !  _l
    / . |  lニ`_‐-`_´-‐_´ニ!. | l  \  普通に微分しようよ〜♪
    i.  / !  |`‐{::::::ヽ./´:::::}‐'|  ! ゝ.   l
    ゝ. i. ,l  l 、t:::::水:::::r'  l  } 、 !   \____________
     jノレ'-ヽN ヽゝ::人::ノ  jハノュl_.ノ
     {_.. -‐|     |:::ハ:::|.   ト、  7
     K ,.-‐l    l´,、`!   l. ``〈-,
     `Lt  ,.|    「i 7   ト、,..-ニ.!
      ヽ`く._l -‐ l !l ヽ. レ' ,┘
        `~ t.L____」‐L__l.7~
         V , , , ,. }=! ,. , , ,V
           ト!JJJ~ , ~iJJJ'|
            l      !l    l
          .l─ --‐l l‐-- ─!
          .l:::::::::::::::l. l::::::::::::::l
         .l::::::::::::::l l::::::::::::::!
         ,!_/二ニ.} {.ニ二ヽ!、
           i ゝ-─-! !-─-ノ i
         .{_  ̄ ̄} { ̄ ̄ _}
           ̄ ̄´  ` ̄ ̄
212205:03/11/18 12:53
>>209
ありがとうございました。
213132人目の素数さん:03/11/18 12:55
>>209
答え違うだろ
もう少し丁寧にやろうよ
214205:03/11/18 12:56
>>211
もしよければ微分での解きかたを教えてください。
215132人目の素数さん:03/11/18 12:57
>>209
=(t+2)/(t^2 +1)
           t=tanθと置いて
=(tanθ+1) (cosθ)^2


=(tanθ+2) (cosθ)^2

だな。
216132人目の素数さん:03/11/18 13:01
>>214
           ,...,_
          /""`'・.,
         ,/'   ..::::\.,__,,,.....,,,__
        ,i'    ...::::::::"""    `''・-、.,_   _,,......,,,_
         i'  ...::::"           ":::`・'"~   :|
        /"                 " ::::  ...::,l
       ./'                     "::::::/
      ./'.                       ::"i,
      |.         \             .::::::|  f(X)をXについて微分するだけニダ
       |.                       .:::::::|   
      i;             i、.,    ,.    /...::::::::|
       '、            | ∨`"~'/      :::::::: | df(X)/dx=0となるのはx=-1+-√5ニダ
        \           l,    /     .::::::: ノ'  
         `/(          i.,_,/     ...:::::/  教えてあげたから
      ,.-'"~ ~"ー-.,,__          .....:::::::ノ').,   賠償ニダッ!!
     ./=ー'"~"`ー-.,_~"\-.,_  ,...,_ "";.-、::/ノ::: 'i,
    / ̄    :::::〜`i.():::\`""| `(^);;;;;|;;/;:::::::: 'i,
  . /         ::::::`i |:::::):;;;;;i、,/人_ノ;;;;'i,::::::::::: i
   |          .:::::::|/::::└ー-,;;;\;i;;;;|/;:::::::::::: i
   i.,_,.人       ..::::::/ :::::::::::::/;;;;;;;'i,'i;;;;|\;:::::::::::: i
      `;      .::...:::::/  :::::::::::`- .,_;;;;i,ノ;|;;;/;:::::::::::: /
      `、.,..:::::::::::::::::/   :::::::::::::::;;;;;;\,i ,/;;;;;:::::::::::|/
        `i"`-.,_,.ノ   ::::::::::::::::;;;;;;;;;;;\|;;;;::::::::::::|
217132人目の素数さん:03/11/18 13:01
(tanθ+2) (cosθ)^2
=cosθsinθ+2(cosθ)^2
=(1/2){sin(2θ)+ 2cos(2θ) +2}
=(1/2){(√5)sin(2θ+α) +2}

cosα=1/√5
sinα=2/√5
か。
218132人目の素数さん:03/11/18 13:03
>>205
こんな糞問題で変数変換、微分なんかす・る・な!
f(x)=(x+1)/(x^2-2x+2)=(x+1)/{(x+1)(x-3)+5}
f(-1)=0
x≠-1 のとき
f(x)=1/{x-3+5/(x+1)}=1/{x+1+5/(x+5)-4}
ここで、相加平均≧相乗平均 より
|x+1+5/(x+1)|≧2√5 ⇔ x+1+5/(x+1)≦-2√5、2√5≦x+1+5/(x+1)
⇔ x+1+5/(x+1)-4≦-4-2√5、-4+2√5≦x+1+5/(x+1)-4 (等号は x=-1±√5 のとき成立)
∴ 1/(-4-2√5)=-(√5-2)/2≦f(x)<0、0<f(x)≦1/(-4+2√5)=(√5+2)/2
以上より、最大値 f(-1+√5)=(√5+2)/2、最小値 f(-1-√5)=-(√5-2)/2
209の努力は偉大です。
だけどここは一つ別解を作ってみましょう
g(x)=(x^2-2x+2)/(x+1)を考えてみます。
x^2-2x-3+5=(x+1)(x-3)+5と考えると
(x^2-2x+2)/(x+1)=x-3+5/(x+1)
=(x+1)+5/(x+1)-4
t=x+1とおくと
t>0の時、t+5/t>=2√5 等号はt^2=5の時
t→-0で∞、t→-0で-∞
t<0の時は-(t+5/t)>=2√5 等号はt^2=5の時
∴-2√5>=t+5/t(t<0)
g(x)>=2√5-4 (x+1>0)
g(x)<=-2√5-4 (x+1<0)
f(x)=1/g(x)<=1/(2√5-4)(x+1>0)
f(x)=1/g(x)<=-1/(2√5+4)(x+1<0)
f(-1)=0
よって-(√5-2)/2<=f(x)<=(√5+2)/2
とでました。まちがってたらごめそ
220132人目の素数さん:03/11/18 13:06
>>218
逆に言われるとすれば、こんな糞問題で場合分けなどするな
ってところか?
221132人目の素数さん:03/11/18 13:07
また糞問題で盛り上がって来ますた
222132人目の素数さん:03/11/18 13:10
  ___,. - '":::::::::ヽ'---、.
   >..        ::::::::::::::::ヽ、
  /::::::::::::::::::::::::::::;/       ヽ、
  //:::::/::::/::::::/ / i:::::|:.:...:.. :.......:::::::ヽ、___
  /:::::/::::/|::::/ /  |:ヽ:::ヽ.ヽ::i:::::::::::::::::::<、  "ヽ、
 /| !::i:l::i:::レ_イ-|  |ヽ⊥ヽ|ヽ:i:::::::::::::::::::::::::三二ヽ、
 | ∨:::|::|-' |_      `ヽ、||::|::i::::ヽ::::::::::::::ヽ、 ヽ こんな問題しか解けない人もけっこういるの‥
  / |::::ヽィ",,i`    イ"‐、  レ|::::i::::::ヽ::_-、:::ゝ
  {  ! |ヽ}ゝ-'     ゝ'',,ノ` ノフ`i:::ヽ、:::::::::::::ゝ :ヽ、
  i! ,-ヽ:ヽ   _'_    ~ ;.__ 人ヽ:::ヽ<" ヽ、_  :ヽ、
  ,イ ::  ヽヽ、     ,. イ  ヽ、 ヽー--,.> ヽ、ニ;  :ヽ
  | ヽ ::.    `┬<''"  /       .:: / ヽ、ヽ  ヽ、
  ! ヽ ::.     { 'ヽ  /      .:: /_,...:--ヽ  ヽヽ
  ヽ ヽ ::.    ヽ   /      .:: /'"    <ヽ.  ヽ
  ヽ ヽ ::................ヽ ,./...........   .:: /  ___ /ヽ ヽ、 !
   ヽ ヽ____∀___ ::::: /  |ヽ/!  ヽ、 ヽ
    ヽ    ヽヽ__Y_ノ   "'t-'   ヽレ'i、  i ヽ-、
223132人目の素数さん:03/11/18 13:12
>>209のアイデアは悪くない。結果的に>>217の修正が入るけども。
>>218は実質的にxからx+1へ恋い焦がれ、それは変数変換と変らない情熱を感じる
>>219>>218の亜流か?

所で、誰も微分しようとしないのは何故?(w
224132人目の素数さん:03/11/18 13:14
隊長、こんどは評論家が現れますた
>>223

解けることは一目でわかるが、面白くないじゃんw
226132人目の素数さん:03/11/18 13:16
基本的に商の微分は、
間違いも多い 神経を使う 疲れる
227132人目の素数さん:03/11/18 13:25
>>225
微分しか方針が思い浮かびません
>>227
(x^2-2x+2)f(x)=x+1
の両辺をxで微分すると
(2x-2)f(x)+(x^2-2x+2)f'(x)=1
f'(x)={1-2(x-1)f(x)}/(x^2-2x+2)
x^2-2x+2>0(判別式は負)
f'(x)=0⇔1-2(x-1)f(x)=0
2(x^2-1)=(x^2-2x+2)よりx^2+2x-4=0,2x=-1±√5
グラフの概形を知っていれば、x=-(2+√5)/2
で最小値、x=(-1+√5)/2で最大値となることが
わかる。(詳細は自分で増減表を作って確認)
この時x^2-2x+2=x^2+2x-4-4x+6=-4x+6だから
f(x)=(x+1)/(6-4x)
これより最大値...(ry
229228:03/11/18 13:30
途中で思いっきり計算ミスあるんで、これは自分で一から書き直して
下さい。(もしくはどなたかもう一度清書お願いします。)
230132人目の素数さん:03/11/18 13:38
>>228
x=-1±√5だな。
もう清書はいいよ。
231おねがいしまつ:03/11/18 14:05
ある高校で全校生徒240人の身長をはかったところ
平均が168cmであった
標準偏差が3であったとき
身長が174cm以上と162cm以下の人は合わせて何人以下であるか?

↑おねがいしまつ。
232132人目の素数さん:03/11/18 15:14
生徒数が多いので身長の分布は正規分布と仮定できる。
平均をμ、標準偏差をσとすると(X-μ)/σはN(0,1)に従う。
P{(X≧174)∪(X≦162)}
=P[{(X-168)/3≧2}∪{(X-168)/3≦-2}]
=P{(X-168)/3≧2} + P{(X-168)/3≦-2}
=0.0228+0.0228 (正規分布表より)
=0.0456
240*0.0456 = 10.944
およそ11人
233ぼるじょあ ◆yEbBEcuFOU :03/11/18 16:07
>>231
解は一意に決まらないが、設問が「何人以下であるか」を問うているのなら(少なくとも文面上はそう解釈できる)、答えは出せる。

最も人数が多くなるのは、174cm以上の人と162cm以下の人はぴったり174cmまたは162cmであり、それ以外の人はぴったり168cmである場合。
このとき、174cm以上の人と162cm以下の人の人数をnとすると、174−168=168−162=6だから、
 分散=3²=6²×n÷240 ⇔ n=60
従って、60人以下。
234132人目の素数さん:03/11/18 16:19
>>231
チェビシェフの不等式 P(|X-μ|≧kσ)≦1/k^2 においてk=2 の場合だから
P(|X-μ|≧2σ)≦1/4
240 * (1/4) = 60 だから 60人以下
235132人目の素数さん:03/11/18 16:26
今日、雁を見ました。集団で飛んでました。
質問です。なぜ鳥は放物線状でなくy=2|x| みたいな形に並んで飛ぶんですか?
236132人目の素数さん:03/11/18 16:31
鬱鬱鬱鬱鬱鬱鬱鬱鬱鬱鬱愛凝鬱鬱鬱鬱鬱鬱鬱鬱鬱鬱鬱鬱
鬱鬱鬱鬱鬱鬱鬱鬱鬱鬱凝恋夾恋凝鬱鬱鬱鬱鬱鬱鬱鬱鬱鬱
鬱鬱鬱鬱鬱鬱鬱鬱凝愛二二二三三夾凝鬱鬱鬱鬱鬱鬱鬱鬱
鬱鬱鬱鬱鬱鬱鬱夾二一一一二二二三夾鬱鬱鬱鬱鬱鬱鬱鬱
鬱鬱鬱鬱鬱鬱恋三一一一一二二二三三愛鬱鬱鬱鬱鬱鬱鬱
鬱鬱鬱鬱鬱凝夾三一一一一二二二三三夾鬱鬱鬱鬱鬱鬱鬱
鬱鬱鬱鬱鬱愛夾二二一一一一一二二二三鬱鬱鬱鬱鬱鬱鬱
鬱鬱鬱鬱鬱恋三二一一一一一一夾恋恋夾恋鬱鬱鬱鬱鬱鬱
鬱鬱鬱鬱鬱三二二二二二二三恋恋三三夾三鬱鬱鬱鬱鬱鬱
鬱鬱鬱鬱愛三恋凝愛夾三二夾夾愛鬱鬱夾三凝鬱鬱鬱鬱鬱
鬱鬱鬱鬱恋夾三夾恋夾夾二三夾夾恋三二三夾凝鬱愛三一
鬱鬱鬱凝恋夾凝鬱凝夾三二三二二二二二二夾夾鬱愛二二
鬱鬱鬱愛恋夾夾夾三一三二二三二一一二三夾三鬱愛一一
鬱鬱鬱愛夾三二二二一三三一三三一一二三夾夾鬱鬱一一
鬱鬱鬱凝夾三二一一一三夾一二夾三二三三恋鬱鬱鬱鬱鬱
鬱鬱鬱鬱恋夾三二一二夾夾一一三三三三夾恋鬱鬱鬱鬱鬱
鬱鬱愛鬱凝恋三二二三夾恋恋恋鬱二三夾恋恋鬱鬱鬱鬱鬱
鬱鬱鬱愛鬱凝恋夾三三三愛愛愛三二夾夾夾恋鬱鬱鬱鬱凝
鬱鬱鬱鬱鬱鬱凝愛恋夾三三二二三夾愛恋恋愛鬱鬱鬱鬱愛
夾愛鬱鬱鬱鬱凝凝愛恋夾恋愛愛凝鬱凝夾恋鬱凝鬱鬱凝愛
鬱凝鬱鬱鬱鬱凝愛恋恋鬱鬱鬱鬱愛夾恋夾恋凝凝愛凝凝愛
夾夾恋鬱鬱鬱鬱凝凝恋恋愛夾三二恋恋夾愛凝愛愛凝鬱凝
恋愛鬱鬱鬱鬱鬱鬱凝凝恋恋鬱鬱鬱恋三恋三鬱凝愛凝鬱凝
鬱鬱鬱鬱鬱鬱鬱愛鬱凝愛三三二二三恋愛一鬱凝愛凝鬱愛
凝凝鬱鬱鬱鬱鬱恋凝鬱凝愛恋夾恋愛鬱一二凝愛愛愛鬱愛
凝凝凝鬱鬱鬱鬱二一鬱鬱鬱鬱凝鬱鬱三一三凝愛凝愛凝凝
愛凝凝鬱鬱凝鬱一一恋鬱鬱鬱鬱鬱夾一一三凝愛凝凝愛凝
凝凝凝鬱凝鬱鬱一一一鬱鬱鬱鬱凝二一一夾愛凝愛愛愛凝
237132人目の素数さん:03/11/18 16:40
a,bは等式 a^2-2ab+4b^2=0 を満たす0でない複素数とする。
(1)a/bを極形式で表せ。
(2)複素数平面上で、3点0,a,bを頂点とする三角形は、どのような三角形か。

お願いします。
238132人目の素数さん:03/11/18 16:48
>>237
                                   ,. -ー-、
                         _,.. -,.='''''''' ー- 、_,/  _,,.,-ヽ,
                      ,.-彡,.-',...ミ_  -、 '_,く  / _ノ-ニ_`>
                     /_,/.i- 、く   ~`ヽ、  、.\,.彡'ム,∠..,,_ 散々既出だよ
                    /r‐./  l  \\    \ ヽ  r'゙,,.-‐''゙~ l, 三角定規を思い出してね
                ,r一‐-'/, ' l   l.ヾ 、ヾ 、\   ヽ \ ヾ, .,._=ニニ=l,
               ノ'''゙゙゙゙`// .|  | l ヽヽヽ,ヽ _ヾ、  ヾ, ヽ. `ーtヮ-'_ノ|
              /,. -─/ l  |  |'、 l、ヽ,ヽ V\_ヽゝ  l'、 | l r'`゙゙゙~ ,. ヘ
               ̄>-| |l  |  .|ヽ l\ヽ'、i /f"{、)`ヾ.}.|ノへ-_‐;ニ-t'゙,,
              ∠~-'''''| | .l .ヾ、 ,キ' ヾ、 `ヾjl. ゙ ヽ  キ ヒミ、},-、__,...ヅ!
               | _,,....l .l | ヽ  V、_lr゙,-'、      \_ヅ 〉リヾ、-‐ニ-'ノ
               ゙-_つヾ、ヽヽ `ヽキ t‐゙ \          lン'`!| L.. -'"
                )ー--`ヾ、t、`ミ!、` ヽ、_ヅ ' ,. ヘ.   /"  j,!
                (__-_-一'''ツヽ~|.|'`ヽ、    ヽ、. }  ,イ__
                 r-‐゙''_'.ヅ  .|.l`'ー-ゝ、...,,,______"/ |_ <`lニ!''''''''''''''/ニ、ー、_
                 Lミ_`ー=ヮ  ヾ、.  ,ム,,,,,,,/~コ     ヽi ヽt--,-,,,,_ l",/ l  ゙`‐ 、 _
239132人目の素数さん:03/11/18 16:50
このスレのAAを見てると、ヤル気が沸くのは何故?
240132人目の素数さん:03/11/18 16:54
>>239
あなたがロリであるという可能性があるのではないでしょうか

数ヲタにはロリコンしかいないの? Part2
http://science.2ch.net/test/read.cgi/math/1058287962/
ここにいってみたらどうでしょうか
241132人目の素数さん:03/11/18 16:54
>>237
(1) a^2-2ab+4b^2=0、a≠0、b≠0 ⇔ (a/b)^2-2(a/b)+4=0、a≠0 ⇔ a/b=1±√3*i=2{cos(60゚)+i*sin(60゚)}、2{cos(300゚)+i*sin(300゚)}
(2) A(a)、B(b)とすると、(1)の結果より arg(a/b)=±60゚、|a/b|=2 
 よって、∠OAB=90゚ または ∠OBA=90゚、∠AOB=60゚ の直角三角形。
x2-3x-1=0
二次方程式の解き方を教えてください
243237:03/11/18 17:07
>>241
ありがとうございました。
244132人目の素数さん:03/11/18 17:27
>>242
マジで訊いてんのか?
一般に2次方程式の解は次のように求められる。
ax^2+bx+c=0 (a≠0) ⇔ a{x+b/(2a)}^2-b^2/(4a)+c=0 ⇔ {x+b/(2a)}^2-(b^2-4ac)/(4a^2)=0
 ⇔ {x+b/(2a)}^2-{√(b^2-4ac)/(2a)}^2=0 ⇔ {x+b/(2a)-√(b^2-4ac)/(2a)}{x+b/(2a)-√(b^2-4ac)/(2a)}=0
 ⇔ [x-{-b+√(b^2-4ac)}/(2a)][x-{-b-√(b^2-4ac)}/(2a)]=0 
 ⇔ x={-b±√(b^2-4ac)}/(2a)    ← これを2次方程式の解の公式という。絶対憶えておけ!!

x^2-3x-1=0 は上の場合の a=1、b=-3、c=-1 のときだから、その解は
x=[-(-3)±√{(-3)^2-4*1*(-1)}]/(2*1)=(3±√13)/2
245132人目の素数さん:03/11/18 17:34
>>244
    /       ヽ! ! l|ト、 ヾ、ヽ!l|   ヾ、!ヽ、ヽ、\  ハヽ、\
    / _        レ' ! l !ヾ、 ト、|l !    ||ト、!| l|  ヾ、/ト、 |l !ー- 、
  ..... ヽ、'i       /イ! l !_,.-ヽ!‐-|l l   l|-、ll,_ |l   |lヽ!、ヾ!| `、 `゙゙''ー--、
 :  :. ヽ!      //!l|´!|、_,,,,,,._ヽ !|l|   l|  |lヾ!ト、ヾ、|ハ,}  lト! ト、     ヽ .......
....::.....:  / i      // ト、ゥ!i'" Ol!゙ l |ll   l|  || l!|}`! i l|ヾ、 |! i | `!'"    / :: ...:: .... マジレスかっこわるい
   /〃 !     // /ノ《 l!;;;:::::j}  l |l!   '  ! !|! j l l!   || l! l |!____,  /  ::.. ..::....:
  / ,イ /l    L! / { ,ゞ、'ニン-      -==='ュ_,/| |l,ハ\、l!/_ヾ、 ヽ   /    ::......::
  〈 {ー、!' l      / !ニi ´ ̄    '       `゙,'- ! |lノi ̄ |!   \`ヽー-、_______
.... `ー-ニ三二ニ‐-、 /  lー'、    ー-一       /一'! l{//  ヾ、    `ーニ三二ニニ`ヽ、
 ::...   `ーニ三、ヾヽ、  \.\          / _/ヽヾ/    ヾ、  / `ー-‐‐‐-、ヾ、〉
...: ::     l  ヾ! lリ }  , i ヽ_,,.-‐‐┐   ,. ィ'´ ̄ /   ヾゝ     \/      ,.--‐ゥノハ
.......:: __________,,!,-‐シノ,イ   l ヽ、  l` ' ´   ! ,/ ヽ  /       /`゙''ー== // ̄フ//ノ
  r!二三ニニニノ-一'  i / / ヽ  !    ゝ/    ヽ/    ___,∠_____    iレ!____ ̄´
  L三二ニ=一i    l l /    ヽ !  __r'"´  /  _,,!--一' _,.‐‐‐‐‐'    |川レ'_//フ __
 r--\->,  /    !l / i ┌-`-┬┬-‐‐┐j //! ̄ ̄,/ ...........     `ー| i //__〃ヘ
 Fニ三ン//j  ヽ,   /  /    ̄フ/ ̄| 「ヽ二二_/ |   / ::  ....:: .....     L!/ ル' // ,ク
 L三二ン´    iヽ /   /    //| O | |  /     ヽ、/  ::...: ...::   ::    ヾリ/-レ'
246132人目の素数さん:03/11/18 17:52
.|         \
  .|           i  \    、 ヽ、
  .!      i.  | |i  i, .\   \  \    i
  l    l  |  .|. |l,  l. ヽ \   \  ヽ、  l
  i     |  |   !  l l, ヽ ヽ \、  ヽ  ヽ. ト、
  |    .|  | ., |i,. l ヽ ヽ、\、 \ 、.  \ \! ヽ
  |    .|  | .| | l, l,. \ ヾ:、 ヽ;.、 ヽ`ヽ、 ヽ |'ト
  |    .|  .| l. | ヽ__l、,,,,ゝ、ヽ ヽ、\ヽ、\ ` ヽ|.l
  |;    l、 _,」.;┼l'´,.=;_テ;ー;ミヽゝ    ヾ ``    | l
  |i     ヾ、 f-‐|く.  i、,.゙_ノ              .| l
  !|     ;ト、ヽ、ヽ``"´               l. l
  !|     { rベ.l`                  l |
  l |!     ヽ\ ヽ                       |.!
  | |l      ヽ、. \         ` ´ __    .!    真面目な人間が疎まれる世の中なんて・・・
  | |.!     ,    i``^ヽ       ,. -‐ ''´     ,1
  | .!.!     |i   |    ` 、            / |
  | !.|     .|.!    |.   i   l`ト.、          /  !
  .! .| l,    | !.   |   .|i,  l, |:::::::`:::::..-.-‐:ク′  |
  .| !. l.   | l    |!   l.l.  l, !::::::::::::::::::::/  __,」
  l | l   |. l,.  |l,.  _」,L.-‐'T!::_;;:: --‐''' ´    .| /
   l. |. l i,   | l.  |l;  lー '' " ´          | /
   ヽ, ヾ:,  |i, ヽ  ||i, |                 j/
       ヽ. |l, ヽ. |.l  |
        \.lヽ. ヽ! __」
          ヽ   /
           ,/
247132人目の素数さん:03/11/18 17:53

      _, -‐ ''" ̄ ̄`゙゙''ー-、__    /ソ、
       .,/^            `'ヽ/   ̄ フ
     ./                ヽ、   /、
     /                   ヽ   ┴ー- 
    /   /   /    /| l       ヾ     ., ヽ
   ,i´  / /  / ハ  /l /| / \     |     ヽ-丶
   .l゙  / ./  / | ./ | ./ | / |/‐‐- ヽ    │.\ ┬-ゝ
  ,l゙  / // /-‐レ  レ` レ′     ヽ    |\ヽ 
  |  .l゙ ,!レ/          rテミヽ、 l    !ノ\|
  |  l゙ l゙./ rテミ、      ト-';;;;;ハ |   .|
  ゙l  .|  | / ト-';;;;i       っ;;;;;ノ  |   .|
   |  l゙ ゙l   っ;;;;;ノ       `ー"  |   .|   
   ,!  | .|   `ー"           ノイ  │     
   │ │ |,゙l、      `        /"l゙ | l   あたしは真面目な人 好きですよ♪
   ゙l .l゙ { ゙\     `ー-‐'    /  | │ |
   .l゙ .゙l l゙  ゙'r,,,、       ,ィ'"   .l゙ │ !
    ゙l  ゙l l   |/  `|''ー-ー-''" |    l゙ .l゙ ,l
    ヽ ゙l゙l,  ヽ   |      |    .l゙ ," /
     ヽ ゙l゙l     ノ       !、  丿.l゙ /
      ゙l、l゙l._,,,、'"`        `゙''‐/ .l゙ /
      _,゙l, l゙l             | │(,、,、  
   .,..-‐'゙l゙゙l,゙l゙l  、 __      ‐-- | l゙ .|//`゛'''-、
 /`   l.| /              | .| |/     ヽ
.│     l/               | | |       l
│     ノ\       , '    _,,/ ヽ| |      │
│    ,/    ̄ー-ー‐---一'''"゙´    `ヽ      l
: ゙l   |,/    `ーーー、             ` l    l
 ゙l   l゙                      |    ,!
 .|   .|                         l゙    l
  |   ゙l                     ,l゙    |
数学Aの三角形の性質を
利用した証明問題見たいなのが分かりません。

3-4問あるんですが1000円くらいで誰かやってくれませんか
suugakuwakaran@hotmail.com
これでメッセつけとくので誰かお願いします。
小学校6年生の問題なんですけど
http://www53.tok2.com/home2/bgzlsd/gtfzxm/question.gif
上の図形で、灰色の部分の面積はどうやって求めたらいいのでしょうか?
250248:03/11/18 18:25
問題を見てやるか決めても構いません
助けてください(ぅ_・)
>>244
ダメダ・・・・ワラカン・・・・
オレハモウダメダ
252132人目の素数さん:03/11/18 18:28
>>249
これもコピペかと疑いながら・・・

角度30度の扇形の面積は求められる?
まずはそれからだ。
>>249
扇形の中心をO,弧上の点を左上から右下へ順にA,B,C,D,
BからODへの垂線の足をE,CからODへの垂線の足をF,
OCとBEの交点をGとおくよー。
OB=CO=6,∠OBE=∠COD=30°,∠BOE=∠OCD=60°だから
△OBE≡△CODだねー。よって求める面積はー、
(扇形OBC)−(三角形OBG)+(四角形CGEF)
=(扇形OBC)−(三角形OBE)+(三角形COD)
=(扇形OBC)=6*6*π*30/360=3π
だーよ。
254132人目の素数さん:03/11/18 18:39
>>248
そんな、金でつらんでも
ここに書けば誰かやってくれるよ
255249:03/11/18 18:43
>>252
ヒントありがとうございます。

>>253
詳細な解説ありがとうございます。

最後は扇形の面積になるというところがおもしろいと思いました。
どうもありがとうございました<(_ _)>
256248:03/11/18 18:44
ほんとうですか、でも学校の宿題見たいなもんで、
別に数学に興味があるわけでないので悪いかなーって
思ったんですけど。

ちょっと問題あっぷしてみます。
257248:03/11/18 18:56
http://kame.kakiko.com/suugaku/

UPしてみました。
うーん、まったく意味不明だ数学落とすと進学できないから
勉強しないとなぁ・・
258132人目の素数さん:03/11/18 18:57
>>256
全然構いません。
259132人目の素数さん:03/11/18 19:03
12個のニット帽があり、一つだけ重さが違う(軽いか重いかはわからない)
この重さの違うニット帽を天秤を3回だけ使って見つける方法は?
260hansin:03/11/18 19:07
(1) ∫e^ax・sinbxdx を求めよ。
(2) f(x)=sin2x・cosxのとき、f^n(x)をもとめよ。
以上よろしくお願いします。
261132人目の素数さん:03/11/18 19:09
@最初に6個ずつ量り、A重い方のグループをさらに3個ずつに分けて量る。
Bさらにその重い方のグループ(仮にA,B,Cとする)のAとBを量り、どちらかが重ければ
それが答え。もし両方同じ重さならCがもっとも重いニット帽となる。
>>259
「天秤 3回」でぐぐれ
263132人目の素数さん:03/11/18 19:10
>>257
多分、1問目はオイラー線の問題
Hは垂心
Oが外心
重心をG

OGHは一直線上にあって(これがオイラー線)
OG:GH=1:2
なんだけど、これの証明は

Aを通るBCと平行な直線
Bを通るCAと平行な直線
Cを通るABと平行な直線

を引いて作られる大きな三角形を考える。
そうするとこれはABCと重心が同じ
辺の長さとか見て貰えば分かるとおり
2倍に拡大されたもの。

拡大の中心はG。

で、大きな三角形から見るとA、B、Cは各辺の中点だから
Hは大きな三角形の外心。

外心OからABに下ろした垂線はどうなってるかっていったら
AHになってるわけですよ。で、AH=2OM
264五歳児:03/11/18 19:10
√50−√2= がわかりません・・・
265132人目の素数さん:03/11/18 19:10
っつーか。軽いかもしれんのね。大失敗
>>261
問題をよく読め
267五歳児:03/11/18 19:12
√って難しいですね・・・・
268132人目の素数さん:03/11/18 19:14
>>257
2問目は、問題がおかしい
PQ//ADは、ベクトルや座標つかえばすぐわかることだけど
Mがどう関係してるのか?
269132人目の素数さん:03/11/18 19:19
>>257
3問目の1は
Iは内心なのでAIは角Aの二等分線
∠BAD=∠DAC
円周角と思えば、それに対応する弦の長さは等しいので
BD=DC
270132人目の素数さん:03/11/18 19:24
>>260
(1)(e^ax*sinbx) ' = a*e^ax*sinbx + b*e^ax*cosbx ・・・@
(e^ax*cosbx) ' = a*e^ax*cosbx - b*e^ax*sinbx ・・・A
@*a - A*b
a(e^ax*sinbx) ' - b(e^ax*cosbx) ' = (a^2+b^2)e^ax*sinbx
両辺を(a^2+b^2)で割って積分すると
∫e^ax・sinbxdx = {a/(a^2+b^2)}e^ax*sinbx - {b/(a^2+b^2)}e^ax*cosbx + C
271132人目の素数さん:03/11/18 19:25
>>257
3問目の2は

円周角を考えて
∠DAC=∠DBC
AI, BIは二等分線なので
∠IBA=∠IBC
∠IAB=∠IAC

△BIAの外角を考えて
∠BID=∠IBA+∠IAB=∠IBC+∠DBC=∠IDB
よって△IBDは二等辺三角形で
BD=DI
272五歳児:03/11/18 19:33
√8/√2=がわかりません、教えてください
273248:03/11/18 19:33
ほんとにありがとうございます。
でも実はあまり理解できてないので、
ちょっと今から検索とかして調べて理解できるように
頑張ってみます。

2問目は分からないです(ぅ_・)
というか本当に馬鹿で悪いんですが
PQ//ADってどのような意味なんですか?
検索できないので、、
>>272
2
275132人目の素数さん:03/11/18 19:51
>>257
1問目
直径に立つ円周角は直角より AD‖FC、CE‖FA
したがって四辺形AHCFは平行四辺形だから AH=CF
CF=(1/2)OM より AH=(1/2)OM
【発展】
AMとOHの交点をGとすると、三角形OMG∽三角形HAG だが
AH=(1/2)OM だから MG:GA=OG:GH=1:2
つまり、点Gは三角形ABCの重心であり、3点O、G、Hは同一直線上にあり、
点Gは線分OHを 1:2 に内分している。

2問目
重心の性質より AP:PM=DQ:QM=2:1
よって平行線の性質より AD‖PQ

3問目
(前半)
内心定理より ∠BAD=∠CAD
円周角の性質より ∠BAD=∠BCD、∠CAD=∠CBD
∴ ∠BCD=∠CBD
二等辺三角形の性質より BD=DC
(後半)
三角形BIAの外角より ∠BID=∠BAD+∠ABI
内心定理より ∠ABI=∠CBI、∠BAD=∠CAD
円周角の性質より ∠CAD=∠CBD
∴ ∠BID=∠CBD+∠CBI=∠DBI
二等辺三角形の性質より BD=DI
276132人目の素数さん:03/11/18 19:52
>>273
PQとADが
平行って意味
>>273
検索するより教科書読んだ方がいい気がする。
278132人目の素数さん:03/11/18 19:59
教科書より参考書だな
でも、検索でもそれなりにいいものがあると思うよ
279132人目の素数さん:03/11/18 20:03
(Q)二次方程式、2ax~2-(a+2)x-5=0 の1つの解が-1と0の間にあり、
  他の解が2と3の間にある。ただし、a>0とする。
  この条件を満たすような定数aの値の範囲を求めよ。

f(-1)*f(0) < 0 ⇒ a > 7/3
f(2)*f(3) < 0 ⇒ 11/15 < a < 3/2
となって、
共通部分が出てきません。。。
どこら辺で間違ってるんでしょうか。。。

正答は、1 < a < 3/2 となってます。
280248:03/11/18 20:04
みなさん、ありがとうございます。
少し必死になってみるので
また分からなくなるか、できたら書き込みしてみます。
281132人目の素数さん:03/11/18 20:10
>>279
グラフを考えてみると

f(-1) = 3a-3 >0
f(0) = -5 <0
f(2) = 6a-9 <0
f(3) = 15a-11 <0

なので、

a > 7/3ってのが計算ミスだろう
282132人目の素数さん:03/11/18 20:12
exp{-a(x-λ)^2}をλについて展開しろって言われたら
いったい何をすればいいんですか?
283132人目の素数さん:03/11/18 20:19
>>282
λの級数で表す。

a0 + a1 λ + a3 λ^2 + …
みたいに
284282:03/11/18 20:26
>>283ありがとー!
285173:03/11/18 22:18
>>174
例えば、いわゆる「一発屋」の芸能人の人気が時間と共にどう変化するか、
ということを、関数を用いてモデル化、グラフでその現象を近似したかったんです。
286132人目の素数さん:03/11/18 22:22
どこまでがネタで
どこからが現実なのか
分からなくなるよ
287132人目の素数さん:03/11/18 22:23
新しく質問です。
加重平均値とはなんでしょうか?
パチンコの平均現金投資額を求めるのに使えるみたいですが、ヤフー検索かけても
公式どころか意味すらわかりません。
基本概念と公式?的なものを教えて欲しいです。
お願いします。
288132人目の素数さん:03/11/18 22:28
放物線:y^2=4x の(0.0)から(1.2)までの長さを求めたいのですが
教えてください・・・
>>288
教科書読んで積分汁。
>>287
普通の平均はどれも同じだけの「重み」だと思って平均するが
加重平均は、それぞれに別々の(同じのがあっても構わんが)「重み」を
つけて平均を取る。ただそれだけだ。
292132人目の素数さん:03/11/18 22:39
>>291
ヤフーは, googleと提携してるんじゃなかったっけ?
293132人目の素数さん:03/11/18 22:53
ヤフー、Googleと提携

2001年4月2日

ヤフー(株)は1日、米Google社と提携し、検索ポータルサイト“Yahoo! JAPAN”において、
全文検索型エンジンの検索結果を併せて表示する検索サービスを開始したと発表した。

これにより、カテゴリー型検索と、データベースを対象とするキーワード検索が行なえる
“Yahoo! JAPAN”の検索サービスで、入力したキーワードがデータベースに一致しない場合、
自動的に“Google”による検索を実行、その結果をYahoo! JAPANとGoogleのコブランディング
ページに表示する。同社では、従来と同じ操作方法で、Googleの膨大なウェブ情報にアクセス
できるようになるとしている。

http://ascii24.com/news/i/serv/article/2001/04/02/624859-000.html
Yahoo! って ps ファイルとか引っかかったっけ?
295132人目の素数さん:03/11/18 23:03
(1)方程式x^2+4xy+5y^2-8y+16=0を満たす実数,yの値を求めよ。

(2)有理数a,bに対して√(3+a)√3=a+b√3が成り立つとき、aとbの値を求めよ。
       二重根号です↑

満たすとか、どう式を立てればよいのでしょうか?
解き方を誰か教えていただけないでしょうか、お願いします。
296132人目の素数さん:03/11/18 23:09
次の面積分を求めよ。
∫(x+y+z)dS s:2x+2y+z=4 x>=0 y>=0 z>=0

まず、r↑=xi↑+yj↑+(-2x+-2y+4)k↑
∂/∂x r↑=i↑-2k↑
∂/∂y r↑=j↑-2k↑

|(∂/∂x r↑)×(∂/∂y r↑)|=3

また、z>=0より、-2x-2y+4>=0
x+y<=4
以上のことを考慮すると、求める値は、
∫(0〜2)∫(0〜2) 3(x+y-2x-2y+4)dxdy
となり、これを計算すると、24になると思うのですが、解凍をみると、16になるらしいのです。
どこが間違っているのか教えてください。
>>295
あちこちで同じこと書くな。広義のマルチだ。
(1)
>実数,yの値を求めよ。
「実数x,yの値を求めよ。」の打ち間違いと解釈する。
0=x^2+4xy+5y^2-8y+16=(x+2y)^2+(y-4)^2 ⇔ x+2y=0, y-4=0 ⇔ x=-8, y=4

(2)
>√(3+a)√3=a+b√3
どこが二重根号かさっぱり分からないが、強引に「√{3+a√3}=a+b√3」の意味と解釈する。
√{3+a√3}=a+b√3 ⇔ 3+a√3=a^2+3b^2+2ab√3 ⇔ a^2+3b^2=3, 2ab=a
⇔ a=3/2, b=1/2
298296:03/11/18 23:22
ちょっと変なところを書き直してみました。
次の面積分を求めよ。
∫(x+y+z)dS S:2x+2y+z=4 x>=0 y>=0 z>=0

まず、r↑=xi↑+yj↑+(-2x+-2y+4)k↑
∂ r↑/∂x =i↑-2k↑
∂ r↑/∂y=j↑-2k↑

|(∂r↑/∂x )×(∂r↑/∂y)|=3

また、z>=0より、-2x-2y+4>=0
x+y<=4
以上のことを考慮すると、求める値は、
∫(0〜2)∫(0〜2) 3(x+y-2x-2y+4)dxdy
となり、これを計算すると、24になると思うのですが、解凍をみると、16になるらしいのです。
どこが間違っているのか教えてください。

299248:03/11/18 23:38
やっと解けましたです。
円周角の性質とかは中学校で習ったものとして
教科書に載ってなかったので多分聞かないと
一生分からなかった予感です(ぅ_・)

レスくれた皆さんどうもありがとうございましたm(__)m
300296=298:03/11/18 23:54
誰か教えてください。おながいします。
301132人目の素数さん:03/11/19 00:11
>>298
ほとんど読んでないけど
一つ気になる点がある。

x+y<=4で、x≧0, y≧0
という領域をxy座標平面に書いてミロ
その重積分でやってる正方形じゃないぞ
302132人目の素数さん:03/11/19 00:14
>>298
yの積分範囲は 0 〜 2-x としないといけない。
303132人目の素数さん:03/11/19 00:21
cos11°< 0.99 を三角関数豹を用いずに示せ。

うまくいきませんが、おねがいします。
304132人目の素数さん:03/11/19 00:25
三角関数豹って上野動物園のあれか?
305132人目の素数さん:03/11/19 00:28
>>303
cosのテイラー展開に入れれば?
306296=298:03/11/19 00:29
>>301
>>302
なるほど。ありがとうございます。
それで計算しなおしたんですが、∫(0〜2)∫(0〜2-y) 3(x+y-2x-2y+4)dxdy
それでも答えは、24になる気がするのですが、解凍は16なんです。どうしてでしょう?
307132人目の素数さん:03/11/19 00:36
>>306
∫(0〜2-x) 3(x+y-2x-2y+4) dx
=3[(4-x)y-y^2/2] [y=0, 2-x]
=3(4-x)(2-x)-(3/2)*(2-x)^2
=(3/2)*(x-2)^2-6(x-2)
∫(0〜2) {(3/2)*(x-2)^2-6(x-2)}dx
=[(x-2)^3/2-3(x-2)^2] [0, 2]
=4+12
=16
308132人目の素数さん:03/11/19 00:36
ローラン展開の使い方教えてください!
具体的に。お願いします!
>>308
教科書嫁。
310296=298:03/11/19 00:37
>>307
ありがとう。すんませんでした。
311307:03/11/19 00:37
>∫(0〜2-x) 3(x+y-2x-2y+4) dx
∫(0〜2-x) 3(x+y-2x-2y+4) dy
の間違い。
312132人目の素数さん:03/11/19 00:38
>>308
何に使うの?
313132人目の素数さん:03/11/19 00:41
教科書、証明しか書いてなくて、実際に使い方わかんないんです。
結局テイラー展開とかで解けちゃうけど、解けない時もありますよね?
>>313
ステイトメントと証明をきちんと嫁。そしたら判る。
>>313
喪舞、なんか誤解してる。
316132人目の素数さん:03/11/19 00:54
>>313
言ってることが全くわからないんだけど
何が解けないと言いたいの?

317132人目の素数さん:03/11/19 01:01
>>313はかなり重度の知能障害を抱えているような気がする
318132人目の素数さん:03/11/19 01:34
聡明な皆様に質問です。よろしくお願いします。
XY平面状に4つ以上の点、(x1,y1),(x2,y2),(x3,y3),(x4,y4),・・・(xn,yn)があります。
すべての点を内部に持ち、半径が最小の円とその中心を求めるにはどうしたらいいでしょう。
最初は簡単かと思ったら、私の頭では良い方法が見つからなくて困ってます。
3つの点のすべての組み合わせで円を求めて、その中の最小の半径の円を
拾えば出来ることはわかるのですが、頭悪い方法ですよね。
>3つの点のすべての組み合わせで円を求めて、
>その中の最小の半径の円を拾えば出来ることはわかる

「最大」の間違いだとは思うが、それが求める円になるとは
限らない。
>>319
どうもです。そうですね。
一つ一つの円の中に、すべての点が含まれているかどうか、
いちいち検証しなければいけませんよね。
めんどくさー。何か良い方法は無いですかね〜。
321132人目の素数さん:03/11/19 01:50
I,JがRのイデアルのとき、I∪Jがイデアルとならない例を教えてください。
それと次の証明も教えてください(I,J,Kはイデアル)
1. I・J⊂I∩J⊂I⊂I+J
2. (I+J)∩K=I∩K+J∩K
>>321
R = Z (有理整数環), I = (2), J = (3) のとき I ∪ J は 2 + 3 = 5 を含まん.
残りは教科書を嫁.
>>318
点の数は有限とする。
2点間の距離の最大を与える2点をA、Bとする。
ABの中点をMとし、Mとの距離の最小値を与える点をCとする。

C=AまたはC=Bのときは、ABを直径とする円を描けば終わり。
そうでないときは、ABCに外接する円を描く。

でどうだろう。
最後の1行が心配。本当にその中に全ての点が
含まれるかどうか、必ずしも明らかではない。
324おねがいしまつ:03/11/19 02:35
>>231
>ある高校で全校生徒240人の身長をはかったところ平均が168cmであった
>標準偏差が3であったとき
>身長が174cm以上と162cm以下の人は合わせて何人以下であるか?

↑おねがいしまつ。

>>232
240*0.0456 = 10.944
およそ11人

>>233>>234
60人以下

どっちが正しい?
>>318
平面はn個の領域D1...Dnに分割される。
ここでDi={(x,y)| d((x,y),(xi,yi))>=d((x,y),(xj,yj))(j=1..n)}
つまりDiに属する点は点Xiからの距離が恒に他の点Xjからの距離よりも
大きい。
この領域の形は恐らく区分的な直線で構成されたものだと思うが、きちんと
調べていない。勢力圏図(→ボロノイ図とかで検索して)という分野があって
そこでは、似た領域で
Si={(x,y)| d((x,y),(xi,yi))<=d((x,y),(xj,yj))(j=1..n)}
という領域が考えるが、Siは区分的な線分で構成された領域(多角形も
含む)になっている。
今欲しいのは、DiとXiの距離が最小のiとその最小を与えるDiの点
Diさえ分かれば簡単にわかるのだが....
Di(i=1..n)を数式で表すことも出来る.
(max(a,b)を絶対値で表すことによる)がコンピュータ計算向けだと
思う。
>>323
ちょっと考えたぐらいでは反例が思い浮かびませんね。
使えそうですね。証明にチャレンジしてみます。
>>325
こ、これはむつかすぃ・・・
先ずは絵を描いてイメージをつかまないと頭パニックです。
コンピュータ向けですと、私の理解の範疇ではしらみつぶし法です。
ボロノイ図、調べてみます。

どうもありがとうございます。
私自身も、もうちょっと考えてみます。
327 ◆HACK/Ayu.k :03/11/19 07:33

うろ覚えの問題なので間違ってるかもしれないのですが、

x^2 + y^2 - xy = 0



x^2 + y^2 - xy = y

のどちらかだと思うのですが、この場合、最大値と最小値を求めるには
どうすればよいのでしょうか?
うろ覚えすぎみたいですね。
>>327
x^2 + y^2 - xy = 0だったら
(x-(1/2)y)^2 +(3/4)y^2=0
なので

x-(1/2)y=y=0よりx=y=0

x^2 + y^2 - xy = yだったら
(x-(1/2)y)^2 +(3/4)y^2 -y=0
(x-(1/2)y)^2 +(3/4)(y -(2/3))^2=1/3

p=x-(1/2)y
q=y-(2/3)
とでも変数変換すれば
よく見る楕円の式


二次関数なので、(楕)円、双曲線、放物線、2直線の積、点
のいずれかになるので適当に変数変換すれば
330327:03/11/19 09:36
>>329 ありがとうございました。
331132人目の素数さん:03/11/19 09:56
曲線Cをy=−xで定義する。xの増加する向きをcの向きとする。以下の積分を計算せよ。
∫C(1/(z^2−i))dz
お願いします。
332132人目の素数さん:03/11/19 09:57
関数f(z)は任意のz1、z2に対し、(z∈C)
f(z1+z2)=f(z1)f(z2)
を満たし、かつz=0でf(0)=f‘(0)=1が成り立つとする。
f(z)を見出せ。(f(z)=u(x、y)+iv(x、y)の満たす微分方程式を解け)
日本が誇る天才数学者の皆さん、お願いします。
333132人目の素数さん:03/11/19 10:11
>331
部分分数分解した後、留数定理
334質問です!:03/11/19 10:14
3x−5x+2の関数を微分するとどうなるんですか?
335132人目の素数さん:03/11/19 10:15
>>332
どうみても
f(z)=exp(z)
336132人目の素数さん:03/11/19 10:15
>>334
微分するまえに
式をまとめろ
337質問です!:03/11/19 10:20
質問まちがえました!
Y=X2乗−3X2乗+2の関数を微分するとどうなりますか?
338質問です!:03/11/19 10:21
また質問まちがえました!
Y=エックス3乗−3エックス2乗+2の関数を微分するとどうなりますか?
339質問です!:03/11/19 10:25
さっきの質問の答えは
Y’=3エックス2乗−6で合ってますか?
340132人目の素数さん:03/11/19 10:31
>>338
y = x^3 -3x^2 +2
y = 3x^2 -6x
341132人目の素数さん:03/11/19 10:31
>>338
しまた
y = x^3 -3x^2 +2
y' = 3x^2 -6x
342132人目の素数さん:03/11/19 11:19
∬(x-y)^2log(x+y)dxdyをお願いします。 (一応、被積分関数を口頭でいうと、カッコx-yカッコトジルノ2乗logカッコx+yカッコトジル) 積分領域は1≦x-y≦e,1≦x+y≦eです。
343132人目の素数さん:03/11/19 11:31
(314/315)^(1828/(250/25))*(1/2)
+
(314/315)^((1828*2-63)/(250/25))*(1/2)
+
(314/315)^((1828*3-63*2)/(250/25))*(1/2)^2
+
(314/315)^((1828*4-63*3)/(250/25))*(1/2)^3
+
(314/315)^((1828*5-63*4)/(250/25))*(1/2)^4
・・・・無限に続く・・・・・・・・=A
これのAの値を求めたいんですけど無理ですか?
344132人目の素数さん:03/11/19 11:51
>>342
p=x-y
q=x+y
で変数変換するとよいかも
345132人目の素数さん:03/11/19 11:53
>>343
わかりにくいけど
等比級数のような気が
346132人目の素数さん:03/11/19 12:07
>>342
u=x-y , v=x+y とおく。 x=(u+v)/2 , y=(-u+v)/2 だから
dx dy = |∂(x,y)/∂(u,v)| du dv = (1/2) du dv
∬(x-y)^2log(x+y)dxdy
=(1/2)∬u^2 logv du dv
=(1/2)∫[1,e] u^2 du * ∫[1,e] logv dv
=(1/2)[u^3/3][1,e] * [vlogv-v][1,e]
={(e^3-1)/6} * 1
=(e^3-1)/6
347132人目の素数さん:03/11/19 13:17
なんで−と−かけると+になるの。
そんなら+と+かけて−になったっていいじゃない!
>>329
二次関数は放物線しか表さんぞ。二次曲線だろ。
>>347
マイナスのほうが偉いから、プラス掛けるプラスはマイナスにはなれません。
でも、マイナス掛けるマイナスは欲張りなので、罰としてプラスに格下げされます。
350132人目の素数さん:03/11/19 13:24
基本的な問題で本当にすみませんがお願いいたします。

http://www.geisya.or.jp/~mwm48961/koukou/sct201.htm

ここ↑にのってる問題の『エ』なんですけれども、
解法を教えてください。

他の計算も良くわからないのですが、
コツみたいなのがあったら教えて下さい。
学生ではないので、薄い参考書片手にやっており、
解説が不十分でスムーズに理解出来なくて困っております。
>>350
普通に通分して整理して約分するだけ。
352350:03/11/19 14:33
分母を(1−sinA)cosA で通分すると
分子がcosA^2+(1−sinA)^2になって、
分子がcosA^2+1−2sinA+sinA^2
ここから先がどうなって2/cosA になるのかが分かりません。。
353132人目の素数さん:03/11/19 14:49
>>352
(cosA)^2+(sinA)^2=1

三角関数で一番忘れちゃいけない公式
354:03/11/19 15:01
「一円玉がX枚あるこれを出来るだけ5円玉と両替すると、硬貨の総数は60枚だけ減る、さらに出来るだけ10円玉と両替すると、硬貨の総数は10枚になる。一円玉は何枚あったでしょうか?」のXを求める式を教えてほしいんですが、ちなみに答えは77です。
355132人目の素数さん:03/11/19 15:07
>>354
1円5枚を5円に両替すると1枚になるので、硬貨の枚数は4枚減る。
60枚減ったのだから、15枚の5円玉に変ったってこと。

75円〜79円のどれか。
で、10円玉に両替すると、75〜79までどれでも
10円玉は7枚。5円玉は1枚。合わせて8枚。の総額75円
残りは1円玉で総数が10枚だから、1円玉2枚

よって77円
356:03/11/19 15:16
求める式っていうのは無いんですか?記号と数字を使ってとか。
>>356
公式至上主義。一番数学ができない人間の多くが示すタイプだな。
358132人目の素数さん:03/11/19 15:20
「それ以上でもそれ以下でもない」という言葉があります。
これを数学的に説明してください。
359:03/11/19 15:22
ごめんなさい。教えてください。気になるんです。
>>358
空集合です。
>>359
4[X/5]=60
X-4[X/5]-[[X/5]/2]=10
362132人目の素数さん:03/11/19 15:46
>>357
そんなことないだろ。ある問題を解いたらそれを一般化したいと思うのは普通じゃない?
>>362
話の一般化をしようという香具師と、楽に計算する方法を教えろという
香具師とでは異なる。

基本的には>>355を式になおせば済む話なのだから、>>354が前者ではなく
後者であるというのは明白。
xyz-空間の点と平面の距離について。
(1)完成平方のテクニックを使って求めよ。

(2)多変数関数の(偏)微分を使って求めよ。

という問題なのですが、まったく分かりません。
おねがいします。
365132人目の素数さん:03/11/19 16:00
343の問題はやっぱ無理ですか?
等比級数というのは何かわからないので今から調べてみます。
誰か343わかる方いましたらとき方教えてください。
代数の問題です
G=<g>を位数が12の巡回群としたとき、
GにはGと異なる最大の部分群はあるか、否か?
Gと加法群Z/(12)は同型であるか、否か?

を調べよということです。
代数が苦手でちょっと辛いです。誰か頼みます。
>>366
教科書に必ず載ってる事柄から分かるから嫁。
>>367
うちの学校教科書使ってないんですけど・・・板書のみで。しかも分かり辛い。
お勧めの参考書があれば教えてください。
大学生の癖に「教科書」を国定教科書見たいな指定書のことと
思ってるヴァカハケーン。
370332:03/11/19 16:34
関数f(z)は任意のz1、z2に対し、(z∈C)
f(z1+z2)=f(z1)f(z2)
を満たし、かつz=0でf(0)=f‘(0)=1が成り立つとする。
f(z)を見出せ。(f(z)=u(x、y)+iv(x、y)の満たす微分方程式を解け)

答えはすぐに分かるんですが、解き方が・・・
>>368
新数学講座「代数学」:永尾 汎 (朝倉書店)とか 数学選書「代数概論」:森田 康夫 (裳華房)
とか?
>>370
微分方程式作って解けって書いてあんジャン。
>>371
ありがとうです。
立ち読みして、いいほうで勉強してみます。
374132人目の素数さん:03/11/19 16:55
>>370
z1で微分
f′(z1+z2)=f′(z1)f(z2)
z1=0をいれてみる。
375132人目の素数さん:03/11/19 17:55
>>365
まず一般項を求めろ
250/25なんて計算しとけ
話はそれからだ。
376hansin:03/11/19 18:30
C[n.r-1]:C[n.r]:C[n.r+1]=1:3:5のとき、n、rをもとめよ。
よろしくお願いします。
>>376
nCr = n!/{(n-r!)r!} なのは知ってるよね?
378132人目の素数さん:03/11/19 18:43
n=7,r=2じゃないのかな
たぶんあってる。
おれもそうなったから。
380132人目の素数さん:03/11/19 18:49
>>376
C[n.r]= (n!)/{(r!)((n-r)!)}

C[n. r-1]= (n!)/{((r-1)!)((n-r+1)!)}=C[n.r] r/(n-r+1)
C[n. r-1]= (n!)/{((r+1)!)((n-r-1)!)}=C[n.r] (n-r)/(r+1)
なので
C[n.r-1]:C[n.r]:C[n.r+1] = r/(n-r+1) : 1 : (n-r)/(r+1) = 1:3:5

3r = n-r+1
3(n-r)=5(r+1)

r=2, n=7

381hansin:03/11/19 19:23
ありがとうございました。
もう1問お願いします。
(1) f(x)=cos^2 x のとき、f^(4)(0) を求めよ。
(2) f(x)=sin2x・cosxのとき、f^(n)(x)を求めよ。
以上よろしくお願いします


382132人目の素数さん:03/11/19 19:35
(1) f(x)=(1+cos2x)/2
f'(x)=-sin2x
f''(x)=-2cos2x
f'''(x)=4sin2x
f''''(x)=8cos2x
f''''(0)=8
383pc:03/11/19 19:54
積分∫e^(-x)cosxdxを求めよ。
この問題をお願いします。 
>>383
i*∫ e^(-x)*sin(x) dx を加えて積分してその実部をとる。
385:03/11/19 19:59
G(x)=∫[a→x] f(t) dt とおくとき、 dG/dx=f(x)である。
これを利用して次を求めよ。
(1) d(∫[a→x^2] f(t) dt)/dx
(2) d(∫[a→x^3] f(t) dt)/dx
この問題をお願いします。 
386132人目の素数さん:03/11/19 20:01
>>385
合成関数の微分

(1)d(∫[a→x^2] f(t) dt)/dx = d(G(x^2)/dx=2xf(x^2)
(2)も同様
>>385
「G(x)=∫[a→x] f(t) dt とおくとき、 dG/dx=f(x)である」の証明と同様のことを
やるか、合成関数の微分をつかう。
388132人目の素数さん:03/11/19 21:14
利用せよという指定があるので
合成関数だろう
389132人目の素数さん:03/11/20 00:26
z=x/√(x^2+ y^2)をxに関する偏導関数を求めたいと考えてます。

z = x * (x^2 + y^2 )^(-1/2)
(uv)' = u'v + uv'を用いて
z_x = (x^2 + y^2 )^(-1/2) + x * ( (-1/2)(x^2 + 1)^(-3/2) * 2x )
を変形しても
z_x= y^2 / (x^2 + y^2)^(3/2)・・・(答)
にはならないのですが、いかがです?
どうしても、(x^2 + y^2 )^(-1/2)の部分である1/(√(x^2+y^2))が消えないのです。
皆さんどうなりましたか?
もし答のようになるのなら、どこが間違いなのか教えてください。
>>389
なりますが…

(x^2+y^2)^(−1/2)+x*(−1/2)(x^2+1)^(−3/2)*2x
=(x^2+y^2)/(x^2+y^2)^(3/2)−x^2/(x^2+y^2)^(3/2)
=y^2/(x^2+y^2)^(3/2)…(答)
391132人目の素数さん:03/11/20 00:48
>>390
うわっ!なりましたぁ!!!
消えないのではなく、『消す』のがコツなんですねぇ。
凄いなぁ。>>390
ありがとうございました。

>>391
喪舞が計算を途中で放っぽりだしただけだ。
393132人目の素数さん:03/11/20 01:03
32445*3235
おね
394132人目の素数さん:03/11/20 01:12
>>392
喪舞とは?
>>394
「お前」の2ch語。いちいち無駄レスすんな。
396132人目の素数さん:03/11/20 01:33
おもしろい。@喪舞
397daiei:03/11/20 01:44
>381
(2) まづフーリエ展開する.
f(x)=(1/2){sin(x)+sin(3x)}
f^(2m)(x) = (1/2){(-1)^m}・[sin(x)+{3^(2m)}・sin(3x)]
f^(2m+1)(x) = (1/2){(-1)^m}[cos(x)+{3^(2m+1)}・cos(3x)]
あるいは複素数を使って
f(x) = (1/2)Im[exp(ix)+exp(i3x)]
f^(n)(x) = (1/2)Im[(i^n)・{exp(ix)+(3^n)exp(i3x)}]か?
398ヒッキー中2:03/11/20 02:53
数学UBの微分法・積分法に入るとき、数学TAまたは数学UBのここは絶対習得済みじゃないとダメ!ってとこはどこですか?
教えてください。例えばUBだと先にベクトルをやってなきゃいけないですね?
ちなみに自分は、数学IAは二次関数・場合の数・確率・三角比はやってます。図形の計量はまだ砂上の楼閣状態ってかんじです。
平面幾何(平面図形)はまだ手をつけてないです。
399132人目の素数さん:03/11/20 03:14
>>398
特にないと思うけど
>>398
・多項式が自在に操れること。
(展開、因数分解、除法、等式・不等式の証明など)

・2次関数の理論
(方程式と根、直線と放物線のグラフなど)

・分数のしつこい計算力

この3つがあれば大丈夫。
401132人目の素数さん:03/11/20 07:55
πの小数部には、いつか必ず任意の数nが現れると聞きました。そこで問題です。
 .141592[65]32(以下略 <- 7桁目にn=65が現れた(このとき、n=任意の数、m=発見した桁数とする)

Q1 では、任意の数nがπの小数点部n桁目より小さい場所から現れる確率を求めてください。
  このときπの小数部は、必要十分なだけ(n桁+数nの桁数)が用意されているとします。

Q2 πの小数部のm桁目(ただし、m<nの場合に限る)に任意の数nを発見できたとき、
  そのmという数を新たなnとして、再びπの小数部からの発見を試みるとします。
  これが成功する確率を求めてください。

お願いします。
402401:03/11/20 07:58
すみません。Q2の再検索はQ1同様m<nのときにだけ成功とみなします……。
>>401
任意の数 n が(中略)現れる確率
なら、n=2 のとき現れないので 0 ってことになっちゃいそう。
それとも、確率を n の式で表して n→∞ を取れってこと?

(2) は、発見できたときの条件付確率?
404132人目の素数さん:03/11/20 11:49
なんか問題が変だな
405132人目の素数さん:03/11/20 15:50
>>398
問題集で、極限の所を自力で解き続けるのが
まずは第一だと思うよ
それさえこえれば何の事は無い。
そういうところを手抜きしちゃうと
表面的な知識になって使い物にならんので
406keita:03/11/20 16:13
平面上の三角形ABCと点Pは3PA↑+2PB↑+PC↑=0↑を満たしている。
(1)AP↑をAB↑とAC↑で表せ。
(2)三角形PAB、三角形PBC、三角形PCAの面積の比を求めよ。
この問題わかる方教えていただけないでしょうか?
お願いします。
407132人目の素数さん:03/11/20 16:51
Rの部分集合Sを次のように定義する
S={5n/4n+1|n∈N}
(1)Sの上限と下限を求めなさい
(2)Sに最大値・最小値があれば求め、なければその理由を説明しなさい
お願いします
408132人目の素数さん:03/11/20 16:57
>>406

(1)3PA↑+2(PA↑+AB↑)+(PA↑+AC↑)=0↑
-6PA↑=2AB↑+AC↑
AP↑=(2AB↑+AC↑)/6

(2)AP↑=(2AB↑+AC↑)/6
=(1/2)*(2AB↑+AC↑)/3
BCを1:2に内分した点をDとおくと、線分ADの中点がPである。

後は自分で考えれ。
409moon:03/11/20 16:58
「一般項がan=np^nで与えられる数列an(n=1,2,3…)に於いて、任意の自然数nに対してa1≦anが成り立つ為に実数pが満たすべき必要十分条件を求めよ。但し、必要ならば|p|<1の時、n→∞ならばnp^n→0であることを用いても良い。」
マルチ
411132人目の素数さん:03/11/20 17:00
>>407
a[n]=5n/4n+1とすれば
a[n+1]-a[n]>0より単調増加列
よって下限、最小値はn=1のとき1
またa[n]は5/4に収束するので上限は5/4
だがこれを取るnは存在しないので最大値はない
412おやじ:03/11/20 17:11
X^2−4X+1=0
これって、どうやると
X=4±√4^2−4/2
こうなるんでしたっけ?
すっかり忘れてしまいまして…
>>412
両辺に3を加えて左辺を因数分解
414132人目の素数さん:03/11/20 17:16
>>412
x^2 -4x+1=0
(x-2)^2 = 3
x-2 =±√3
x=2±√3
415おやじ:03/11/20 17:26
>>413>>414
どうもありがとうございます。
記憶が甦ってまいりました。
>>414
詳しく書いていただき感謝です。
416132人目の素数さん:03/11/20 17:31
>>409
a1=p
an=np^n
より
任意のnについて
p≦np^n (1)
が成立するのに必要十分な条件を求める。

p≦-1の時

3p^3 ≦ 3p<p
より(1)は不成立

p≧1の時
np^n≧np≧p
よりnの値に関わらず(1)は成立

|p|<1の時、
np^n→0 (n→∞)より
p≦np^n→0
で(1)が成立するためにはp=0であることが必要十分

よって、p=0, p≧1であることが必要十分
417:03/11/20 17:51
命題論理で、
A |- ¬¬(A∨¬A) の証明ができません。おねがいします。

ちなみに
¬¬(A∨¬A) by ¬-I{
(x::¬(A∨¬A))[
⊥ by ¬-E{
x;
A∨¬A by …

から排中律の処理がわかりません。
>>417
マルチ
419132人目の素数さん:03/11/20 18:40
>>416
公に定義されてもいない指数法則を縦横無尽に使っていて元気でヨロシイ!(藁
420132人目の素数さん:03/11/20 18:43
>>419
どういうこと?
421132人目の素数さん:03/11/20 19:13
>>419
n=1,2,3…とあるから指数の範囲の話じゃないよね?
とするとpが実数ってことに対して言ってるのか?
しかし、|p|<1の時、n→∞ならばnp^n→0である云々とあるから
これも当てはまらないし
422132人目の素数さん:03/11/20 19:24
例えば、48と120の公約数は
1、2、3、4、6、8、12、24
であると。
この中で最大なものは24であるから、48と120の最大公約数は24であると。

今、二本の数直線があります。(もちろん1単位ずつの区切りがあります)←意味分かるよね。
そのうちの一本は48で点が打ってあり、もう片方は120に点が打ってあります。
48で点が打ってある直線をA直線、120で点が打ってある直線をB直線と呼ぶ事にする。
で、歩幅を設定してやるんだよね。
3単位ずつ歩く事にすれば(歩幅3で歩く事にすれば)、A直線で48という点を通るし、B直線で120という点を通る。
だから、3は公約数だと。

まあ、ここまでは前置きなんですけど。

<互助法手順その1>
120を48で割ってやる。すると、2が立ち24が余りとして出てくる。
きちんと書いてやると、120=48×2+24
(余りがあるから手順その2へ進む。)
これは、まず48単位ずつ歩く(歩幅48で歩く)事にしたんですよね。
すると、A直線の方はうまくいくと。うまく48という点を通ると。(至極当然な事ですが)
でも、B直線の方がうまくいかない。二歩だけ歩いて96という点で途方に暮れてしまう。
もう一歩あるけば、144という点に達してしまい120を通り過ぎてしまうから。

<互助法手順その2>
次に、48を24で割ってやる。
これは、あれですよね?歩幅を24にしてやろうってことですよね?
まあ、ここでうまくいくんですよね、この場合。
だから、24が最大公約数って事なんですけど、24と48の間の数x(24<x<48)の存在が気になるんです。
例えば30は?って。飛ばしちゃっていいの?って。
48が30とかその辺の数で割り切れない保障は?って事なんです。
そりゃーこの場合数が小さいから、すぐ「割り切れるわけないだろ」って思えるんですけどね。
423132人目の素数さん:03/11/20 19:31
>>416
|p|<1の時、
np^n→0 (n→∞)より
× p≦np^n→0

424132人目の素数さん:03/11/20 19:56
ここで聞いていいのでしょうか、大学の宿題わかりません。
教科名は計算理論。オートマトンとか扱ってます。

今回は
1、 正規表現とは何か、調べること
2、 3つの倍数となる2進数全体を正規表現で表せ
とあって、問1は本やらインターネットやらを調べてやったのですが、2がわかりません。
ヒントだけでもいいからアドバイスお願いします。ってか(;´д`)答え教えてください
425132人目の素数さん:03/11/20 19:57
難問。

m, n(≧5), a_1, a_2, ……, a_m-1, a_mはすべて自然数とする。

<a_m, a_m-1, ……, a_2, a_1>m = Σ[1≦k≦m]a_k*k! (1≦k≦m, 0≦a_k≦k, a_m≠0)とおく。

n!を <a_m, a_m-1, ……, a_2, a_1>m で表せ。

1hかけた自分を卓越するような解答を、願います。
426132人目の素数さん:03/11/20 20:09
>>422
互助法の手順は
120と48の最大公約数を求めたい

48と24の最大公約数を求めればよい

48は24の倍数である。

という構造になっています。
要は、48と24の最大公約数を求めているので
24より大きな数字にはなりません。

aとbの最大公約数を求めるために
互助法では
a = mb+r (0≦r<b)
という式を使っています。
aとbの最大公約数をpとして
a=px
b=py
であれば xとyは互いに素ですが
r=a-mb=p(x-my)
でrもpを約数に持ちます。
r=pzと置けば
a=mb+r=p(my+z)となります。
yとzは互いに素です。
互いに素でなければ 1より大きな自然数qを用いて
y=q f, z=q gと置くことができ
a=pq(mf+g)となり、aとbはpより大きなpqを公約数として持ってしまいます。
y=√(1 - sin^2(x-π/2)) (-π/2 ≦ x ≦ π/2)のグラフ

の場合、

1 - sin^2(x-π/2) を cos^2x + sin^2(π/2) に直して

y = √(cos^2x + sin^2(π/2))

になって、

x= 0のとき y = √2

x = -π/2 のとき y = 1

x = π/2 のとき y = 1

よって 山のような形のグラフになるってことでいいのでしょうか?
428132人目の素数さん:03/11/20 20:13
>>425
使っていいものがよく分からない
mやa_kは固定されたものなのか?
何を動かしていいんでしょう?
429132人目の素数さん:03/11/20 20:13
>>425
>・・、a_1, a_2, ……, a_m-1, a_mはすべて自然数とする。
と言っておきながら
>・・・、0≦a_k≦k, a_m≠0)とおく。
って、何だよ?
それに
> <a_m, a_m-1, ……, a_2, a_1>m
って表現マンドクセ (ry
430132人目の素数さん:03/11/20 20:14
>>416
指数法則調べてねぇ〜な! ブォゲ!
>>425
マルチ
432132人目の素数さん:03/11/20 20:19
>>427
>1 - sin^2(x-π/2) を cos^2x + sin^2(π/2) に直して

↑ここが変
433424:03/11/20 20:23
だ・・・だれか頼む・・・
>>432

1-sin^2x = cos^2x

ですよね?どこが違うのでしょうか?
435:03/11/20 20:37
次の指数方程式を解け。
6^x + 2^(2x+1)=3^2x
よろしくお願いします。
436132人目の素数さん:03/11/20 20:52
>>434
それで、そこからどうやったら

1 - sin^2(x-π/2) が cos^2x + sin^2(π/2) になるの?
っていうか、なんでsin(π/2)のような値のはっきりしているものを
いつまでも直さず、sin(π/2)のまま置いておくのかな?
そういうことしてるから計算ミスもなかなか見つからないんだよ。
>>436

1 - sin^2(x-π/2)を分配法則で計算すると

1 - sin^2x + sin^2(π/2)



cos^2x + 1

じゃないのですか?
438132人目の素数さん:03/11/20 21:02
>>435
6^x+2^(2x+1)=3^(2x) ⇔ 6^x+2*2^(2x)=3^(2x) ⇔ (3/2)^x+2={(3/2)^x}^2
⇔ {(3/2)^x}^2-(3/2)^x-2=0 ⇔ {(3/2)^x-2}{(3/2)^x+1}=0 ⇔ (3/2)^x=2
⇔ x=log2/(log3-log2)
439moon:03/11/20 21:10
>>416
pが負の場合が問題なんです。
>>437
ネタ?
441424:03/11/20 21:14
だ・・・だれか・・・頼む
もう時間がないんだよおおおおおお
>>440

ネタじゃないので教えていただけませんか?
443437:03/11/20 21:22

1 - sin^2(x - π/2) = cos^2(x - π/2)



y = cos(x - π/2)

ってことでしょうか?
444132人目の素数さん:03/11/20 21:23
>>442
sin(x-π/2)=-cosx だぜぃ!
y=√[1-{sin(x-π/2)}^2]=√{1-(cosx)^2}=√(sinx)^2=|sinx| (-π/2≦x≦π/2)のグラフ
445132人目の素数さん:03/11/20 21:29
>>437
分配法則なんて成り立たないよ
三角関数は加法定理でばらすんだよ
>>437
1 = cos^2(0) = cos^2(0+0) を君の言う「分配法則」で計算すると

= cos^2(0) + cos^2(0) = 2.
447401:03/11/20 21:31
>>403 後者です。Q1はnを含む式でお願いします。
Q2は、二回連続で一連の操作が成功(m<n)する確率でお願いします。
昨日から実際にやってみているのですが、Q2が成功したの見たことないです。確率がかなり知りたいです。
448437:03/11/20 21:33
分配法則でできないことはわかりました。
>>443も間違ってるのでしょうか?
449132人目の素数さん:03/11/20 21:33
ABCDEFGHの8文字を無作為に1列に並べる時AはBより左でBはCより左になる
確率はどうやればいいんですか?
450132人目の素数さん:03/11/20 21:36
>>448
平方根を取る時ってのは符号に気を付けないといけないよ
>>449
ABCDEFGHの8文字を無作為に1列に並べる時の全くみあわせの数は8!。
うちAはBより左でBはCより左になるくみあわせの数は8!/3!。
(□□□DEFGHをならべて(8!/3!通り)□の部分を左から順にABCをうめていく。)
よって確率は1/3!=1/6。
>>449

ABCDEFGHの8文字を一列に並べる順列の総数で、
そのうち「AはBより左でBはCより左になる」ような順列の数を
割ればよい。
453132人目の素数さん:03/11/20 21:38
>>447
言ってることがよく分からないけど
円周率は何か関係あるの?
円周率の小数点以下を最初から与えられたものとして
その数列に対して割合を計算したいのか、
乱数列に対してそういったものを求めたいのか?
454437:03/11/20 21:38
>>450
では
y = ±cos(x - π/2) ということでしょうか?

>>449
いまABCの文字は対等なのだから、
これら3文字がどういう順序で登場するか(3!=6通りある)は
すべて同様に確からしい。
よって求める確率は1/6 。
456moon:03/11/20 21:45
「一般項がan=np^nで与えられる数列an(n=1,2,3…)に於いて、任意の自然数nに対してa1≦anが成り立つ為に実数pが満たすべき必要十分条件を求めよ。但し、必要ならば|p|<1の時、n→∞ならばnp^n→0であることを用いても良い。」
誰でもいいから助けて下さい!pは正とは限らないんです!
457132人目の素数さん:03/11/20 21:53
>>456
>>416は p≦-1の時まではやってあるよ

>>416の|p|<1の時、ってのは、p≧0の時しかやってないから

残りは、-1<p<0の時
p≦np^nが成り立つためのpの条件
458437:03/11/20 21:54
>>454>>444のどちらが正解なのでしょうか?
なぜsin(x-π/2)=-cosx になるのかよくわからないのですが
459moon:03/11/20 21:56
私もそこまでは解いていたんですが、上手く-1<p<0の時を絞りきれないんです。
460132人目の素数さん:03/11/20 21:59
数列{a_n}は、任意の自然数Nに対して
a_1+a_2+…+a_N = N(a_1+a_N)/2 を満たしている。
このとき{a_n}は等差数列であることを証明せよ。

と言う問題なのですが、何を証明 すればいいんでしょおか?。
これは等差数列の和の公式から明らか、としてはだめなんですか?。
>>460
>等差数列の和の公式から明らか
それは、等差数列ならばあの式が成り立つ、ということであって
問われているのはその逆だろ。
462132人目の素数さん:03/11/20 22:09
>>461
えーと、よくわからにいのですが、
では何を証明すればいいのです?
寝る前にもう一度書いておきます。
>>454>>444のどちらが正解なのでしょうか?
よろしくお願いいたします。m(_ _)m
464132人目の素数さん:03/11/20 22:21
>>463
もう一度書くけど
平方根を取る時ってのは符号に気を付けないといけないよ
465132人目の素数さん:03/11/20 22:22
次の極方程式で表される曲線を直交座標(x,y)で表せ。
r^2(7cos^2θ+9)=144

よろしくお願いします。
>>462
問題にあるように、「{a_n}が等差数列である」こと。
>>465
r^2(7cos^2θ+9)=144
⇔7(r^2)cos^2(θ)+9(r^2)=144
⇔7(rcosθ)^2+9(r^2)=144
⇔7x^2+9(x^2+y^2)=144
⇔x^2/9+y^2/16=1
>>464

±をつけることじゃないんですか?よく分からないのですが。。
469132人目の素数さん:03/11/20 22:27
>>466
「「{a_n}が等差数列である」こと。」を証明するには
何を言えばいいんですか?
470132人目の素数さん:03/11/20 22:27
>>468
中学校の教科書で、√の定義を再確認すること。
471132人目の素数さん:03/11/20 22:30
http://coolnavi.com/up/file16001/
ここの問題の

点Dを通り、BFに平行な線を引き、ACとの交点をGとする。
BD:BC=2:3より、FG:GC=2:3
AE:ED=2:1より、AF:FG=2:1はわかったんですが、
それからAFが求められません。

ご教授お願いします。
>>467
ありがとうございました!
>>469
何を言うって、方法は色々あると思うが
漏れは最終的にa_n=a_1+nd(d:公差)の形になることを
導いて終わった。

それより、「公式から明らか」では駄目なことは
理解したのか?
(n-1)dの間違い。
475132人目の素数さん:03/11/20 23:04
>>459
-1<p<0の時
p≦np^n
すなわち
np^(n-1)≦1
となる条件を調べる。

n=1の時、いかなるpであれ成立
nが偶数の時、p^(n-1)<0となり常に成立
nが3以上の奇数の時
n=2m+1と置くと
(2m+1) p^(2m) ≦1
p^(2m) ≦1/(2m+1)
p^2 ≦ {1/(2m+1)^(1/2m)}^2
を調べることになるが

考えているm≧1において
1/(2m+1)^(1/2m)は増加関数であるため
m=1の所が重要で

p^2 ≦ 1/3

つまり
-1/√3 ≦p<0
曲線 x^4+y^4=1 上を点(x,y)が動く時の関数
 x^3+2y^3 の最大値最小値を求めよ
477132人目の素数さん:03/11/20 23:20
>>476
普通にy=〜に直して
入れて微分。

あと、x≧0、y≧0で最大値を求めれば、最小値は反対側にあるんで
478132人目の素数さん:03/11/20 23:25
>>471
うまく表示されませんが何か?
479132人目の素数さん:03/11/20 23:31
>>460
N≠1 のとき
    a_1+a_2+…+a_N = N(a_1+a_N)/2
−)a_1+a_2+…+a_(N+1) = (N+1){a_1+a_(N+1)}/2
        -a_(N+1)={a_1+Na_N-(N+1)a_(n+1)}/2
⇔ (N-1)a_(N+1)-Na_N+a_1=0 ⇔ (N-1){a_(N+1)-a_1}=N(a_N-a_1)
⇔ {a_(N+1)-a_1}/N=(a_N-a_1)/(N-1)
∴ (a_N-a_1)/(N-1)=(a_2-a_1)/1=a_2-a_1 ⇔ a_N=a_1+(N-1)(a_2-a_1)
これは N=1 のときも満たすので、任意の自然数nに対して a_n=a_1+(n-1)(a_2-a_1)
ここで、a_2-a_1 は定数だから数列{a_n}は等差数列である。
480132人目の素数さん:03/11/20 23:34
ttp://winny.info/fileboard/files/img20030928013030.gif
何人?
だれかズバッと明快な答えを!
481132人目の素数さん:03/11/20 23:35
>>343
の問題について>>375で一般項を出せって言われたんですが一般項ってなんですか?
(250/25)=10です。25の部分は変えても使いたいのでそうしてましたがごめんなさい。
482480:03/11/20 23:44
>>481
他板の俺様がexcelで計算したところ

0.532008144
に近似されることが分かった。
483132人目の素数さん:03/11/20 23:52
>>481
等比級数について調べたんじゃないの?
一般項も、一緒に載ってると思うのだけど

>>365
>等比級数というのは何かわからないので今から調べてみます。
484リアル中3:03/11/20 23:52
>>169
氏ね!!!!!!!!!!!!!!!!!!!!!!!
485132人目の素数さん:03/11/20 23:53
>>456=http://math.dot.thebbs.jp/p.cgi?1045491055.832
マルチポストは止めましょう。
486moon:03/11/21 00:08
>>486
ごめんなさい
487132人目の素数さん:03/11/21 00:09
602って簡単なんですか?
488132人目の素数さん:03/11/21 00:10
ちゃんと待ってたら
ここでも分かったのにねぇ
489132人目の素数さん:03/11/21 00:12
>>487
602=2*7*43ですが
この数字が簡単なものと思えるかどうかは
人それぞれじゃないでしょうか。
490132人目の素数さん:03/11/21 00:22
調べたのですが理解できず・・・・
491132人目の素数さん:03/11/21 00:22
高です。

AB>AC である△ABCに於いて,辺BC上に点Pを,点B,点C,と異なるようにとる.
このとき,AP<AB であることを証明せよ.

この証明は、
――――
仮定より,AB>AC .
ということは,∠C>∠B .
点PはCBの内分点であるから,CB>CP .
よって,AP<AB
―――――
これでも問題ないですか?
492132人目の素数さん:03/11/21 00:27
曲面Z=1-√(x^2+y^2)の平均曲率を計算せよ。ただし(x,y)≠(0,0)とする。
この問題教えてくれる人いますか?
493132人目の素数さん:03/11/21 00:30
>>491
よってのところで
どういう事実を使っているの?
なんか循環してない?
494132人目の素数さん:03/11/21 00:39
解法がわからないので、教えてください。

0°≦θ≦180°のθに対し、関係式、sinθ+cosθ=1/2 ( 2分の1)が成り立つ時、sinθの値を求めよ。
>>494
マルチはいけないと思います
496494:03/11/21 00:52
>>495
もう片方のスレで答えてもらえなかったので・・・・ダメでしょうか?
正規表現を勉強し始めたばかりなのですが
■3つの倍数となる2進数全体を正規表現であらわしなさい
が解けません。
教えてください。
498132人目の素数さん:03/11/21 00:58
>>494
sinθ+cosθ=1/2
cosθ= (1/2) - sinθ
(cosθ)^2 = {(1/2) -sinθ}^2
1-(sinθ)^2 = (1/4) - sinθ +(sinθ)^2
2(sinθ)^2 -sinθ-(3/4)=0

t=sinθとでも置いて、2次方程式を解け
499132人目の素数さん:03/11/21 01:01
>>497
3つの倍数ってのは何?
正規表現って何?

とりあえず正規表現で検索をかければ分かるとおり
いろいろな場所で使われる言葉を、何の説明も無しにもってくるのはよくない
>>497
3つの倍数?
3の倍数となる数の2進表示、と解釈すればいいのか?
501132人目の素数さん:03/11/21 01:07
3次の整式P(x)が条件

(A)P(x)の最高次の係数は1である。
(B)P(x)は(x+1)^2 で割り切れる。
(C)P(x)をx-1 で割った余りと、x^2+x-2 で割った余りは等しい。

をすべて満たすとき、P(x)を求めよ。

よろしくお願いします。
>>497
とりあえずアイデアだけ。
・100≡1 mod 11より、100を受け入れた状態=1だけ受け入れた状態、と考えられる。
・w∈{0,1}^*が3の倍数の2進表記ならw0*も3の倍数の2進表記。
・10101は3の倍数の2進表記

>>501
3次でAの条件があるわけだから
P(x)=x^3+ax^2+bx+cと置いて計算するくらいはやったんだろうな?
503497:03/11/21 01:19
3の倍数という解釈で大丈夫です
正規表現とは、計算理論という授業でオートマトンなどを中心に扱ってきたので、この場合はオートマトンなどに関わってくるものです。
それを前提に調べたら「記号列の集合を表すための記述方法」のことだと理解しました。
正則表現とも呼ぶらしいです。
記号を使うのでしょうが、ここから先がわからなくて・・・
504497:03/11/21 01:19
>>502
ありがとうございます。
しばらく考えてみます。
505132人目の素数さん:03/11/21 01:24
23 名前: 132人目の素数さん 投稿日: 03/11/21 01:09
数学板でのこの記号は何ですか?→ ^

×(かける)ですか?
新着レス 2003/11/21(金) 01:22
24 名前: 132人目の素数さん 投稿日: 03/11/21 01:10
              ヽ \
           / \ \  / ̄ ̄ ̄ ̄ ̄ ̄
       ∧_∧/     ̄  < 俺を笑い死にさせる気か!この!
      (;´Д`)     i i i    \______
      /    ヽ _   i i i--、
     ./| |   | |   ̄ ̄ ̄ |:::::|.
    / \ヽ/| |       ノ__ノ..
   /   \\| |
   / /⌒\ し(メ    .i i i . .
 / /    > ) \  ノノノ
/ /     / /    .\_  ザックザック
し'     (_つ   /:::::/::...   /ヽ
          ; "ノ・ ./∴: / )i iヽ-、_へ    ,ヘ
          '',, : :―― / / i i i iヽ . ̄ ゙― ノ /
    n_    _/;    i  .ノ / /ノ-' ̄ ゙ ― 、__ノ
  _ノ 二二二、_( _Д_ ;)-ヽ_ノ-'←>>23
506132人目の素数さん:03/11/21 01:30
>>475
1/(2m+1)^(1/2m)は増加関数であるため
とありますが、どうしてそういえるんですか?微分してもきれいな形にならないし。
507501:03/11/21 01:38
>>502

はい、それでBとCから式が一つづつ出て、cが出たんですけど、
その後行き詰まってしまいまして・・・
508497:03/11/21 01:40
ttp://maborosi.kobe-du.ac.jp/sugiura/soft/super/fsm.htm#sec2
(´д`)ここで正規表現については大体調べたのですが・・・
2進数の3の倍数を全て、一つの正規表現の式で表せるんですよね。
うーん。。。
509132人目の素数さん:03/11/21 01:49
1.x^2002 を x^4-1 で割った余りを求めよ。
2.nを自然数とする。 x^2n+1 を x^2-4 で割った余りを求めよ。

modを使うような気はするのですが、どう書いたらいいのやら・・・
お願いします。
510132人目の素数さん:03/11/21 01:53
>>506
1/(t+1)^(1/t)
のlogを取って

f(t)=-(log(t+1))/t
微分すると
f'(t)={(t+1)log(t+1) -t}/{t^2(t+1)}

log(3) > log e=1だから
t≧2の時
log(t+1)>1で
{(t+1)log(t+1) -t} >(t+1)-t=1
なので
f'(t)>0 ゆえに単調増加
511132人目の素数さん:03/11/21 01:53
>>507
できたところまで書きましょう
>>507
(B)の条件についてもう一度考え直してみ。

>>509
1.x^2002=(x^4-1)x^1998+x^1998=…
2.帰納法を使う
513471:03/11/21 02:00
すいません。こっちでした。
http://coolnavi.com/up/file16001.zip

よろしくお願いしたします。
514132人目の素数さん:03/11/21 02:07
>>513
メネラウスの定理で検索しれ
515497:03/11/21 02:11
つまり
31 ≡1 mod 3より、31を受け入れた状態=1だけ受け入れた状態
となるから、0〜31くらいだけを考えればいいのかな。
516471:03/11/21 02:13
>514
検索してみましたが、さっぱりです。

BD:BC=2:3より、FG:GC=2:3
AE:ED=2:1より、AF:FG=2:1
からどうやって導くのですか?
517497:03/11/21 02:17
10進数なら3〜27のあとに0*(0が0回以上続く)って表すのですよね。
2進数の場合の3の倍数の見分け方がわからない・・・
518132人目の素数さん:03/11/21 02:18
>>501
A,Bより
P(x)=(x+1)^2 (x+a)とおける。

P(1)=4(x+a)

Cより、
P(x)=(x+2)(x-1)(x+b) +4(x+a)とおける。

Bに戻って
この右辺が(x+1)^2で割り切れる。
519132人目の素数さん:03/11/21 02:19
>>516
少しは考えろよ…
http://club.pep.ne.jp/~asuzui/page11.html
の図にあるとおりに重ねてみれば
AF:FCが出るからさ…
>>517
2 mod 3.
521497:03/11/21 02:23
>>520

この場合、2mod3が何の役に立つのでしょうか。
(;´д`)馬鹿でごめんなさい。
>>515
>>502のアイデアの出し方が良くなかったが
それ以前に何か誤解してないか?
>>521
10進数の場合の3の倍数の判定法は知ってる?
スレ違いだとおもいますが、"平行"の記号はどうやってだすのですか?
スラッシュ2つのやつです。
525497:03/11/21 02:31
>>523
mod3=0のときですよね。

>>522
2進数自体めったに使わないので、それを前提とした問題のコツとかがわからなくて・・
>>524
|| : 縦棒二本が世界標準です。
// : 日本語方言ならスラッシュふたつでOKです。
527501:03/11/21 02:33
>>518

P(1)=のところで、4(x+a)のxに1は代入してはいけないのでしょうか?
528523:03/11/21 02:34
>>525
・・・(゚д゚;) そうきたか・・・ 10 mod 3 を考えさせたかったのだけれど;
>>526
ありがとうございました。
530497:03/11/21 02:41
 ̄ ̄ ̄\:::::::::::::::::::::::::::::::::::::::::::::::::::::::::::::::\       /
       l:::::::::::::::::::::::::;;:::ヘ::rv'レ'、::::::::::::::::::ゝ    /    挨
  答   |::::::;::vrヘl/''"`"_,, ノ 、;;、 ''ヽ;:::::::::::!    |     もう
  え   \'' "   _,,..-=、''_  >',=-、 ' i:::::::;!    ヽ    時間  
  を    |   / ,rニ・L  | { ':ニ・i,.  |:::::l    /   が
  、   /    'i ,''≡'   | i`== i i;/l    >    な
  た  /,i     | !   r┤ i、  J ,:、!,!    |.      い 
  、  ヽ,! ‐=、;、 | l  , , ,`',,`''´,',  〃 |i    ヽ    ん
  頼  r‐-`;-  'ヾ;;''、 ,' , ',,;===; ' ,i/ l  ‐=、--'    だ
  む  |ヽ__,i  ,   i! , /r──-l! i! /;、_   ``i    
___/'´ i ', ' , ' ,  | ''" ̄ ̄ ̄  !./ i;;;;;``''-、 ``i   
;;;;;;;;;;;;;;/  i  `''-、, ', , ; , ' , ', ', ' ,', ' ,/'  |;;;;;;;;;;;;;;;;;;`''-i
;;;;;;;;;;;;/   i  ,' , .`''ー- '    ,. ‐'"/  |;;;;;;;;;;;;;;;;;;;;;;;;;゙ー、
>>517
交互に足したり引いたりして、できた数が
3の倍数なら、もとも3の倍数だよ。

たとえば10010111なら、1-0+0-1+0-1+1-1=-1
つまりこの数は3の倍数ではない。
(奇数桁目を+、偶数桁目を-と計算すれば、余りがわかる。
つまり-1+0-0+1-0+1-1+1=1 なので、3で割った余りは1。)

まあどちらにしろ、これを受理するオートマトンをつくり、
それで受理される正規表現を考えることになる。
3で割った余りは012のどれかなので、状態数は3でいい。
532132人目の奇数さん:03/11/21 02:48
問)ax^2 + bx + c = 0 (a,b,cは定数) を満たすx(∈C)の解を求めよ.

x の解 などというものは無い。
×問)ax^2 + bx + c = 0 (a,b,cは定数) を満たすx(∈C)の解を求めよ.
○問)ax^2 + bx + c = 0 (a,b,cは定数) を満たすx(∈C)を求めよ.
○問)ax^2 + bx + c = 0 (a,b,cは定数) の解x(∈C)を求めよ.
535132人目の素数さん:03/11/21 02:58
1/xのフーリエ逆変換はどういう形になりますか?
自分で計算してみたところ 間違っているような気がするので、
教えてください。
結果がわかったら計算しなおしてみるので、結果だけで構いません。
>>532
先生、a,b,c は定数だったら、H(ハミルトンの数体) の元や F_q (q 元体) の元
とかでもよかですか?
>>535
それは、まず、計算してないんだろうなぁと疑うのが、此処の第一の鉄則と
なっております。
539ひまじん:03/11/21 04:00
「3.14159265358979323846」・・・
カッコ以降があいまいなため、以降20ケタ程教えて頂きたいのですが・・・
540132人目の素数さん:03/11/21 04:06
Machinの公式でpiを計算するプログラム組んだ方が速いと思うよ。
26433832795028841971693
>>497

開始
終了
┏━┓──1─→┏━┓──0─→┏━┓
┃.0 ┃      ..┃.1 ┃      ..┃.2 ┃
┗━┛←─1──┗━┛←─0──┗━┛
↑ .│                   ↑ .│
└0┘                   └1┘
543132人目の素数さん:03/11/21 05:04
等差数列の和の公式、
Sn=(1/2)n(a[1]+a[n])
が導かれる過程を教えてください。
一般項を求める公式はすぐに理解できたんですけどこれがどうしても直感で理解できません。
ちなみに大学受験板には説明できるやつはいませんでした(全部参考書(しかも殆どGakkenのw)の丸写しでした)
544132人目の素数さん:03/11/21 05:23
>>541
ありがとうごさいます
>>543
こう見たらどうか?

Sn = n * (a[1]+a[n])/2

たとえば 3 5 7 9 11 13 の総和だったら、両端から
1つづつ取って (3,13) (5,11) (7,9) と考えると、両端の
平均である8が6個並んでるのと同じ事。
546132人目の素数さん:03/11/21 05:44
>>545
すっきり理解できました。ありがとうございました。
547132人目の素数さん:03/11/21 05:47
あと545さん。
こういうのって公式を見た瞬間に気づかなきゃいけないことですか?
それとも問題を解いているうちに自然と理解するものですか?
たとえばこの公式について、545さんのような説明をしている参考書も問題集もありませんでした。
ただ公式だけが書かれてありました。
これは瞬間に気づけるものなのかと思って。
548ヒッキー中2:03/11/21 05:55
>>400
感謝。
因数分解は,x^3+y^3=(x+y)^3-3xy(x+y) なども覚えとくべきですかね?
こういうの覚えるのって記憶の無駄遣いのような気がしてならないのです
なんか英単語の馬鹿暗記と同じ種類のような感じがしてしまって
549ヒッキー中2:03/11/21 05:57
538は裸の外人の女が風呂に入ってあがる動画
>>548
マジレスしとくとそんなもん覚えようとして覚えるものじゃない。
あほか
551132人目の素数さん:03/11/21 06:03
a,zを複素数とするとき、次の命題(※)が成り立つようなa
の存在範囲を複素数平面上に図示せよ。
(※)|z|^2+2a(z+1)=0を満たすzは存在しない。

はぁ・・・さぱーりわかりません
>>547
>問題を解いているうちに自然と理解する

これ。
見た瞬間に理解するヤシは、天才か、または過去に
似たような事例を取り扱ったことがあるだけだよ。
(1-x)^2=p^2+q^2   ただしpとqは定数である

xの値を求めたいのですが因数分解がうまくできません

おねがいします
>>553
x = 1±√(p^2+q^2)
555ヒッキー中2:03/11/21 06:55
p^2+q^2=t とおく。
1-2x+x^2=t
 x^2-2x+1=t
  x^2-2x+(1-t)=0
   x=[2±√{4-4*1*(1-t)}]/2
    x={2±√(4-4+4t)}/2
     x={2±√(4t)}/2
      x={2±2√t}/2
       x=1±√(p^2+q^2)
できた!!!!!!!!!!!1
556ヒッキー中2:03/11/21 06:56
>550
じゃあその都度求めるんですか?
x^3+y^3 これをずーっと分解していってから。
>>556
好きなようにしろ。
558132人目の素数さん:03/11/21 10:25
>>556
x^3+y^3 = (x+y)(x^2-xy+y^2)
なら覚えている人はいるかも知れないが
x=-yを入れたら0になることは一目でわかるため
因数定理からでる
559132人目の素数さん:03/11/21 10:32
>>551
自分自身も自信がない解答なので品評会会長に評価していただきたいと思います。

与式を
α(z+1)=-(|z|^2)/2……実数……@
と変形して、z+1=r(cosθ+isinθ)……A
とする。右辺も、負の実数より、(cosπ+isinπ)を使って表す。
Aより、|z|^2=(rcosθ-1)^2+(rsinθ)^2……B
これらから、@をaについて解くと、
α=-{(r^2-2rcosθ+1)/2r}cosθ+i{(r^2-2rcosθ+1)/2r}sinθ
となり、デカルト平面で考えると、r=0とr≠0で場合分けして
r≠0の時……(省略)……
y=-xtanθ ……C
r=0の時、a=0……D
よって、題意を満たすのはCDで表せない領域であるから、0を除く虚軸上。
560132人目の素数さん:03/11/21 10:33
>>547
等差数列の和の公式で天才といえば有名な話
http://homepage2.nifty.com/hi-rose/ijinden/4gatu/0430.htm
561132人目の素数さん:03/11/21 10:48
>>555
(1-x)^2をばらすな馬鹿
562132人目の素数さん:03/11/21 10:53
>>535
自分で計算してみてどうなったのか書くこと
563132人目の素数さん:03/11/21 11:37
>>559
xの値域について
何か一言欲しい所だね
564132人目の素数さん:03/11/21 12:28
>>556
>じゃあその都度求めるんですか?
x^3+y^3でyを定数と思って、x=-yを代入すれば0になる。
だからx^3+y^3は因数(x+y)を持つ。
わざわざ覚えなくてもこれくらいすぐ思いつくだろ。
すぐ思いつかないようなら、センスがないと思ってあきらめろ
565132人目の素数さん:03/11/21 12:30
かぶった
>>564
きますた本日のガロア気取りさん!
567566:03/11/21 12:38
ここはラウンジじゃなかった。スマンw
568132人目の素数さん:03/11/21 12:59
可分なヒルベルト空間の、可算個の一次独立なベクトル
{e_i}i\subset N
を取ってきた時、
<e_1,u>=1
<e_i,u>=0 for i \neq 1
となるベクトルuは存在しますか?

存在するならそれをどうやって示せばいいでしょうか?
569132人目の素数さん:03/11/21 13:03
  ,,v‐v-/l_  (⌒) 
  _「/ ̄  く   /
  lYノノ/_ノl_ヽ))
  <イ( l l )l>   / ̄ ̄ ̄ ̄ ̄ ̄ ̄ ̄ ̄ ̄ ̄ ̄ ̄ ̄ ̄ ̄ ̄ ̄ ̄ ̄ ̄ ̄ ̄ ̄ ̄ ̄ ̄ ̄ ̄ ̄
 <|)'|l、"(フノ|l  < なんでもかんでもセンスの所以にしちゃいけないと思います!!!
   ,(ヨリ<>o<>リ'] |  中学生の頃のわたくしが一瞬で気づけたかというと・・・・あやしい!!!
  |ト‐!]-ム- i']l  |  わたくしもセンスがないのでしょうか...
  ヽ_ノv__l/ / |  やっぱりセンスですね!!!
.  ノ[//‐─‐/_/、  |  それといいですか566さん!!!それはラウンジではなくヌ速の悪ノリ
 ( /E|,   (フlヨ \ |  というのですよ!!!!BAD SPEED PLAY というのです!!!
 ,-| El___lヨ / |  ああ恐ろしい・・・・!!!!
└-\`^^^^^^´/ | 
            \
>>564
F=(cosx-cosy)
yを定数と思って、x=-yを代入すればF=0になる。
だから(cosx-cosy)は因数(x+y)を持つ。w
>>569
そんな弱気なまほろさんはいない。出直し。
572132人目の素数さん:03/11/21 13:48
>>570
因数分解を何だと思っているのですか?
因数定理を語るなら“整式”を省略するな、と?
>>570
cos(x)-cos(y)=(x+y){(y-x)/2!+(x-y)(x^2+y^2)/4!+....}
てな感じのことは成り立つんだけどな
575132人目の素数さん:03/11/21 14:26
>>570って、Weierstrassの予備定理に出てくる分解のよーな
576132人目の素数さん:03/11/21 14:28
ウホッ
577132人目の素数さん:03/11/21 14:29
>>498
ありがとうございました!!
実はおれもリアル厨房なんですけど、数学に目覚めました。
実はすごいのにジョーク飛ばしたりして爪を隠す、秋山ジンさんみたいな天才数学者になりたいと思ってます!
ありがとうございました!!
578132人目の素数さん:03/11/21 14:36
>>577
メジャーを志す野球少年がパンチ佐藤を目標にするような話だな
579132人目の素数さん:03/11/21 15:22
x<<1のとき coth(x)=1/x + x/3 + x^3/45 + ・・・ と近似できます。

x<<1のとき coth(ax) はどういうふうに近似できるのですか?
580132人目の素数さん:03/11/21 15:31
>>579
coth(ax)=1/(ax) + (ax)/3 + (ax)^3/45 + ・・・ 
581数学板初心者:03/11/21 15:39
質問はここに書いてよろしいのでしょうか
平行記号(//)はどのように出せばよいのでしょうか?
582132人目の素数さん:03/11/21 15:48
>>581
普通に//
>>581
此処はただのネタスレだが。
平行記号はそれでいい。ただしその記号は日本の初等数学教育における方言
で、世界標準は || と縦棒二つを書くそうだ。
584132人目の素数さん:03/11/21 15:51
>>581
質問はここに書いていいよ。
質問スレであることは
ログ見て貰えばわかるから。
585数学板初心者:03/11/21 15:52
どうしてもちゃんと出したい人はどうすればよいのですか
586132人目の素数さん:03/11/21 15:53
>>585
ちゃんと出したいってどういう意味?
// のどこに不満があるの?
587132人目の素数さん:03/11/21 15:56
>>585
標準の文字として存在しないから無理
個人的に出したいだけなら、自分で描いて外字登録とか
だれか統計学なんでもスレッドで質問に答えてくれませんか…?
人があんまりいないんで…
http://science.2ch.net/test/read.cgi/math/1068288283/
589132人目の素数さん:03/11/21 16:17
>>588
質問したいならageなきゃだめだよ。
590132人目の素数さん:03/11/21 16:39
>>577
せめてピーターフランクルを目指せ
591132人目の素数さん:03/11/21 16:43
>>577
秋山じんなんて終わってるぞ。絶対天才じゃない。
電話でうざい勧誘とか身に覚えのない督促とか来たら、
初め必死に話聞く格好とって相手に『カモ』だと思わせる。
そしたらそのまま電話機放置してください。
相手は必死に話して1分くらい過ぎてから
「もしもし、もしもし」の連発。
そしてむかついて相手側から電話を切る。
この方法が一番有効です。
まだこの方法はあまり広まっていませんがみんながやれば
この手の裏の仕事は赤字続出!!
皆さんもこの手の電話は相手が電話切るまで放置してあげてください。
後この方法みんなに広めてね。
593132人目の素数さん:03/11/21 17:39
複素数平面上に0と異なる3点z_1、z_2、z_3があり、次の3つの条件を満たしている。
(1) arg(z_1)=arg(z_2)+120°
(2) 点z_3は2点z_1、z_2を通る直線に関して点0と反対側にある。
(3) △z_1z_2z_3は正三角形である。
問い
α=cos60°+isin60°とするとき、αz_1=pz_2、αz_2=qz_1+sz_2となる。
ただし、p=-|z_1|/|z_2|、q=|z_2|/|z_1|、t=1 (見難いですが要は絶対値です)
このとき、z_3=az_1+bz_2となる実数a、bをそれぞれ|z_1|、|z_2|を用いて表せ。

よろしくお願いします。
594132人目の素数さん:03/11/21 18:06
>>593
tって何?
595132人目の素数さん:03/11/21 18:12
>>594
すみません、間違えました。問題再掲

複素数平面上に0と異なる3点z_1、z_2、z_3があり、次の3つの条件を満たしている。
(1) arg(z_1)=arg(z_2)+120°
(2) 点z_3は2点z_1、z_2を通る直線に関して点0と反対側にある。
(3) △z_1z_2z_3は正三角形である。
問い
α=cos60°+isin60°とするとき、αz_1=pz_2、αz_2=qz_1+z_2となる。
ただし、p=-|z_1|/|z_2|、q=|z_2|/|z_1|(見難いですが要は絶対値です)
このとき、z_3=az_1+bz_2となる実数a、bをそれぞれ|z_1|、|z_2|を用いて表せ。
596統計学の問題。:03/11/21 18:29

統計学の問題。
300人の生徒の吸うg核の成績が、平均65点、
標準偏差12点で正規分布に近い分布をしています。
50点から70点までの生徒は何人くらいいると考えられますか?
597132人目の素数さん:03/11/21 18:45
>>596
       _,. -‐‐‐--‐‐‐--- 、 _
         /             \
      /                \
     / / / / l | l i l |   l       ヽ
     / / / |l | | l l l ||   l | l  ヾ、  l
    ,' l |/| || | |_l,,l__l !|  i l| l l   l|  l教科書読みましょう
     l ! l_,_l |l 'i´|l | |l |`l |l l| l | l   ! || l|
     |l'"|,,_l`|l |  ,;テ=ト、l || |l | | |  l | | | |
     |l_」,;=、 |ト、! '" i'_ノ;;ゞiト| |l || |l |, | | | | |,
    ,|l |,j {;i!    ゞ心シリ'! l| || |l |, |i |l l |i
   /|l |l トツ,    `=-' ` ト、 |l || || | |l | | | !
   //l l| ハ ´      ' ' ' |l ヾ、|| || | |ト| ト| |l
  〃ノ /l  ヽ. 丶        || | ト、 |ト、ヽ.|ヾl |l ヾ、
  /ノリ//l //iヽ      _,. |l |l | ヽ|l ヽ、ト、ヾ、\ー-
 ノ//〃/ 〃 |l /ヽ_,. -‐、´  || |l ト、/ヾ、 \`ー-  `゙'ー-、_
 ノ // /  |l/  |l//  \  |ト、|ヽヾ  /`ー-         `'ヽ、
  ノ //  リ   lリ    〉/ |l \/ヽ i   /         / ヽ
   ノ    /   /    |/    /   l   l             ト、__________
      ノ   ノ    /レヽ  /    l  l            ___,,,    丶ヽ
598132人目の素数さん:03/11/21 18:48
>>595
α(z_3-z_1) = (z_2-z_1)
z_3 -z_1 = α^(-1) (z_2-z_1)

α^(-1)z_2 とα^(-1)z_1を計算する。
αz_1=pz_2より

α^(-1) z_2=(1/p) z_1

αz_2=qz_1+z_2より

q α^(-1)z_1 = z_2 - α^(-1) z_2 = z_2 - (1/p) z_1

あとは、z_3=〜に入れてまとめれ
サイコロを何回か振って、出た目の数の和と積が共に偶数になったら
サイコロを振るのをやめることにする
ただし、1回目については偶数の目が出ればさいころを振るのをやめ、
奇数の目が出ればサイコロを振り続ける
また、10回振っても終わらないときは10回でサイコロをふるのをやめるものとする
このとき、4回以上サイコロを振り続ける確率
600132人目の素数さん:03/11/21 19:00
>>599
10回云々がなんのためにあるのかしらんけど
3回以内で終わる確率を考える
1回目 偶数なら終わり (1/2)

1回目 奇数なら継続
 2回目奇数でも偶数でも終わらない。

 2回目が偶数の時
  3回目奇数なら終わり (1/2)(1/2)(1/2) = 1/8

 2回目が奇数の時
  3回目が偶数なら終わり (1/2)(1/2)(1/2) = 1/8

よって3回以内に終わる確率は

(1/2)+(1/8)+(1/8)=3/4

4回以上降り続ける確率は1/4
601:03/11/21 20:30
G={1,2,3,4,5,6}において、x,y∈Gのとき、x*yはxyを7で割ったときの
余りであると定める。
(1) (3*5)*6,3*(5*6)を求めよ。
(2) すべてのa∈Gに対して、a*x=1となるx∈Gを求めよ。
(3) 1次方程式a*x=b(∀a,∀b∈G)がG内で唯一つの解を持つことを示せ。
以上よろしくお願いします。
(1) ぐらい小学生でもできるだろ
>>598
ありがとうございました!おかげで解けました!!
>α(z_3-z_1) = (z_2-z_1)
は(z_3-z_1) = α(z_2-z_1)の間違いだと思うんですが、方針を汲み取って
自分でやってみたら出来ました!ありがとうございました。
604132人目の素数さん:03/11/21 20:50
>>601
(1)
(3*5)*6=1*6=6
3*(5*6)=3*2=6
(2)
(a, x)=(1,1), (2, 4), (3, 5), (4, 2), (5, 3), (6, 6)

(3)
(x*y)*z = x*(y*z)
x*y = y*x
が成り立つ。(←理由を考えてみてくれ

(2)の通り、aに対して、a*y=1となるyが唯一定まる。
a*x=b
y*(a*x)=y*b
(y*a)*x=y*b
(a*y)*x=y*b
1*x=y*b
x=y*b
605スクリプトキディ:03/11/21 21:15
>>503 正規表現ってのは、求める文字列の全可能性を簡素に網羅する手段だ。
. 任意の一文字(今回の課題では不要だと思う)
X* 文字Xが0個以上 という意味
Y+ 文字Yが1個以上 という意味
Z? 文字Zが0個または1個という意味
[AB]+A*B 1個以上のAまたはB(AABBABAAB等)、0個以上のA、B という意味
(ABC*|DEF+)? ((文字A文字B文字Cが0個以上)または(文字D文字E文字Fが1個以上))が0個または1個という意味

参考になっただろ?
606132人目の素数さん:03/11/21 21:55
はい参考になりました。
607132人目の素数さん:03/11/21 23:03
>>559
>>563
xが実数全体を動くかどうかをどう評価したらよいでしょうか?
608132人目の素数さん:03/11/21 23:15
a_n = 1/√5 * (1+√5)/2 (n = 1,2,…) とおく。

(1)
a_n に最も近い整数 b_n を n の式で表せ。

(2)
a_n 以上の最小整数 c_n, a_n 以下の最大整数 d_n を n の式で表せ。

(1)は、b_n = 1/√5 * [{(1+√5)/2}^n - {(1-√5)/2}^n] だと。たぶん。
(2)は床&天井関数がNGなのでお手上げ。

何方か 完答して頂けませんか?((1)も念のため……。)
>>608
a_n って定数列に見えるんだが・・・?
610132人目の素数さん:03/11/21 23:21
>>607
気を付けなければならないのは
xがrとθの関数であるってこと。
つまり、xを固定したとき、tanθがいくらでも動いて
yの値がどうとでもなるように見えるけど、
tanθが動くとxも動く(w
そこらへん、変数を分離するなり適当な変数変換をするなりしないと
面倒なんじゃないかなぁ
611607:03/11/21 23:45
>>610
私もそう思う。となるとやっぱり他の解法がいいのかしら。
612132人目の素数さん:03/11/21 23:54
考えてもわからなかったので、下の問題証明してもらえませんでしょうか?
sin40°/sin80°*sin50°/sin30°=1
電卓でやったら一応成り立ちました。よろしくおねがいします。
>>612
コイツは嘘をついている味だ・・・
>>612
積→和
>>612
sin40°/sin80°*sin50°/sin30°
=sin40°/sin80°*sin50°×2
=sin40°/(2sin40°cos40°)*sin50°×2
=sin50°/cos40°
=sin50°/sin50°
=1
616612:03/11/22 00:30
>>615 さん
ありがとうございました。
>>615 さんの式を使うとhttp://www.geocities.co.jp/WallStreet-Stock/5773/mondai.html
これの問題を正弦定理とこの式だけでとけるようになると思います。
617132人目の素数さん:03/11/22 01:19
α,zを複素数とする時、次の命題(※)が成り立つようなαの存在範囲を複素数平面上に図示せよ。
(※)(|z|^2)+2α(z+1)=0を満たすzは存在しない。
>>617
マルチ?コピペ?
619551:03/11/22 02:28
>>618
オレじゃないです。

α=-{(r^2-2rcosθ+1)/2r}cosθ+i{(r^2-2rcosθ+1)/2r}sinθ
=-{(|z|^2/|z+1|)(cosθ+isinθ)
サパーリなことを言ってるかもしれませんが、|z|^2/|z+1|の範囲だけ
調べればいいのではないのですか?
>>608
>a_n = 1/√5 * (1+√5)/2 (n = 1,2,…) とおく。

その予測されたb_n(フィボナッチ列)から察するに
本当はa_n=(1/√5)*{(1+√5)/2}^nなんだろう。

(1)
B_n=(1/√5)*[{(1+√5)/2}^n-{(1-√5)/2}^n]として
B_nが整数列であることと、任意のnで|(a_n)-(B_n)|<1/2を示せば
b_n=B_nとしてよいことになる。

(2)
{(a_n)-(b_n)}の符号からa_nとb_nの大小関係を判別する。
nが奇数のとき、(b_n)-1<a_n<b_nを示して、c_n=b_n、d_n=(b_n)-1
nが偶数のとき、b_n<a_n<(b_n)+1を示して、c_n=(b_n)+1、d_n=b_n
(2)を一つの式で表したかったら
例えばe_n={1+(-1)^n}/2を使うと
c_n=b_n + e_n
d_n=b_n - e_(n+1)
622hansin:03/11/22 07:14
x^2 + y^2 + z^2=1 , a^2 + b^2 + c^2=1
⇒-1≦ax+by+cz≦1 を証明せよ。
(x,y,z,a,b,cは実数とする。)
よろしくお願いします。
623132人目の素数さん:03/11/22 07:42
>>622
x、y、z、a、b、cは実数、x^2+y^2+z^2=1 , a^2+b^2+c^2=1 のとき
(ax+by+cz)^2=(x^2+y^2+z^2)(a^2+b^2+c^2)-{(bx-ay)^2+(cy-bz)^2+(az-cx)^2}≦(x^2+y^2+z^2)(a^2+b^2+c^2)=1
∴ -1≦ax+by+cz≦1 (等号は bx-ay=cy-bz=az-cx=0 のとき成立)

まぁ ケーリー・ハミルトンの不等式だけどね。
こししゆわるつじゃなかった?
625623:03/11/22 08:33
>>624
仰る通りに御座いまする。
寝ぼけてた ゴメン
626132人目の素数さん:03/11/22 09:07
>>622
3次元空間でのベクトルの内積を考えたら瞬時だろ?
627132人目の素数さん:03/11/22 09:33
>>619
じゃ、自分で調べれば?
628pc:03/11/22 09:41
nは正整数とする。Zの元の間に関係〜を
x〜y⇔x−yがnで割り切れる。
と定義する。関係〜が同値関係であることを証明せよ。
この問題をお願いします。
同値関係の定義を順に確認していくだけだが、何が分からんの?
反射律、対称律くらいはできるだろ。
推移律もx-z=(x-y)+(y-z)などとすればいいし。
630132人目の素数さん:03/11/22 09:48
原点、O、A、Bの三転は複素数平面でいっぺんが正三角形をなす。また、P=α^2−2α、q=β^2−2βとする。
|p−q|の最大値、最小値を求めよ。

631132人目の素数さん:03/11/22 09:56
複素数平面か…嫌な響きだ
>>626
こししゆわるつと内積は同じこと
634別解厨:03/11/22 10:29
一方が現れると、他方がもれなくついてくる
635132人目の素数さん:03/11/22 11:05
l(エル)を複素数平面上の直線z=t(1+i) (tは実数)、α、βを複素数とする。
ただし、点αはl(エル)上にないとする。
(1)α=iβまたはα=β~(ベータバー)ならば、l(エル)上の全ての点zに対して|z~-β|/|z-α|=1
 であることを示せ。
(2)l(エル)上の全ての点zに対して|z~-β|/|z-α|=1ならば、α=iβまたはα=β~であることを示せ。
(3)l(エル)上の異なる2定点z_1、z_2があって
 (z_1~-β)/(z_1-α)=(z_2~-β)/(z_2-α)=γ(ガンマ)
が成り立つとする。このとき、l上の全ての点zに対し
 (z~-β)/(z-α)=γ
となることを示せ。また、γの値を求めよ。

昨日スルーされたんですが今日もよろしくお願いします。
637132人目の素数さん:03/11/22 11:12
l(エル) l(エル)
l(エル)はリップのl(エル)
638132人目の素数さん:03/11/22 11:13
>>636
板が違うからいいじゃないですか。実はそこでも答え出てないんです。
639132人目の素数さん:03/11/22 11:14
・゚・(ノД`)・゚・ウワーン
まじめに考えてくれないよー
板が違えばマルチしてもいいのか。初めて聞いた。
641132人目の素数さん:03/11/22 11:18
>>640
マルチしたのはここの板で答えが出なかったのであっちの板でも聞こうと思ったんです。
向こうにはレスついてるな。
結局どっちで聞くのかはっきり汁。
>>641
で、どこまで考えたわけ?
まさか丸投げ?
644132人目の素数さん:03/11/22 11:35
>>568も教えて下さい
645132人目の素数さん:03/11/22 11:36
>>635
(1)から全部??なのかい?
646132人目の素数さん:03/11/22 11:52
\subset
\neq
647132人目の素数さん:03/11/22 11:55
>>643>>645
(1)は昨日ひらめいてできました。(1)を後々使うのかもしれないので
一応問題文通りに書き写しました。
(2)からわかりません。よろしくおねがいします。
>>568
考えたいのは山々ですが、\subset とか\neqとか知らないし…
649568:03/11/22 12:16
すみません

2行目
{e_i} iは自然数

5行目
<e_i,u>=0 が1以外の全ての自然数iについて成り立つ

という意味です
650132人目の素数さん:03/11/22 12:27
⊆とか⊇とか⊂とか⊃とか
≠とか…
651132人目の素数さん:03/11/22 12:29
>>650
ぉぃ。だめだろ。
|
654132人目の素数さん:03/11/22 12:51
>>651
だめって何が?
>>868
今日、初めて夜鳴きやいってみた。
混むという噂を聞いてたから開店と同時に入ろうと思ったが甘かった。
11時到着ですでに満員。でも端っこに座れてセーフ。
同じく、「すぐ無くなるよ」と聞いてた細切り肉入りを注文。
これは11時10分には売り切れてたw

ラーメンは良くダシの出た醤油味で美味しかったです。
お新香も美味しい。細切り肉の脂身がウマイ。
しかし中ライス250円って・・・これで儲けてるのかな。
山科区に越して来たんだけど色々有るね。
「天」、「金ちゃん」「名門」とか。次は金ちゃん逝ってみます。
656132人目の素数さん:03/11/22 13:13
細麺との相性が抜群の秘伝のスープを味わおう
夜鳴きや
ヨナキヤ
 
毎日深夜3:00から作りはじめる鶏ガラや豚の背脂などで
丹念に取ったダシをベースに、醤油味に仕上げた中華そばが人気。
焼豚とモヤシ、ネギのみと具材はシンプルなので、独特の歯ざわりが
秀逸な細麺とスープの味わいが存分に堪能できる。好みに応じて、
濃口などのオーダーにもこたえてくれる。

お店データ 駐車場あり ランチ
住所 : 京都府京都市山科区
営業時間 : 朝11:00〜夜6:00
定休日 : 第1・3日曜日,月曜日
交通手段 : JR琵琶湖線山科駅から
京阪バス26系統北大塚下車、南へ、左側すぐ。
徒歩1分   駐車場 : 11台
657132人目の素数さん:03/11/22 13:17
>>651
\subset は ⊂
\neqは ≠
658132人目の素数さん:03/11/22 13:46
武術の神と言われる亀仙人より強い天津飯でも歯が立たないドラムを瞬殺した悟空が8年修行して、
同じ位の強さのピッコロさんと二人がかりでやっと倒したラディッツと匹敵する戦闘力を持つ栽培マンを
あっさり倒した天津飯・ピッコロ等が束になっても敵わないナッパを悠々倒した悟空の2倍以上強いベジータが
かなりパワーアップしても全く相手にならない強さのリクームを一撃で倒した悟空が更にパワーアップしても、
それを半分の力で殺せるフリーザをあっという間にバラバラにして消した未来のトランクスでさえ
仲間と束になっても敵わない人造人間17・18号に匹敵する強さを持った神コロ様でも敵わない程に
生体エネルギーを吸って強くなったセルと互角の16号を大きく越える17号吸収態セルを子供扱い出来る
精神と時の部屋パワーアップ後のベジータと随分差がある悟空でも勝てない完全体セルを
一方的に痛めつけることが出来るブチ切れ悟飯をも越えたベジータが命を賭けても倒せなかった魔人ブウが
更に凶悪になったブウと互角以上の戦いをしたゴテンクスよりも強くなった悟飯でも全く歯が立たない
ゴテンクス&ピッコロ吸収ブウが更に悟飯を吸収して強化しても全く歯が立たない
659hansin:03/11/22 14:48
y=f(x)=4x^2 - 8x + 9  ( x∈[-2,1]) の
逆関数 y=f(x)^(-1)を求めよ。逆関数の定義域、値域も明記すること。
よろしくお願いします。
660132人目の素数さん:03/11/22 14:49
a,zを複素数とするとき、次の命題(※)が成り立つようなa
の存在範囲を複素数平面上に図示せよ。
(※)|z|^2+2a(z+1)=0を満たすzは存在しない。
sin A=2/3のとき、cos A.tan Aの値、
cos A=2/5のとき、sin A,tan Aの値

なんか公式みたいなのがあったと思うんですが・・・見事に忘れてます。解説お願いします・・・。
662132人目の素数さん:03/11/22 14:57
>>661

sin^2(x)+cos^2(x) = 1
tan(x) = sin(x)/cos(x)
663こん:03/11/22 14:59
664132人目の素数さん:03/11/22 15:08
>>659
f(x)=4(x-1)^2 +5

-2≦x≦1の時
5≦f(x)≦41

x=4(y-1)^2 +5

x-5 =4(y-1)^2

2(y-1)=-√(x-5)
y=- {(√(x-5))/2} +1=f(x)^(-1)

定義域
5≦x≦41
値域
-2≦f(x)^(-1)≦1
665132人目の素数さん:03/11/22 16:19
>>607

>>559
y=-xtanθ
のθを固定する。 (

yとxの比が保たれるということは、
原点を通る、ある直線上のx,yだけを考えるということ。


x={(r^2-2rcosθ+1)/2r}cosθ

は、rの連続関数である。
cosθ>0なら
r→∞の時  x→∞
r→+0の時 x→-∞

これはy=-xtanθ上の点を全て表せている事を意味する。

cosθ< 0なら
r→∞の時 x→-∞
r→+0の時 x→∞
>√112aが整数になるような自然数aのうち、最小の値を求める。

これってどうやるんすか?(;´Д`)
667132人目の素数さん:03/11/22 17:18
>>620
(1)の結果を信じてくださるのは有難いのですが、b_n = 1/√5 * [{(1+√5)/2}^n - {(1-√5)/2}^n]であることは、
問題文にかかれていませんし、推測も難しいかと……。(1)だけですが、解答を掲載させてください。

先に、任意の自然数nに対して、命題" a_n = p_n+q_n√5 を満たす正の有理数 p_n, q_n が存在する。"……………@
が成立することを数学的帰納法で証明する。
(T)n=1のとき
   a_1 = 1/2(1+√5/5) = 1/2+√5*(1/10) であるから、p_1 = 1/2, q_1 = 1/10 が存在し、@が成立する。
(U) n = k( k は自然数 )のとき、@の成立を仮定すると、
   a_k = p_k+q_k√5 を満たす正の有理数 p_n, q_nが存在して、a_k+1 = (1+√5)/2*(p_k+q_k√5) = (p_k+5q_k)/2+(p_k+q_k)*√5/2 であるから、
   n = k+1のときも(*)を満たす正の有理数 a_k+1 = (p_k+5q_k)/2, q_k+1 = (p_k+q_k)/2, が存在する。
 よって、(T), (U)より、任意の自然数nに対して、 @が成立する。(証終)

 @より、数列{p_n},{q_n}は、p_1 = 1/2, q_1 = 1/10 p_n+1 = (p_n+5q_n)/2, q_n+1 = (p_n+q_n)/2 ( n = 1,2,…… )……………Aで定められるから
 p_n+1-q_n+1√5 = (p_n+5q_n)/2-(p_n+q_n)*√5/2 = (1-√5)*(p_n-q_n√5)/2
 ゆえに、{p_n-q_n√5}は初項:p_1-q_1√5 = -(1-√5)/2√5 項比:(1-√5)/2 の等比数列で、p_n-q_n√5 = (-1)/√5*{(1-√5)/2}^n
よって、r_n = p_n-q_n√5 とおくと、 a_n = (1/√5)*{(1+√5)/2}^n = p_n+√5q_n r_n = (-1)/√5*{(1-√5)/2}^n = p_n-√5q_n

a_n + r_n = 2p_n, |r_n|<1/2 であることとAより、q_n = (2p_n+1-p_n)/5, q_n+1 = (p_n+1+q_n)/2 q_nを消去して、
 (2p_n+2-p_n+1)/5 = p_n/2+(p_n+1-p_n)/10
p_n+2 = p_n+1+p_n
 
ここで2p_n+2 = 2p_n+1+2p_nとすると、2p_1 = 1,2p_2 = 1 であるから、帰納的に2p_n( n = 1,2,…… )はつねに整数である。
 a_n = 2p_n-r_n, |r_n| < 1/2 より、 2p_n-1/2 < a_n <2p_n+1/2 となり、a_nに最も近い整数b_nは、
 b_n = 2p_n = a_n+r_n = (1/√5)*[{(1+√5)/2}^n-{(1-√5)/2}^n]

 (2)自分も未だだめぽ。何方か助けて下さい。
668132人目の素数さん:03/11/22 17:22
ギャンブルのモンテカルロ法を証明してください。
669661:03/11/22 17:31
>>662
うーん計算が消化しきれない・・・。
670k:03/11/22 17:39
次の合同式を解け。
(1) 8x≡7 (mod19)
(2) 31x≡2 (mod13)
よろしくお願いします。
671k:03/11/22 17:46
lim[x→±∞](x + 1/x)^x = e は用いてよいと仮定して、次の極限値を求めよ。
lim[x→∞]((2x-1)/(2x+1))^3x
>>666
√(112a) = √(16*7a) = 4*√(7a) なので a=7

>>670
(1)8
(2)3
673132人目の素数さん:03/11/22 17:55
668 名前:132人目の素数さん :03/11/22 17:22
ギャンブルのモンテカルロ法を証明してください。

そんなの証明できるほど頭のいいヤシは2ちゃんに来ない。
674OK:03/11/22 17:56
G(x)=∫[0→3x] (t-1)(t-2)(t-3) dtとおくとき、
 dG/dx<0となるxの範囲を求めよ。
よろしくお願いします。
675132人目の素数さん:03/11/22 18:16
Sin[Pi/180]とCos[Pi/180]の値っていくつですか?
小数じゃなくて↓のサイトみたいなかんじでおながいします
ttp://www.gifu-nct.ac.jp/sizen/nakasima/trilist.htm
676132人目の素数さん:03/11/22 19:01
>674
微分すれば良いんじゃないの?
677132人目の素数さん:03/11/22 19:05
>>665
計算違っとる

x={(r^2-2rcosθ+1)/2r}cosθ
= {(r-cosθ)^2 +1-(cosθ)^2} (cosθ)/(2r)
cosθ>0
r→∞の時  x→∞
r→+0の時 x→∞
だ。放物線だからな。

これは、最小値は相加相乗平均を使うことにより
x= { r +(1/r) -2cosθ}(cosθ)/2 ≧ {1-cosθ}(cosθ)

cosθ<0の時も
最大値は
x≦ {1-cosθ}(cosθ)
になっとる

つまり、y=-xtanθの内 (-π/2<θ<π/2)
{1+cosθ}(-cosθ) <x<0, 0<x< {1-cosθ}(cosθ)
の部分が解?
678132人目の素数さん:03/11/22 19:06
>>677
×放物線だからな。
○分子が放物線だからな。
>>667
予測がどうこうって、そういうことじゃないんだけどね・・・。

そんなことよりも問題を解く以前の話で、
a_nの正しい式は何?

>>608ではnの式になってないんだけど
a_n=(1/√5)*{(1+√5)/2}^n
これでいいの?

ていうか模範解答を自分で持っているのに何が不満なんだ。
模範解答を読んでも意味がわからないのか、
>>620が理解できないのか、どっちよ。
あと、予測がつくつかないの話は
本来どうでもいいことなんだけど
一応説明すると、

B(1)=B(2)=1 B(n+2)=B(n+1)+B(n)
この漸化式を解くと
B(n)=(1/√5)*[{(1+√5)/2}^n-{(1-√5)/2}^n]

・・・という結果を知っている人ならば
a_n=(1/√5)*{(1+√5)/2}^nという式が与えられたときに
何をしたらいいか想像がつくわけ。

この漸化式の固有方程式の2根が
α=(1+√5)/2 β=(1-√5)/2 であって
|β|<1なので|a_n-B(n)|=(1/√5)*|β^n|<1/2

以上からB(n)はb_nの性質を満たすので
b_n=B(n)としてよい、と。

(2)は何がわからないのかわからない。
>>620は答えそのものだよ。

また同じことを書くと、
(1)の結果からa_nにもっとも近い整数がb_nとわかったと。
nによってa_nとb_nの大小関係がかわるのがミソ。
例えば、もしa_n<b_nならばc_nはb_nそのもの。

a_n-b_n=(1/√5)*β^nであって、いまβは負なので
nの奇遇で符号が変わる(大小が変わる)ということ。
予測->証明が気に食わないのかも。ハミ出シ削リ論法など言語道断。
682132人目の素数さん:03/11/22 20:32
668 :132人目の素数さん :03/11/22 17:22
ギャンブルのモンテカルロ法を証明してください。
683132人目の素数さん:03/11/22 21:21
>>677
違うと思う。
cosθ>0でもcosθ<0そのθに対応したtanは実数全体を取りうる。677さんの議論から、tanを固定するとθに関係なくxは実数全体を動く(tanθ一つに対してcosが2つあるから)。
よって、結局y=-xtanθで表現できないのは虚軸上……かな?
http://www2.makani.to/akutoku/upload/dat/1069505233.jpg

問)正四面体を、向かい合う辺の中点を通る直線の周りに回転させた時
の体積をa,xを用いてあらわせ。
お願いしまつ
周り出来てるのに一人わからなくて鬱だった…
685132人目の素数さん:03/11/22 22:06
y=√(1-x^2)
単位円の上半分です。こいつの第n次導関数って簡単な式で表せますか?
表せるとしたら、教えて下さい。お願い。
686132人目の素数さん:03/11/22 22:10
>>683
tanθは, 実数全体を取るけど
y=-xtanθ
のtanθは傾きね。

θに関係なくxは実数全体を動くわけではなく
θの固定により、

cosθ>0なら
x≧ {1-cosθ}(cosθ) >0
cosθ<0なら
x≦ {1-cosθ}(cosθ) <0 (cosθの符号に注意)

なので、y=-xtanθ上だけで見ると
0の近傍は、この区間に入りません。

tanθに対応するcosθは 2つあるように思えますがこれは、cosθと -cosθ=cos(θ+π)です。
なので、xが動ける範囲は
x≧ {1-cosθ}(cosθ) >0

x≦ {1-cos(θ+π)}(cos(θ+π))={1+cosθ}(-cosθ)になり
y=-xtanθの上全体を動けるわけではないです。
687132人目の素数さん:03/11/22 22:14
>>684
403:アクセス不可

22時〜2時は、一部アクセス制限をしています。アップロードは、常時可能です。
また、同時ダウンロード数が多いIPも制限しています。
2chからの直リンは、常時制限しています。2ch以外のスレッド掲示板からのリンクは制限していません。

アクセス制限の内容

* ファイルへの直接リンク(アップローダのページからクリックすれば、見れます)
688132人目の素数さん:03/11/22 22:16
>>685
簡単には無理
689132人目の素数さん:03/11/22 22:19
>>688
そうですか。では√(1-x^2)の級数展開って無理ですかね?
690132人目の素数さん:03/11/22 22:21
>>689
フーリエ級数展開はどうだろう?
691132人目の素数さん:03/11/22 22:21
中3なんだけど
x+1=4
この方程式どうやって解けばいいの?
693132人目の素数さん:03/11/22 22:23
>>690
やってみますです。
694685:03/11/22 22:24
>>692
おれ二回しかカキコしてない。あとの一回は知らない。
>>685
簡単な式?
なにがどうなってんのか知らないけど>>551ならa=0はz=0で成立してしまうのでダメ。
a≠0のときa=r(cosθ+isinθ)とおいて
(※)|z|^2+2a(z+1)=0を満たすzは存在しない。
⇔|w+1|^2≠2aw (∀w)  ←w=-(z+1)とおいた。
⇔a≠1/2 & a≠(1/(2w))|w+1|
⇔a≠1/2 & a≠(1/2)λ(cosη+isinη)|1+(1/λ)r(cosη-isinη)| ∀λ>0 ∀η (←1/wを極形式にした)
⇔a≠1/2 & a≠(1/2)(cosη+isinη)|λ+(cosη-isinη)| ∀λ>0 ∀η
⇔a≠1/2 & r≠(1/2)|λ+(cosθ-isinθ)| ∀λ>0
 
そこで(1/2)|r+cosθ-isinθ| (λ>0)の値域をもとめればよい。
それは容易で結果は
「f(λ)=(1/2)|r+cosθ-isinθ| (λ>0)の値域」=「|sinθ|≦f<∞」。
つまりaのとりうる値は(極形式なら)
a≠0,1/2 & 0<r<|sinθ|
という領域だと思うけど。
697132人目の素数さん:03/11/22 22:38
>>696
>|w+1|^2≠2aw (∀w) 
>⇔a≠1/2 & a≠(1/(2w))|w+1|

この変形は、どうなってんの?
698132人目の素数さん:03/11/22 22:41
>>551 あらら こんなの如何?(藁
|z|^2+2a(z+1)=0 −@
これを満たす z が存在するとき
|z|^2=-2a(z+1)
a=0 のとき z=0
a≠0 のとき z+1=ta~ ⇔ z=-1+ta~ (tは実数) と表せて、@へ代入すると
(-1+ta~)(-1+ta)+2t|a|^2=0 ⇔ (|a|^2)t^2-(a+a~-2|a|^2)t+1=0
この2次方程式の判別式をDとすると、実数tが存在する条件は
D=(a+a~-2|a|^2)^2-4|a|^2≧0 −A
ここで a=r(cosθ+i*sinθ) (0<r、0≦θ<2π) とおくと
A ⇔ (cosθ-r)^2-1≧0 ⇔ 1+cosθ≦r
よって求めるaの範囲は r≦1+cosθ つまり、Cardioid:r=1+cosθ で囲まれる領域である。
699132人目の素数さん:03/11/22 22:52
>>698 がっはははは 最後の行を訂正!
× よって求めるaの範囲は r≦1+cosθ つまり、・・・
○ よって求めるaの範囲は r<1+cosθ つまり、・・・
700132人目の素数さん:03/11/22 22:57
>>698
a=r(cosθ+i*sinθ) (0<r、0≦θ<2π) とおくと

D=(2r cosθ -2|a|^2)-4|a|^2≧0
(r cosθ -|a|^2)^2 -|a|^2 ≧0
(rcosθ)(r cosθ-2|a|^2)≧0

になると思うんだけど

(2) ⇔ (cosθ-r)^2-1≧0
は、どういう変形?
701132人目の素数さん:03/11/22 22:58
ああそういうことか >>700取り消し
702132人目の素数さん:03/11/22 22:59
悪くない。
703132人目の素数さん:03/11/22 23:16
>>698-699が正解だろうな

>>696はあまり読んでないけど変な所あるし
間違いだろう
704696:03/11/22 23:45
スマソ。式変形の2行目以降2乗がぬけてた。
 
⇔|w+1|^2≠2aw (∀w)  ←w=-(z+1)とおいた。
⇔a≠1/2 & a≠(1/(2w))|w+1|^2
⇔a≠1/2 & a≠(1/2)λ(cosη+isinη)|1+(1/λ)r(cosη-isinη)|^2 ∀λ>0 ∀η (←1/wを極形式にした)
⇔a≠1/2 & a≠(1/2)(cosη+isinη)|(√λ)+1/(√λ)(cosη-isinη)|^2 ∀λ>0 ∀η
⇔a≠1/2 & r≠(1/2)|(√λ)+1/(√λ)(cosθ-isinθ)|^2 ∀λ>0
 
そこでf(t)=(1/2)|t+(1/t)(cosθ-isinθ)|^2=t^2+(1/t)^2+2cosθ (t>0)の値域をもとめればよい。
それは容易で結果は
「f(λ)=の値域」=「|1+cosθ|≦f<∞」。
つまりaのとりうる値は(極形式なら)
・・・
以下>>698さんの答えに同じ。
705132人目の素数さん:03/11/22 23:52
>>704
よくわからないんだけど
a≠1/2を別にする理由は?
706696:03/11/23 00:03
>>705
もっとちゃんとかくと
・・・
⇔|w+1|^2≠2aw (∀w)  ←w=-(z+1)とおいた。
⇔|w+1|^2≠2aw (∀w≠0) &|0+1|^2≠2a・0
⇔a≠1/2 & a≠(1/(2w))|w+1|^2
・・・
つまり式変形の途中でwをまとめるために両辺をwで割りたいので
(∀w)での不成立を(∀w≠0)と(w=0)での不成立にわけて前者だけ割り算して
式変形を続行したという意味。最終的にはまとまるみたいだけど。
707132人目の素数さん:03/11/23 00:03
1から1000までの自然数をすべて書き並べるとき、
6という数字は何個あらわれるか。

お願いします
>>707
勘で100個
709132人目の素数さん:03/11/23 00:11
>>706
ますます分からない。

w=0はもともと
|w+1|^2≠2aw
を満たしている。

「aの値に関わらず」常に
|0+1|^2≠2a・0=0
だからだ。

で、a≠1/2はどこから出てくるんだい?
710132人目の素数さん:03/11/23 00:13
>>707
1〜100までは、11個。
101〜200までも、11個。
201〜300までも、11個。
601〜700までのときだけ、12個。
ってことで、
11+11+11+11+11+11+12+11+11+11=111
答え111個
711132人目の素数さん:03/11/23 00:14
601〜700までのときだけ、12個。
601〜700までのときだけ、12個。
601〜700までのときだけ、12個。
・・・・・・
712132人目の素数さん:03/11/23 00:15
>>707
1〜100までは、11個。
101〜200までも、11個。
201〜300までも、11個。
601〜700までのときだけ、112個。
ってことで、
11+11+11+11+11+11+112+11+11+11=211
答え211個
713132人目の素数さん:03/11/23 00:17
>>707
1桁…1個
2桁…9+10=19個
3桁…9*20+100=280個

全部で、300個
1の位に100個
10の位に100個
100の位に100個
計300個、じゃないの?
715132人目の素数さん:03/11/23 00:19
>>707
1桁…4個
2桁…19+10=29個
3桁…9*40+300=280個

全部で、万個
>>709
ああ、ゴメンw=0を代入した式ときそこなってた。|0+1|≠2a・0は恒に成立しないね。
脳内で↓の左辺に0いれて右辺に1いれてた。
 
>⇔|w+1|^2≠2aw (∀w)  ←w=-(z+1)とおいた。
>⇔|w+1|^2≠2aw (∀w≠0) &|0+1|^2≠2a・0
 
書いてるとき脳内で計算した計算まちがいに気付かなかった・・・・スマソ。
717707:03/11/23 00:22
みなさまありがとうございます。
1から100まで、単純にかぞえて
6,16,26,36,46,56,60,61,62,63,64,65,66,67,68,69,76,86,96
で20ですね。
718707:03/11/23 00:25
1から2000までの自然数の中で、
少なくとも1つは8という数字を含むものはいくつあるか。

お願いします
300個だった。0123456789から重複をゆるして3つならべた
リストをつくるとき6を使う回数だから(リストには000がでてきて1000が
でないけどこの2つには6がないから無視して)
当然0がでる回数=1がでる回数=・・・=9がでる回数。計3000文字あるので
6は300回でる。
・・・一組3文字つかうのうっかりして1000÷10してた。0〜999なので0も1〜9と
同等にあつかえるから楽勝だとおもうんだけど。
720132人目の素数さん:03/11/23 00:29
>>716
それと言葉を正しく使うべき

|0+1|≠2a・0は常に成立している。
|0+1|=2a・0は常に成立しない。

≠を使っての変形なのか、=を使っての変形なのかの違い

>|0+1|≠2a・0は恒に成立しないね。

これは
恒に|0+1|=2a・0が成立してるね。
と言ってるのと同じ。
721132人目の素数さん:03/11/23 00:31
6 16 26 36 46 56 66 76 86 96 106 116 126 136 146 156
166 176 186 196 206 216 226 236 246 256 266 276
286 296 306 316 326 336 346 356 366 376 386 396
406 416 426 436 446 456 476 486 496 506 516 526 \
536 546 556 566 576 586 596 606 616 626 636 646 656 66\
6 676 686 696 706 716 726 736 746 756 766 776 786 7\
96 806 816 826 836 846 856 866
876 886 896 906 916 926 936 946 956 966 976 986 996 \

数えたら、117個
722132人目の素数さん:03/11/23 00:32
>>721
消防はもう寝なよ
>>720
スマン。いろいろ間違ってたのはあやまるけどもう勘弁してよ。
def count6(n)
if(n==0)
return(0)
end
nn=(n-n%10)/10
if(n%10==6)
return(count6(nn)+1)
else
return(count6(nn))
end
end

sum=0
l=0
for n in 1..1000 do
c=count6(n)
if c!=0
l=l+1
if l==10
l=0
puts
end
end
sum=sum+c
end
puts
puts sum

結果
300
725132人目の素数さん:03/11/23 00:58
>>721
600番台は全てに6がありますので
省略せずに全部書いてください
>>707
1000を000として、000や001なども3桁の数とみなすことにする。
このとき3桁の数は合計1000個。数字は3*1000個出現する。
0,1,2,・・・,9の10種類の数字はそれぞれ均等に出現するので
6の出現個数は3*1000/10=300個
727132人目の素数さん:03/11/23 10:15
>>686
x≦ {1+cosθ}(-cosθ)
に(-cosθ)が付いてるのは
動経方向ではなく、x軸方向だからだな
って今更誰も気にしてないか(w
728132人目の素数さん:03/11/23 11:24

               ノ      ゚.ノヽ  , /}
            ,,イ`"     、-'   `;_' '
   ,-、  _.._,,-'' ̄         (,(~ヽ'~
 )'~  レー' ▒            i`'}
 ~つ   ░▓▒░           | i'
 /   ▒▓█▓░       。/   !
/     ░▓▓▒       /},-'' ,,ノ
i、        ,i' _,,...,-‐-、/    i
..ゝ        <,,-==、   ,,-,/
 )       {~''~>`v-''`ー゙`'~
 {        レ_ノ
ノ         ''

ずれ過ぎ(w
730132人目の素数さん:03/11/23 12:14
まぁ、趣向を変えてちょっとしたクイズだ

1円玉が(  )枚あります。
これらを出来るだけ5円玉に両替すると
「硬貨」の総数が「60枚減る」ことになりました。
さらにこれらを出来るだけ10円玉に両替すると
「硬貨」数は「10枚だけ」になる

(  )に埋まる数字を答えよ
ガイシュツ
77
あの数学の質問ではないのですが ここのスレで様々な問題に答えている
いわば数学のスペシャリストの方々にお聞きしたいことがあります。
1、どのような職についているか?
2、数学の勉強は一日どのくらいしているか?
3,、数学の力をつけるためにはどんな努力が必要だと思うか? 
4、趣味は?
5、勉強に手がつかないときどうするか?
6、平均睡眠時間は?

私は高校生なのですが数学ができる人をとても尊敬しています。私からしてみれば神の
ような存在の方々に私が疑問に思っていることを書いてみました。答えて頂けたら幸いです。
>>733
> 1、どのような職についているか?
大学生
> 2、数学の勉強は一日どのくらいしているか?
宿題のみ
> 3,、数学の力をつけるためにはどんな努力が必要だと思うか?
どんな努力でもいい。努力するかしないか
> 4、趣味は?
ゲーム制作。エロゲ。2ch。
> 5、勉強に手がつかないときどうするか?
ゲーム制作。エロゲ。2ch。
> 6、平均睡眠時間は?
8時間くらい
>>733
1.無職(フリーターとも言う) 2.してない 3.妄想すること
4.ない 5.寝る 6.約8時間
典型的ダメ人間ぽ
736132人目の素数さん:03/11/23 16:09
>>735
俺も似たようなもん
737132人目の素数さん:03/11/23 16:12
ProductLogって何ですか?教えて下さい、お願いします!
738132人目の素数さん:03/11/23 16:14
>>734-736はいつもハズレな解答or難しい問題はスルーばかりしている
数学板住人気取りなので>>733は見習わないようにね☆
739293:03/11/23 16:26
1:大学生(SEで内定)
2:高校1,2年のときは週に3時間ほど。
  高校3年のときは1日2,3時間。
 大学いってからは・・・・皆無
 ちなみに、自習時間(宿題含まない)
 まぁ、数学しか勉強してなかったわけですが・・・
3:高校生なら問題といとこう。わからんかったら調べよう
たまに、大学生がよむような本を読むのもいいかも
4:囲碁、オンラインゲーム
5:勉強しない。やる気がでるまでまつ
6:最近は12時間(死
 高校生のときは8時間

私も>>738に当てはまる人間
740132人目の素数さん:03/11/23 18:10
あっ、復活した
741132人目の素数さん:03/11/23 19:38
>>738
オマエモナー
742132人目の素数さん:03/11/23 19:44
>>738
>いつもハズレな解答or難しい問題はスルーばかりしている
99%の数学板住人がこれに当てはまると思われ

743132人目の素数さん:03/11/23 19:45
>>737
z exp(z)=定数
の解が

ProductLog[z]
744132人目の素数さん:03/11/23 19:46
745738:03/11/23 19:52
>>741>>742
( ´,_ゝ`)プッ
ハズレが必死だなwww
746132人目の素数さん:03/11/23 19:59
>>745がどんな問題でも解いてくれるようです。
747132人目の素数さん:03/11/23 20:08
煽りに熱くなるな
俺達は目の前の問題を解いていけばいい
無理な物は仕方ない
仕方ないが、それでも自分のできることを
していくしかない
748132人目の素数さん:03/11/23 22:46
そうだそうだ
1<α<16   y=-x2乗+2(log2α)x+log2β
(0≦x≦4)の最大値が5、最小値が−4となる定数α、βの値を求めよ。

よろしくお願いします。
750132人目の素数さん:03/11/23 23:07
1x2x3x4x5x6x7x8x9x10の積は?
751132人目の素数さん:03/11/23 23:09
>>749
y= -x^2 +2(log_{2} α)x + log_{2} β
p=log_{2} α
q=log_{2} β
と置く

1<α<16より
0<p<4
y=-x^2 +2px+q=-(x-p)^2 +p^2 +q
最大値は
x=pのところ、 y=p^2 +q
p^2+q=5
q=-p^2+5
最小値は
0<p<2の時
x=4のところ、y=8p+q-16= -p^2 +8p-11= -(p-4)^2 -27 <-27
なので-4とはなるpは無い。
2≦p<4の時
x=0のところ、y=q= -p^2+5 = -4
p^2=9
2≦p<4を満たすpは
p=3
q=-4
よって、α=2^3=8, β= 2^(-4)=1/16
752132人目の素数さん:03/11/23 23:10
>>750
3628800
>>752
xはどこに消えた?
>>751
どうもありがとうございます。
755132人目の素数さん:03/11/23 23:16
10・n本の中にn本の当たりくじがある。
くじ3本を引き、少なくとも1本当たりくじがある確率を求めてください。
>>752
答えは3628800ではないようでが・・・
757132人目の素数さん:03/11/23 23:21
sardの定理の証明を読んでいてわからなくなりました。
R^nの部分集合Aが測度0であるとは、
任意のε>0について、開立方体の列Qn(n∈N)が存在して、
A⊆∪Qnで、Σ|Qn|<εと出来ることをいいます。(|Qn|はQnの体積)
ここで、
Aを、ある閉立方体Q(r)に含まれているとすると、(一辺の長さがr)
任意のε>0について、開立方体の列Qnを考えたとき、
rを十分大きくすれば各QnがQ(r)に含まれるように出来るのでしょうか?
感覚的には出来ると思うのですが、いまひとつピンときません
758132人目の素数さん:03/11/23 23:24
>>755
10n本から3本引く組み合わせは

C[10n, 3]=(10n)(10n-1)(10n-2)/(3*2*1)

はずれ9n本から3本引く組み合わせは
C[9n, 3]=(9n)(9n-1)(9n-2)/(3*2*1)

一本もあたりを引かない確率は

{(9n)(9n-1)(9n-2)}/{(10n)(10n-1)(10n-2)}

少なくとも1本当たりくじを引く確率は
1-{{(9n)(9n-1)(9n-2)}/{(10n)(10n-1)(10n-2)}}
759132人目の素数さん:03/11/23 23:26
>>756
その問題がどういったところで出されている
どういう問題なのか説明しないと
どう答えていいのかわからんだろう。

クイズならラウンジへでも行ってくれ。
760132人目の素数さん:03/11/23 23:31
Z=f(y+ax)+g(y-ax)に対して偏微分
Zx=af'(y+ax)-ag'(y-ax)となるのはどうしてですか?
よろしくお願いします。
761755:03/11/23 23:32
>>758
ありがとうございます!
762132人目の素数さん:03/11/23 23:33
lim[n→∞] [1-{{(9n)(9n-1)(9n-2)}/{(10n)(10n-1)(10n-2)}} ]
を求めてください
>>762
 {(9n)(9n−1)(9n−2)}/{(10n)(10n−1)(10n−2)}={9(9−1/n)(9−2/n)}/{10(10−1/n)(10−2/n)}
  →(9/10)^3 (n→∞)
を使えば解けるだろう?
>>760
この場合、xによる偏微分は、yを定数と思って単にxで微分するのと同じだからだ。
>>763
読みづらい。見づらい。氏ね。
766132人目の素数さん:03/11/23 23:53
>>757
>Aを、ある閉立方体Q(r)に含まれているとする

>A⊆∪Qnで、Σ|Qn|<ε

だから、Qnの一辺は ε^(1/n)より小さい。

ので、もし、Q(r)からはみ出したとしても、大きく見積もってさえ
ε^(1/n)もはみ出せない。

よって、はみ出しても、一辺がr+ε^(1/n)くらいの閉立方体で
覆うことができる。
767132人目の素数さん:03/11/23 23:56
「ProductLogって何ですか?」
という質問をしたら、
「z exp(z)=定数  の解がProductLog[z]」
という答えを頂いたのですが、この“定数”っていうのがいまいち意味が・・・
定数って何なの??どゆ意味?(もちろん“定数”という単語の意味はわかりますよ。)
768132人目の素数さん:03/11/23 23:57
>>766
なるほど!確かにそのとおりですね。ありがとうございます。
>>757
一次元ユークリッド空間で考える。
A:={0,1,2,…}は、Qn:=[n−ε/3,n+ε/3]と置くことにより測度0であることが分かるが、
全てのQnをQ(r)に含まれるようにするr>0は存在しない。
770132人目の素数さん:03/11/24 00:03
>>767
それ間違い。
正しくは、
z exp(z)=wの解がProductLog[w]だろ。
実数解だったかな?
771132人目の素数さん:03/11/24 00:05
772132人目の素数さん:03/11/24 00:09
>>770
よくわかりました!
z exp(z)=w ⇔ z=ProductLog[w]
ってことすね。あと、書くときもいちいちProductLogって書くんですか?
773132人目の素数さん:03/11/24 00:10
>>771
ありがとうございます。
774132人目の素数さん:03/11/24 00:17
(A*sin(wt-kx))+(B*sin(wt+kx+d))

上式をXsinYの形に直すことってできるの?

いや合成波の問題なんだけどさ。
>>774
合成するだけだろ?
776132人目の素数さん:03/11/24 00:26
日夲語 しゃべれ おまいら バーカ
>>776
日夲語 おしえれ もまい ヴァーカ
778132人目の素数さん:03/11/24 00:33
>>777
日夲語って 何だ もまい ぱか!
779774:03/11/24 00:33
>>776は俺じゃないぞ。

できれば途中経過教えてください。
780magnus:03/11/24 00:36
宝くじの共同購入をして分配すると、同数量を個人購入した場合よりも収益率が下がりますよね?
781774:03/11/24 00:39
途中経過教えれ もまい ヴァーカ
782777:03/11/24 00:41
>>778
>>776に訊いてくれ。

>>779
そんなこと思ってる香具師なんかオメーしかいねーって。
783774:03/11/24 00:41
あーあ、マジうぜえわこいつら
つうか解けないからって文句言うのってアホらしくねーの?www
あーくだらねー。しつけー馬鹿ばっかだyここ
>>783
そのコピペ飽きた。
785774:03/11/24 00:44
ここにいる奴馬鹿ばっか。死ねば?
生きててはずかしくないわけ??wwwwwwwwwwwwwwwww
786774:03/11/24 00:45
>>784
「飽きた」だぁ?調子こくんじゃねぇぞ偉そうに。この俺様のコピペを
飽きるなんざ100年早いんだタコ
>>785
しばらく見ないうちに、此処の煽りのレヴェルも落ちたね。
昔はもっと歯ごたえがあったものだけれど・・・。
788774:03/11/24 00:46
偽者ばっか(w
>>786
少ない語彙で捻りのない文章、ご苦労様。
790132人目の素数さん:03/11/24 00:47
>>780

10人で買ってあたれば、当選金を10等分だけど
出資金も10等分されているので
収益率は等しい。

当たらなければ、0円だし。
791774:03/11/24 00:47
必死な奴がいるぞ。誰か相手してやれ(プゲラ
792774:03/11/24 00:48
死ね・くたばれ・消えろ・失せろ・潰れろ・
馬鹿・あほ・間抜け・ドジ。 ポンコツ・
トンチキ・ガラクタ・クズ・ゴミ・カス・最低以下の下劣・
下等種族。 劣等種族・下衆野郎・腐れ外道・邪道・外道・
非道・ウジ虫・害虫・ガン細胞。 ウィルス・ばい菌・
疫病神・病原体・汚染源・公害・ダイオキシン・有毒物質。
廃棄物・発ガン物質・猛毒・毒物・アメーバ・ダニ・ゴキブリ・
シラミ・ノミ。 毛虫・蠅・蚊・ボウフラ・芋虫・掃き溜め・汚物
・糞・ゲロ・おりもの・糞虫野郎・ほら吹き。基地外・デタラメ・ハッタリ・
穀潰し・ろくでなし・ごろつき・ヤクザ者。社会の敵・犯罪者・反乱者
・前科者・インチキ・エロ・痴漢・ゴミ・シデムシ。 ゴミ虫・毒虫
・便所コオロギ・詐欺師・ペテン師・道化師・危険分子・痴呆・白痴。
魔物・妖怪・悪霊・怨霊・死神・貧乏神・奇天烈・奇人・変人・毒ガス
・サリン。 ソマン・マスタードガス・イペリット・クソブタ・ブタ野郎
・畜生・鬼畜・悪鬼。邪気・邪鬼・ストーカー・クレイジー・ファッキン
・サノバビッチ・シット・ガッデム。小便・便所の落書き・不要物・障害物
・邪魔者・除け者・不良品・カビ・腐ったミカン。土左衛門・腐乱・腐臭
・落伍者・犯人・ならず者・チンカス・膿・垢・フケ・化膿菌。 放射能
・放射線・鬼っ子・異端者・妄想・邪宗・異教徒・恥垢・陰毛・白ブタ。
ケダモノ・ボッコ・ろくでなし・VXガス・ヒ素・青酸・監獄・獄門
・さらし首・打ち首・市中ひきずり回し・戦犯・絞首刑・斬首・乞食・
浮浪者・ルンペン・物乞い。 放射性廃棄物・余命1年・アク・割れたコップ
・精神年齢7歳・3審は必要なし。 不良品、規格外、欠陥品、不要物、
埃、掃き溜め、吹き溜まり、塵埃、インチキ、居直り。 ふてぶてしい
、盗人、盗賊、残忍、残酷、冷酷、非情、薄情者、ガキ、クソガキ。
ファッキン、ガッデム、サノバビッチ、シット、ブルシット、ボロ
、ボッコ、妄信。 狂信者、有害物質、毒薬、猛毒、発ガン物質、
誇大妄想狂。 他人の悪口は山ほどほざくが反省は一切しないガキ根性野郎
、腐れ根性。 腐って歪んだプライドの持ち主、狭量、ボケ、ボケナス、
アホンダラ、たわけ、寄生虫。>>1は死ね・くたばれ・消えろ・失せろ・潰れろ・馬鹿・あほ・間抜け・ドジ。
>>791
わかった。必死な奴=>>791、ガンガレ。
794774:03/11/24 00:49
>>791
俺はパス
粘着は嫌いだからな
795774:03/11/24 00:50
君たち、醜い。
やめようよ!争いなんて!

でへでへ。俺かこいい
796774:03/11/24 00:51
で、質問の答えはどうなったのでしょうか?
797magnus:03/11/24 00:51
>>790
ところが、当選本数が一本以外のときは、共同購入して分配した時の収益率が下がったのです。微妙に。
シミュレーションはまだしていませんが。
当選本数増やすほど、ドンドン減っていくのです。
誤差ですかね〜。

>10人で買ってあたれば、当選金を10等分だけど
出資金も10等分されているので
収益率は等しい。
当たらなければ、0円だし。

780>>宝くじの共同購入をして分配すると、同数量を個人購入した場合よりも収益率が下がりますよね?
798774:03/11/24 00:52
>>795
俺の方がかっこいいよヴァーカww
799774:03/11/24 00:53
774たちも必死すぎ(w
800 :03/11/24 00:53
ちょっとしつれい・・・・・

理科大二部受験生の皆様へ
11月21日〜24日まで開催される理大祭にて、二部受験を希望しているのに赤本がない!!という声にお答えして、解答つき問題集を配布いたします。
冊子はインフォメーションセンターか354教室にて。(神楽坂キャンパスのみ)
二部希望者は今すぐGO!
詳細は以下に載ってます。

http://www.ed.kagu.tus.ac.jp/suuken2/guide/
801774:03/11/24 00:53
もう教えてくれなくて良いです。
802774:03/11/24 00:53
(A*sin(wt-kx))+(B*sin(wt+kx+d))

上式をXsinYの形に直すことってできるの?

いや合成波の問題なんだけどさ。
phase ずらせば?
804774:03/11/24 00:54
(A*sin(wt-kx))+(B*sin(wt+kx+d))

上式をXsinYの形に直すことってできるの?俺必死です。
誰か教えれヴァカもまえヴァカ
805774:03/11/24 00:55
(A*sin(wt-kx))+(B*sin(wt+kx+d))

上式をXsinYの形に直すことってできるの?俺必死です。
誰か教えれヴァカもまえヴァカ
ようやくこのスレの本来の姿に戻ったな。がんがれ>>774ども
807774:03/11/24 00:55
ワロタ
808774:03/11/24 00:56
いちぬけぴっぴ
809774:03/11/24 00:57
もれも、もうこんなくだらねーことやめる。だから最初の質問も取り消す!
>>806
うまいことやったな。
811132人目の素数さん:03/11/24 00:58
∫e^(-x^2)dx ってできますかね?
(・∀・)ニヤニヤ
813132人目の素数さん:03/11/24 01:01
>>811
区間が0から無限だったら超有名だもまいヴァカ
814811:03/11/24 01:02
>>813
そんなの知ってます。不定積分のときはどうかって聞いてるんです。
で、此処ってネタスレだよな? 質問してもいいし、煽ってもいいし
荒らしてもいいんだよな? >>1 には何も書いてないんだし。
816811:03/11/24 01:03
あーあ、マジうぜえわこいつら
つうか解けないからって文句言うのってアホらしくねーの?www
あーくだらねー。しつけー馬鹿ばっかだyここ
>>814
原始関数は存在するよ。
>>816
だから、もうそれ飽きたって。
819811:03/11/24 01:03
>>815
真面目に答えてください。本当に困ってるんです。
820811:03/11/24 01:04
>>816は俺じゃないです。誰か質問に答えて下さい
>>819
判らなかったらどう困るのか、滔々と説明してくれたら答えを考えてやってもいいよ?
822 :03/11/24 01:04
ちょっとしつれい・・・・・

理科大二部受験生の皆様へ
11月21日〜24日まで開催される理大祭にて、二部受験を希望しているのに赤本がない!!という声にお答えして、解答つき問題集を配布いたします。
冊子はインフォメーションセンターか354教室にて。(神楽坂キャンパスのみ)
二部希望者は今すぐGO!
詳細は以下に載ってます。

http://www.ed.kagu.tus.ac.jp/suuken2/guide/
823811:03/11/24 01:04
>>816は俺です。本人です。誰か質問に答えて下さい
>>820
ログ嫁。
825811:03/11/24 01:05
>>821
だまれもまいヴァカ、ヴァカ ヴァカ ヴァカ
これでよろしいでしょうか?
826811:03/11/24 01:06
いちいちうぜえな そんなに粘着するなら答えろよ
827811:03/11/24 01:06
>>824
ログにどんなこと書かれてんの?
>>825
じゃあ別に困らないってことね。
829132人目の素数さん:03/11/24 01:07
>>797
収益率の定義は?
>>827
ここは中の人が増殖するスレだってことが書かれている。
831811:03/11/24 01:07
いちいちいちいちいちいちちんちんうぜえな そんなに粘着するなら答えろよ
832821:03/11/24 01:08
>>825
そんな事実を言われても、誰も困らないんだけど・・・;
>>831
がんがれ。
834811:03/11/24 01:08
>>828
困るんです!教えてくださいもまいヴァカ
お願いします!ヴァ〜カww
835811:03/11/24 01:10
わかったからもういいよ。
ヴァカ ヴァカ ヴァ〜カwww
>>834
だから、何をどう困るのかを滔々と(ry
>>835
がんがれ。
838132人目の素数さん:03/11/24 01:12
荒らしは↓こっちに専用のネタスレがあるのでどうぞ

http://science2.2ch.net/test/read.cgi/math/1068762275/
839magnus:03/11/24 01:13
>>829
収益率の定義は、(一枚の期待金額)/(宝くじ一枚の価格)
です。

>>790
ところが、当選本数が一本以外のときは、共同購入して分配した時の収益率が下がったのです。微妙に。
シミュレーションはまだしていませんが。
当選本数増やすほど、ドンドン減っていくのです。
誤差ですかね〜。

>10人で買ってあたれば、当選金を10等分だけど
出資金も10等分されているので
収益率は等しい。
当たらなければ、0円だし。

780>>宝くじの共同購入をして分配すると、同数量を個人購入した場合よりも収益率が下がりますよね?
840811:03/11/24 01:13
もういい。もまいら、ションベンかけてやる!!
>>838
自治厨ご苦労。
842132人目の素数さん:03/11/24 01:14
a,bを任意の実数とする点(cosa+sinb,cos3a+sin3b)の存在範囲を
xy平面上に図示せよ
cosa+sinb=X
cos3a+sin3b=Y
とおいてやっていったのですが
どうしてもできません
どうやれば良いでしょうか
>>840
かけに来い。ほら、待っててやるから。かけに来てみろ〜〜w

と小学生のように煽ってみるテスト。 ・・・ツマンネ。
844811:03/11/24 01:15
じょろじょろろろろ・・・・
うわ・・・液晶に・・・!!!!
全角英数字キモーイ。
846132人目の素数さん:03/11/24 01:16
>>844
ワロタ
>>844
ちょっとワロタw
>>842
http://bbs5.cgiboy.com/p/74/00242/ に逝って故意。類題を訊いてる香具師がいる。
849132人目の素数さん:03/11/24 01:18
>>839
買っているのが団体だろうが
個人だろうが、同数量買えば 期待値分戻ってくるわけでしょ?

どういうシミュレーションしたのか知らないけど
この部分はちゃんと一致してる?

共同購入した団体に戻ってきた金額と
同数購入した個人に戻ってきた金額の一致
850132人目の素数さん:03/11/24 01:20
死にたいんですけど?
>>850
どうぞ。
852magnus:03/11/24 01:23
>>849
いえいえ、シミュレーションをまだしていないのです。
個人の場合と、団体で分配の場合の二つの理論式を計算してます。
共同購入した団体に戻ってきた金額=同数購入した個人に戻ってきた金額とは、
もちろん一致します。

>>790
ところが、当選本数が一本以外のときは、共同購入して分配した時の収益率が下がったのです。微妙に。
シミュレーションはまだしていませんが。
当選本数増やすほど、ドンドン減っていくのです。
誤差ですかね〜。

>10人で買ってあたれば、当選金を10等分だけど
出資金も10等分されているので
収益率は等しい。
当たらなければ、0円だし。

780>>宝くじの共同購入をして分配すると、同数量を個人購入した場合よりも収益率が下がりますよね?
853132人目の素数さん:03/11/24 01:27
>>852
>シミュレーションはまだしていませんが。
>当選本数増やすほど、ドンドン減っていくのです。
>誤差ですかね〜。

シミュレーションしていないのに、誤差がでるわけなかろ?

>宝くじの共同購入をして分配すると、同数量を個人購入した場合よりも
>収益率が下がりますよね?

根拠を示してください。
854132人目の素数さん:03/11/24 01:28
>>851
通報
855132人目の素数さん:03/11/24 01:30
関西人がムキになってるよ
856magnus:03/11/24 01:31
たとえば、総数10000本の宝くじ、当選本数3本、グループでの購入本数100本、個人での出資比率10%
とすると、
個人で10本 0.002994602
共同で10本 0.002940885

見たいな感じになります。
857magnus:03/11/24 01:42
くじの総数をN
一等当選本数をH
グループでの総購入数をG
個人の出資分本数をn
とすると、
共同購入したときの期待収益率は、
COMBI(H,1)COMBI(N-H,G-1)/COMBI(N,G)*(n/G)
としました。
858132人目の素数さん:03/11/24 01:42
>>856
グループでの購入本数100本
ならば、個人での購入本数も100本では?
問題の設定は

個人でn本
m人の団体で各人がn本買うのか

個人でn本
m人の団体全部でn本買うのか

というあたりをちゃんと指定しないと。

上の場合であれば
10本買って3本あたり。というのは当たったら高いけど
当たる確率が低いってことね。

100本買って3本あたり。というのは当たりやすいけど
当たっても、かかった金額が高いために収益率を落とすってことね。
859magnus:03/11/24 01:57
>>グループでの総数が100本で、個人がその10%ずつですから、10本です。


>>856
グループでの購入本数100本
ならば、個人での購入本数も100本では?
問題の設定は

個人でn本
m人の団体で各人がn本買うのか

個人でn本
m人の団体全部でn本買うのか

というあたりをちゃんと指定しないと。

上の場合であれば
10本買って3本あたり。というのは当たったら高いけど
当たる確率が低いってことね。

100本買って3本あたり。というのは当たりやすいけど
当たっても、かかった金額が高いために収益率を落とすってことね。
860132人目の素数さん:03/11/24 02:10
>>859
レスのコピペはやめてください。
すぐ上見れば分かる。
861132人目の素数さん:03/11/24 02:17
>>859
多分、他人のレスを殆ど読まれていないと思うのですが
個人でn本購入したとき
m人の団体で各人がn本買い、団体全体としては mn本購入したとき
を比べて
団体全体で買った時の収益率がどうなるかという意味の
問題だとしたら、当然下がるでしょうそりゃ。


で、何が聞きたいの?
862magnus:03/11/24 02:26
>>861
なぜかというと、共同購入でより高確率であたると謳っているサービスがあったので。
結局手数料の分だけ損するですよね。(もちろん確率的に)
で訊きたいのは、同然下がるのはなぜか、お訊きしたいのです。
論理的にご教授願いします。
だれかさんは当たる確率と収益率を混同して話をしていないか?
864magnus:03/11/24 02:44
混同してますか?
出資した一個人のあたる確率=予想収益率=(共同購入して分配後に得られる出資比率に応じた当選金の期待金額)/(出資金額)
になるはずです。
865132人目の素数さん:03/11/24 03:07
4-2√3 を (√3-1)の二乗 と書きかえれるんですが、どうしても理解できません。
どのように考えていけば4-2√3 を (√3-1)の二乗 と出すことができますか?
どなたか分かりやすく説明していただけないでしょうか・・
>>865
和が4、積が3になる二数が3と1だから。
867132人目の素数さん:03/11/24 03:19
>>774

解答期待age
868865:03/11/24 03:27
>>866
ありがとうございました
869132人目の素数さん:03/11/24 04:46
整数700の約数の中で、正の数でかつ偶数であるものの個数と、それらの総和を求めよ

ただし書き出してはいけない
700=2^2・5^2・7
(2+4)(1+5+25)(1+7)を展開して出てくる項数=12。積=1488。
しかし書き出してはいけないって・・・
871132人目の素数さん:03/11/24 05:43
質問です。
−ー
a,b,cが条件a+b+c=abcをみたしながら動くとき
1/√(1+a^2)+1/√(1+b^2)+1/√(1+c^2)
の最大値をもとめよ。
−−
これ答え(3√3)/2だと思うんですがどうでしょう?
某サイトの問題なんですがこの答えいれても正解になりません。
(↑どこのサイトかわかる人にはわかると思いますが一応内緒ということで)
まちがってます?勘じゃなくてちゃんと証明できたつもりなんですが・・・
872132人目の素数さん:03/11/24 05:48
MathNoriの最新問題をここで質問するな、ヴォケ!
873132人目の素数さん:03/11/24 05:49
見てほしいのなら、自分のウソ解答を書け!
874132人目の素数さん:03/11/24 05:50
?枚の一円を、できるだけ5円玉に両替すると
60枚減ります
それをさらにできるだけ10円玉に両替すると
10枚になります

これをとく式を作れない証明を教えてください
875132人目の素数さん:03/11/24 05:54
   スッキリシタモナ
    ∧_∧
    ( ´∀`)
   /   つ
   (_(__⌒)  |^lヽ、  (´・ω・`) ← >>874
  ┌─(_)─┘.| )  (∩ ∩)
 ̄ ̄ ̄ ̄ ̄ ̄ ̄ ̄ ̄ ̄ ̄ ̄ ̄ ̄


         ∧_∧
         ( ´∀`)
         /    ヽ、
      (( (_'(_, )´ ふきふき
        (:・:ω:・:)
        (∩ ∩)
876871:03/11/24 06:03
計算まちがいしてますた。
 
>見てほしいのなら、自分のウソ解答を書け!
 
計算まちがいに気付いて答え修正して書きこんだら正解してしまったのでやめときます。
でもこの答えだと勘のいい人なら勘で正解してしまう・・・
877132人目の素数さん:03/11/24 06:11
正解したと言ってる割には、正解者にお前の名前が載ってないぞ!
最後に正解した人が、お前が書き込む1日前なのは不思議だな。ン?
>>877
「正解です」って画面がでてきてそのまま消した。べつに自分の名前載せたくないし。
10円硬貨4枚、100円硬貨6枚、500円硬貨2枚の全部または一部を使って支払える金額は何通りあるか??
>>871
本当に最大値なんてあるのか?
例えば、a=i、b=-i、c=1とすると、a+b+c=abcだが、1/√(1+a^2)+1/√(1+b^2)+1/√(1+c^2)→∞になる。
ちゃんと計算していないが、1/√(1+a^2)+1/√(1+b^2)+1/√(1+c^2)は任意の大きい値をとれそうな気がする。
881言語学板の数学好き:03/11/24 10:01
今日中にに答えていただけると有難いのですが

放物線y=x^2 と 直線l があり、交点をP, Qとする。
lは A(0, 1)を通り、 PA:AQ=2:1である。
点Qのx座標を求めよ

お願いします

こことどう違うの?
http://science2.2ch.net/test/read.cgi/math/1064879216/
882132人目の素数さん:03/11/24 10:05
>>871
a=b=c=0 のとき 3>3√3/2 じゃん! (藁
883言語学板の数学好き:03/11/24 10:58
>>881は気にしないでください
884132人目の素数さん:03/11/24 11:33
>>862
COMBI(H,1)COMBI(N-H,G-1)/COMBI(N,G)は一本だけ当たる確率
なんで期待値を計算しないのかわからないけど
G本買って、一本だけ当たる確率

(G/n)本買って、一本だけ当たる確率の違いを考えてみる

COMBI(N-H,G-1)/COMBI(N,G)の部分、Gが小さくなったらどうなるか?
885132人目の素数さん:03/11/24 12:12
>>880
複素数までいれちゃっていいのか?
最大最小の問題で複素数にまで定義域を広げると
値の無いところばかりだぞ
886132人目の素数さん:03/11/24 12:14
立方体の対角線を軸に回転させた体積を求めるのって
どうやればいいの?
887132人目の素数さん:03/11/24 12:22
>>886
昔、入試問題であったような
対角線に垂直な面で立方体を切ってやると
その切り口は、上の方と下のほうは三角形
これは回せば円錐になる。
真ん中の部分は、6角形だけど上のほうは大きく回り
真ん中の方になると小さく回る。
対角線からの最大の距離を出してやって(といっても6角形の頂点までの距離)
あとは回転体の積分。
>>886
三角錐が底面どうしでくっついているでいいのでは?
あとは三角錐の体積×2でどうでしょう?
889888:03/11/24 12:26
あー、円錐だった。失礼です。
890132人目の素数さん:03/11/24 12:41
>>888-887
いえ、真ん中の六角形になる部分は単純に行かないのです。
ちゃんと計算しないといけないけど
一辺の長さが1の立方体で
円錐になる部分は、底面の半径が(√6)/3
であるのに対し、切り口が六角形になるところの真ん中(正六角形が現れるところ)
の半径は(√2)/2で小さくなってます。
この部分を回転したものは円柱の側面が窪んだ形になります。(双曲面?)
891888:03/11/24 13:32
>>890
んんん?
>>887さんが書かれておられるように

対角線に垂直な面で立方体を切ってやると
その切り口は、上の方と下のほうは三角形
これは回せば円錐になる。

んだから、内側の部分は回転させちゃえば、大きい方の中になるんで
つまり円錐の内側なんで、内部なんで、体積の計算には無関係。

積分もいらないかと。

大学卒業してずいぶんたつんで忘れてることも多いかも。
でも、これは簡単な円錐の体積の計算でしょう。
一辺をhとおいてやると、円錐の底面の半径rはhで表せるから
ごりごりやればいいかと。

こういう問題はちゃんと図を書けば理解し易い。

だーれも助け舟を出さないのが無言の圧力でほろ苦いww
892132人目の素数さん:03/11/24 13:42
>>886
>>891

>>887=>>890です。

ttp://web2.incl.ne.jp/yaoki/asaikoro.htm
こちらに綺麗な図入りの解答がありましたので
御覧ください。

一番上の解答は、間違いですけど
絵は非常に綺麗で忠実に再現されています。
その下に積分を用いた解答があります。
どこの大学の入試問題か忘れましたが
問題集や参考書に載っているのではないでしょうか?
893132人目の素数さん:03/11/24 13:51
東工大の問題だったかな?
894132人目の素数さん:03/11/24 13:57
対角線をz軸に重ね、立方体の重心を原点に取る時、
ねじれの位置にある辺は、適当に回転させれば
x=(√2)/2
z=(√2)/2 *y
直線上のある一点と対角線の距離rは
変数zで表せて、
r^2 = x^2+y^2 = 1/2+2*z^2
確かに双曲面ですね。
積分範囲は[0,(√6)/3]の二倍
中央部分の体積は
2π*∫[0,(√6)/3](1/2+2*z^2)dz
あとは二つの円錐部分を加えればOK?
895888:03/11/24 14:09
>>892
ありがとうです。

うーん。
なんというか。ずいぶんややこしい計算なんですね。
ていうか、いまはこういう一般解まで求めさせるんだーとちょいとびっくり。
でも答えはきれいだから、いいのかな。

>>893
あー。東工大ですか。
入試受けて落ちましたw
できないはずだと今さら納得ww


あのページのトップの今週の問題でも解いてみます。
Q1、Q2は簡単なんだけど。
たまの休みに学生気分に戻るのも楽しいということで。
現役のみなさん、頑張って。
積分範囲間違えた。
897132人目の素数さん:03/11/24 16:04
>>871
S=1/√(1+a^2)+1/√(1+b^2)+1/√(1+c^2)
とします。
この式を見たときに、最初に考えるのは
a=tan x, b=tan y, c=tan z
という変数変換でしょう。
-π/2 < x, y, z, < π/2
としておけば、

S= cos x + cos y + cos z

これは当然、最大値は3です。
x=y=z=0の時。
これは、a=b=c=0の時であり
偶然ではありますが条件式a+b+c=abcを満たします。
898132人目の素数さん:03/11/24 16:10
>>897
ちゅーか、最初から、各項が0で最大なのは明らか。(w
899132人目の素数さん:03/11/24 17:53
はっはっはっ
>>897-898
オリジナルの問題はa,b,cが非負実数。
901900:03/11/24 17:56
しまった。まちがった。オリジナルの問題はa,b,cが正の実数。
902132人目の素数さん:03/11/24 18:06
正である必要ないだろ。
オリジナルは正の実数とかいてある。オリジナルの出題者にいってくれ。
904132人目の素数さん:03/11/24 19:08
>>897
a、b、c は独立変数じゃないのに
>a=tan x, b=tan y, c=tan z
という変数変換は、直には無理でしょ?!
正の実数の場合、上極限=3は存在しても、最大値は存在しないんじゃないか?
906132人目の素数さん:03/11/24 19:17
>>904
無理じゃないよ?
独立じゃなくても変数変換はできるし
条件式に変換をぶち込めば
変換後の条件式が得られる。
>>905
29/10になるようなa,b,cみつかる?みつからないハズだけど。
908132人目の素数さん:03/11/24 19:21
>>906
そういうのを『泥縄』と言います。(藁
909132人目の素数さん:03/11/24 19:25
デムパか?ワラ
いづれにせよこの話題はやめとこう。
オレのしってる答えはそのサイトにあった正解と同じだったからたぶんあってると思う。
(それは3ではないし確かに最大値になってるハズ)でも答えかくのはばかられるし。
答えがかけない以上議論をおわらせられないよ。
911132人目の素数さん:03/11/24 19:44
>>908
キミは何か他のものと勘違いしているような気がする。
正則な座標変換とか、
912132人目の素数さん:03/11/24 19:45
>>871 論理矛盾を平気でする奴は無視してっと ・・・
例えば、a+b+c=abc より c=(a+b)/(ab-1) (ab≠1) 
∴ 1+c^2={(ab-1)^2+(a+b)^2}/(ab-1)^2=(a^2b^2+a^2+b^2+1)/(ab-1)^2=(1+a^2)(1+b^2)/(ab-1)^2
P=1/√(1+a^2)+1/√(1+b^2)+1/√(1+c^2)=1/√(1+a^2)+1/√(1+b^2)+|ab-1|/{√(1+a^2)√(1+b^2)}
ここで a=tanθ、b=tanφ (0<a、0<b より 0<θ<π/2、0<φ<π/2) とおくと、-π/2<θ-φ<π/2 より 0<cos(θ-φ)≦1 だから
P=cosθ+cosφ+|tanθtanφ-1|cosθcosφ=cosθ+cosφ+|sinθsinφ-cosθcosφ|=cosθ+cosφ+|cos(θ-φ)|=cosθ+cosφ+cos(θ-φ)
また、0<φ/2<π/4 より 1/√2<cos(φ/2)<1、-π/4<θ-φ/2<π/2 より -1/√2<sin(θ-φ/2)<1 だから
P=2cos(φ/2)cos(θ-φ/2)+cosφ<2cos(φ/2)+2{cos(φ/2)}^2-1=2{cos(φ/2)+1/2}^2-3/2<2*(1+1/2)^2-3/2=3
最大値なんて無い?!(藁

微分してもいいぞっ!
P=f(θ) (0<θ<π/2) とすると
f'(θ)=-sinθ-sin(θ-φ)=-2cos(φ/2)sin(θ-φ/2)
0<φ/2<π/4 より 1/√2<cos(φ/2)<1、-π/4<θ-φ/2<π/2 より -1/√2<sin(θ-φ/2)<1 だから
maxf(θ)=f(φ/2)=cosφ+2cos(φ/2)=2{cos(φ/2)}^2-1+2cos(φ/2)=2{cos(φ/2)+1/2}^2-3/2<2(1+1/2)^2-3/2=3
やっぱり最大値は無い?!(藁

出題者が馬鹿ってことで1つ・・・(藁
913132人目の素数さん:03/11/24 19:53
>>912 がはははは 使いまわしでミスった 【訂正】 スマン
× また、0<φ/2<π/4 より 1/√2<cos(φ/2)<1、-π/4<θ-φ/2<π/2 より -1/√2<sin(θ-φ/2)<1 だから
○ また、0<φ/2<π/4 より 1/√2<cos(φ/2)<1、-π/4<θ-φ/2<π/2 より 0<cos(θ-φ/2)<1 だから
914132人目の素数さん:03/11/24 19:55
>>912
そんな計算はどうでもいいんだよ、
>>907の言ってることに答えることが重要
そこまで、最大値が無いと言える。
ただ上から押さえるだけではなくさ
915132人目の素数さん:03/11/24 20:01
>912-913もアホってことでいい?
abcが正の実数なら、
c>0より ab>1 という制約条件がまだある。
917132人目の素数さん:03/11/24 20:08
ってことは、abc>1か。
a=0, b=0, c=0には近付きようがないということだな。
a=b=c=0が有り得ないなら、
a=b=c=√3に落ち付くんかな。勘任せだけど。
919132人目の素数さん:03/11/24 20:21
では、最大値が3/2であることを証明してください
920132人目の素数さん:03/11/24 20:23
>>904
直にはってどういう意味?
だからこの話題やめよって。証明のっけるのは気がひけるよ。
正解たしかめたい方はこちらでご自分でってことで。
http://jp.mathnori.com/top/math/math.asp?math_type=previous&level=all&order=CREATION%5FDATE&sort=desc&ID=235
922132人目の素数さん:03/11/24 20:28
3/2が正解だった。
三辺の長さがそれぞれ3,6,7の三角形の面積の出し方教えてください
三角比の範囲です
924132人目の素数さん:03/11/24 20:36
証明のっけるのは毛が抜けるよ
925132人目の素数さん:03/11/24 20:41
>>923
三角比の範囲というのは、
三角関数を使えということなのか?
ヘロンの公式はダメなのか?
普通に絵を描いて三平方の定理はOKなのか?
>912のPをaで偏微分したものを0とすると
a+b = a*(1+a^2)*√(1+b^2)
同様に
a+b = b*(1+b^2)*√(1+a^2)
とりあえず
a*√(1+a^2)=b*√(1+a^2)
最後の行は
a*√(1+a^2)=b*√(1+b^2)
の間違い
928132人目の素数さん:03/11/24 21:03
>>923
ヘロンの公式でいいのでないかい??
929132人目の素数さん:03/11/24 21:04
>>912 くっそ! ネタバレ早すぎ! (藁
正解は・・・
例えば、a+b+c=abc より c=(a+b)/(ab-1) (ab≠1) 
a=tanθ、b=tanφ (0<a、0<b より 0<θ<π/2、0<φ<π/2) とおくと、0<c=-(tanθ+tanφ)/(1-tanθtanφ)=-tan(θ+φ) より π/2<θ+φ<π
P=1/√(1+a^2)+1/√(1+b^2)+1/√(1+c^2)=cosθ+cosφ-cos(θ+φ)
また、0<φ/2<π/4 より 0<sin(φ/2)<1/√2、π/2-φ/2<θ+φ/2<π-φ/2 より 0<min[sin(π/2-φ/2),sin(π-φ/2)]<sin(θ-φ/2)≦1 だから
P=2sin(φ/2)sin(θ+φ/2)+cosφ≦2sin(φ/2)+1-2{sin(φ/2)}^2=-2{sin(φ/2)-1/2}^2+3/2≦3/2
よって、求める最大値は θ=φ=π/6 つまり、a=b=c=√3 のとき 3/2
一時間か。よくがんばった。
931132人目の素数さん:03/11/24 22:08
>>923
http://ja.wikipedia.org/wiki/%E3%83%98%E3%83%AD%E3%83%B3%E3%81%AE%E5%85%AC%E5%BC%8F
ヘロンの公式

s=(3+6+7)/2=8

S=√{8*(8-3)*(8-6)*(8-7)}=4√5
932132人目の素数さん:03/11/24 23:27
コレとかどうですか?
http://webnews.fc2web.com
933132人目の素数さん:03/11/24 23:55
業者か?
934132人目の素数さん:03/11/24 23:57
さっきから業者うざいね
(a+b)^2(a-b)^2(a^4+a^2*b^2+b^4)^2

を計算したいのですが、解き方がわかりません。
よろしければどなたか教えて下さい。
>>935
= ( (a+b)(a-b)(a^4+a^2*b^2+b^4) )^2
= ( (a^2-b^2)(a^4+a^2*b^2+b^4) )^2
= (a^6-b^6)^2
= a^12-2a^6*b^6+b^12
937132人目の素数さん:03/11/25 00:39
業者上げまくりだね
>>936
ありがとうございました。
全部まとめて二乗しちゃえば良かったのですね……。
939132人目の素数さん:03/11/25 00:42
「△ABCの辺上に,AP:PB=2:1,AQ:QC=2:3となるように点P,Qをとる。
CPとBQの交点をOとし,AOの延長と辺BCとの交点をRとするとき,BR:RCを求めよ。」

という問題があります。
これをチェバの定理を使わずに解きたいのですが,うまくいきません。
どうしたら解けますか?
>>939
ベクトルでも使えば?
941132人目の素数さん:03/11/25 00:50
>>940
ぉぃぉぃ…
942940:03/11/25 00:56
>>941
はやっ!

>>939
じゃ、メネラウスでも使えば。
943132人目の素数さん:03/11/25 01:00

   /⌒'´     \
  /          \
 /    /⌒\      \
(   (     \       }
 |    )      \    l
 L   i' , ‐‐--、  , ‐ゝ、_ l
 {`ヽ, l _/(●>、 ;  /●ゝ{'
 .l( Y      .;  ;    |
  l`ー、,    ,. (Y  )ヽ、  )
  V l   ''~  ~` '~    |
   ゝ、|    -、‐'^ ^'ーr' ノ
     \   ゝ--''´;,/
       \\__ノ'

ゴージャス松野さんがこのスレに興味をもたれました
944132人目の素数さん:03/11/25 01:15
>>942
あほですか。
945132人目の素数さん:03/11/25 01:16
逆ラプラス・スチルチェス変換の仕方がわかりません。
946sage:03/11/25 01:17
AとBの最大公約数が1の時、A+BとA*Bの最大公約数も1である。
の証明お願いします。
947132人目の素数さん:03/11/25 01:22
>>939
分割した三角形の面積を使う。
a=△AOP
b=△BOP
c=△BOR
d=△COR
e=△COQ
f=△AOQ
と置く。
AP:BP=2:1から
a:b=2:1
(f+e):(c+d)=2:1
AQ:QC=2:3から
f:e=2:3
(a+b):(c+d)=2:3
BR:RC=x:yとでもしておいて
(a+b):(f+e)=c:d=x:y
これらを解いて、cとdの比がわかればよい。
上から4本は
a=2b
2(c+d)=(f+e)
e=(3/2)f
2(c+d)=3(a+b)
これから,
9b=(5/2)f
あとは↓を
(a+b):(f+e)=c:d
(f+e)c=(a+b)d
(5/2)fc=3bd
3c=d
よって、x:y=1:3
948132人目の素数さん:03/11/25 01:28
同意して欲しくないだろ、おまえみたいな語彙力0のバカに。
キモキモは消えろ。最近このスレ低年齢化進みすぎてねえか?
949132人目の素数さん:03/11/25 01:33
>>948
誤爆か?
次スレ
分からない問題はここに書いてね140
http://science2.2ch.net/test/read.cgi/math/1069691754/
951132人目の素数さん:03/11/25 01:42
rわr
>>946
次スレに移動
953939:03/11/25 02:02
>>940
書き忘れていましたが、中3です。
ベクトルは習ってないのでだめぽ。

>>947
面積を使うんですか、分かりました。
丁寧な解説、ありがとうございます!
         スコココバシッスコバドドドンスコバンスコ  _∧_∧_∧_∧_∧_∧_
            从 `ヾ/゛/'  "\' /".    |                     |
        ≡≪≡ゞシ彡 ∧_∧ 〃ミ≡从≡=< 「分からない問題はここに書いてね3-A」まだぁー?       |
.          '=巛≡从ミ.(・∀・# )彡/ノ≡》〉≡ |_  _  _ _ _ _ __ _|
...        《゛=!|l|》リl⌒! I⌒I I⌒I I⌒I从=≡|l≫,
          《 l|!|!l!((つT(つ) ((つT(つ)) !|l!|l;》;
       《 l|!| ̄| ̄γ ⌒ ヽ γ ⌒ ヽ三ll≡|l》;
..        《l|!|  | ((TAMA))((TAMA))||l|||l 》;
      ≡丿-へ/人 _ 人 人 _ 人//へヾ
          ドドドドドドドドドドドドドドドドドドド
次スレに移動しる!!!


分からない問題はここに書いてね141
http://science2.2ch.net/test/read.cgi/math/1069692272/

956811(本物):03/11/25 17:49
あのー、811ですが、僕は811にしか書き込みしてません。他はすべて偽者です。
誰か>>811の答えを知っている人がいたら、教えてもらえません?
「存在するけど計算はできない」っていうのなら、納得できます。
>>956
存在する。
積分範囲が
-∞〜∞の定積分であれば計算できる。
が不定積分は計算できないので
近似値で求められている。(正規分布表)
>>957
×が不定積分は計算できないので
×近似値で求められている。(正規分布表)

○が不定積分は計算できない。
○-∞〜∞ではない途中の点での値は近似値で求められている。(正規分布表)
959811(本物):03/11/25 20:26
>>957-958
ありがとうございます。
960132人目の素数さん:03/11/25 20:28
条件 xy=1のもとで関数x^2+2y^2の極大値は何か?の答えを教えてください。
961132人目の素数さん:03/11/25 20:29
lim  cosX/sionX
x→0

よろしくお願いします。
962132人目の素数さん:03/11/25 20:29
埋め
963132人目の素数さん:03/11/25 20:29
X sion
○ sin

訂正です;
964132人目の素数さん:03/11/25 20:44
>>961
あんたばか
>>960
 x^2+2y^2=x^2+2/x^2=(x−√2/x)^2+2√2
よって、最小値は2√2だが、極大値は存在しない。

>>961>>963
 cos(x)/sin(x)→±∞(x→±0)
966132人目の素数さん:03/11/25 22:25
954 :132人目の素数さん :03/11/25 07:27
         スコココバシッスコバドドドンスコバンスコ  _∧_∧_∧_∧_∧_∧_
            从 `ヾ/゛/'  "\' /".    |                     |
        ≡≪≡ゞシ彡 ∧_∧ 〃ミ≡从≡=< 「分からない問題はここに書いてね3-A」まだぁー?       |
.          '=巛≡从ミ.(・∀・# )彡/ノ≡》〉≡ |_  _  _ _ _ _ __ _|
...        《゛=!|l|》リl⌒! I⌒I I⌒I I⌒I从=≡|l≫,
          《 l|!|!l!((つT(つ) ((つT(つ)) !|l!|l;》;
       《 l|!| ̄| ̄γ ⌒ ヽ γ ⌒ ヽ三ll≡|l》;
..        《l|!|  | ((TAMA))((TAMA))||l|||l 》;
      ≡丿-へ/人 _ 人 人 _ 人//へヾ
          ドドドドドドドドドドドドドドドドドドド
>>961
cos x→1
sin x →0

ですが、
sin xの方は、0のどちら側から近付くかで符号が変りますので
±∞
分からない問題はここに書いてね140
http://science2.2ch.net/test/read.cgi/math/1069691754/
>>946
A+BとABの最大公約数が1より大きいと仮定する。
このとき、ある素数pが存在し、A+BとABの公約数となっている。
ABがpで割り切れるから、特に、A、Bの少なくともどちらか一方はpを約数に持たなければならない。
これをAと仮定すると、A=apと書ける。
A+B=ap+Bがpで割り切れるから、Bもpで割り切れなければならない。
これは、AとBの最大公約数が1であることに矛盾。
970960:03/11/26 00:17
>>965
ああ、この板 来てよかった〜。
とってもありがとうございます。
でらっくすちんこ
でらっくすちんこ
973132人目の素数さん:03/11/26 18:16
でらっくすまん(ry
974132人目の素数さん:03/11/26 18:33
反比例の変化の割合って常に一定ですか?
975132人目の素数さん:03/11/26 18:34
でらっくすうんこ
976132人目の素数さん:03/11/26 19:10
でらっくすまんこ
977132人目の素数さん:03/11/26 19:28
すぺしゃるまんこー
978132人目の素数さん:03/11/26 19:29
奥様うっとりこんぼうー
979132人目の素数さん:03/11/26 19:32
すぺしゃるまんこー
980132人目の素数さん:03/11/26 20:28
任意の実数a、bに対して、次の不等式が成り立つ事を何方か証明して下さいm(__)m


{∫0から1(ax+b)dx}二乗≦∫0から1(ax+b)二乗dx
鬼仏表:学生による大学の授業評価

制作したばかりなので
まだ全くデータがありませんが、
よろしくお願いします。

http://www.internext.ne.jp/kibutu/

どんどんコピーしていろんな掲示板に貼ってくれるとうれしいです。
982132人目の素人さん:03/11/26 23:07
>980
【シュワちゃんの不等式】
|∫[0,1] f(x)g(x)・dx|^2 ≦ {∫[0,1] |f(x)|^2・dx}・{∫[0,1] |g(x)|^2・dx}
等号成立は f(x)/g(x)=定数. 区間は[0,1]でなくてもよいが。
さくら140−[218] を参照
983132人目の素数さん:03/11/26 23:34
      __
    / ̄   \
    |   大  :::|    , -―/\
    |   道  ::::|   /_/__\
   |    寺  ::::::|   Vw;:fLi_l」」l_l」i
   |    家  ::::::|   !i(6|:| l  l |   / ̄ ̄ ̄ ̄ ̄ ̄ ̄ ̄ ̄ ̄ ̄ ̄ ̄ ̄ ̄
   |    代  :::::|   ノ;ノi;|:ト、 lフノ < 逝く時は一緒ですわ〜、さくらちゃん
   |    々  :::::::|  (:(:(:(( ∪ ∪  \_______________
  |     之  :::::::|  ););)|   |
  |     墓  :::::::|  (:;(:(:(; ) /
  |  ∬      ∬ :::|     ν ′
  |  ii ,,≦≧、 :ii :::::|
 |  旦‖===‖旦::::::| _
┘二二二二二二二二二└--ff---\--ff-\-
984132人目の素数さん:03/11/26 23:42
                    .    /\
                       / う \
                      ,´彡  め ..\
                     ノ ノっ\  て \
                   ./ /´   \  も .\
                 / /´.      \  い \
                 |  / /ヘ;;;;;      .\  い \
                 |  ) ';=r=‐リ        \ か. \
                 |    ヽ二/         \ ?....>
                 )ヽ `_  〈、_ _、___,-っ_ / |!
                 〉,; /   ,⌒´  ,,____,,、τイミ、
                 /  `  イ/⌒ ̄      .⌒` i!|!;
              /⌒\   /              ;i!|
            / 人  ヽ、/               .!i
           / ,/´ |\. .\               !;
          / ;/    > ヽ  )
          ノ )   /  `フ〜´
985132人目の素数さん:03/11/26 23:44
r;;;;;ノヾ 
ヒ‐=r=;'  
ヽ二/    茶でも飲みながら、3分間待ってやる!
(、っiョc)
゙'ー'゙ー'
         スコココバシッスコバドドドンスコバンスコ  _∧_∧_∧_∧_∧_∧_
            从 `ヾ/゛/'  "\' /".    |                     |
        ≡≪≡ゞシ彡 ∧_∧ 〃ミ≡从≡=< 「分からない問題はここに書いてね3-A」まだぁー?       |
.          '=巛≡从ミ.(・∀・# )彡/ノ≡》〉≡ |_  _  _ _ _ _ __ _|
...        《゛=!|l|》リl⌒! I⌒I I⌒I I⌒I从=≡|l≫,
          《 l|!|!l!((つT(つ) ((つT(つ)) !|l!|l;》;
       《 l|!| ̄| ̄γ ⌒ ヽ γ ⌒ ヽ三ll≡|l》;
..        《l|!|  | ((TAMA))((TAMA))||l|||l 》;
      ≡丿-へ/人 _ 人 人 _ 人//へヾ
          ドドドドドドドドドドドドドドドドドドド
987132人目の素数さん:03/11/26 23:46
.       ∧_∧  / ̄ ̄ ̄ ̄ ̄ ̄ ̄ ̄ ̄ ̄ ̄ ̄ ̄ ̄ ̄ ̄
       (;´Д`)< スンマセン、直ぐに片付けます
  -=≡  /    ヽ  \________________
.      /| |   |. |
 -=≡ /. \ヽ/\\_  r;;;;;ノヾ
    /    ヽ⌒)==ヽ_)= ヒ‐=r=;'
-=   / /⌒\.\ ||  ||  ヽ二/  どこへ行こうというのかね?
  / /    > ) ||   || ( つ旦O
 / /     / /_||_ || と_)_) _.
 し'     (_つ ̄(_)) ̄ (.)) ̄ (_)) ̄(.))
988132人目の素数さん:03/11/26 23:46
みるまらっっ!!!
 ̄ ̄∨ ̄ ̄ ̄ ̄ ̄ ̄ ̄ ̄ ̄ ̄ ̄ ̄ ̄
 ,‐ ' ´  ヽ
/ i lレノ)))
 / 人il.゚ ヮノ
   ⊂)Yib    /⌒)
    く|: _フつ/@ニ)'
     ∪ / ,/
    ._/@二)
    `ー―'"
989132人目の素数さん:03/11/26 23:47
どこへ行こうというのかね?
 ̄ ̄∨ ̄ ̄ ̄ ̄ ̄ ̄ ̄ ̄ ̄ ̄ ̄ ̄ ̄
 r;;;;;ノヾ      ,.:´ ̄::ヽ  
 ヒ‐=r=;'      l::;.w''w;::〉 
 ヽ二/      |(l|゚ ヮ゚ノ|
⊂)Yib  /⌒) ⊂)Yib  /⌒) 
く|: _フつ/@ニ)' く|: _フつ/@ニ)'
  ∪ / ,/    ∪ / ,/
 ._/@二)   . _/@二)
 `ー―'"     `ー―'"
990132人目の素数さん:03/11/26 23:48
            ノミ;;;;;r
            ';=r=‐ヒ   ははは、どこへ行こうというのだね?
        o、_,o丶▽ / _ _
        o○o⊇⊂ |__  __
        /___/| /  丿 |o  _ ___
       γ,-/| |UU'//耳
        | |(),|_| | |/二) =3ブベベ
        ゝ_ノ ̄ ̄ ̄ゝ_ノ
      ̄ ̄ ̄ ̄ ̄ ̄ ̄ ̄ ̄ ̄ ̄ ̄
991132人目の素数さん:03/11/26 23:49
        r;;;;;ノヾ            _________________
        ヒ‐=r=;'      ∬   / 
        'ヽ ▽/  っ━~~  < はっはっは、さっさと逃げればいいものを
      _と~,,  ~,,,ノ_.  ∀   \
          ミ,,,,/~), │ ┷┳━   ̄ ̄ ̄ ̄ ̄ ̄ ̄ ̄ ̄
       ̄ ̄ ̄ .じ'J ̄ ̄| ┃
       ̄ ̄ ̄ ̄ ̄ ̄ ̄  ┻
         スコココバシッスコバドドドンスコバンスコ  _∧_∧_∧_∧_∧_∧_
            从 `ヾ/゛/'  "\' /".    |                     |
        ≡≪≡ゞシ彡 ∧_∧ 〃ミ≡从≡=< 「分からない問題はここに書いてね3-A」まだぁー?       |
.          '=巛≡从ミ.(・∀・# )彡/ノ≡》〉≡ |_  _  _ _ _ _ __ _|
...        《゛=!|l|》リl⌒! I⌒I I⌒I I⌒I从=≡|l≫,
          《 l|!|!l!((つT(つ) ((つT(つ)) !|l!|l;》;
       《 l|!| ̄| ̄γ ⌒ ヽ γ ⌒ ヽ三ll≡|l》;
..        《l|!|  | ((TAMA))((TAMA))||l|||l 》;
      ≡丿-へ/人 _ 人 人 _ 人//へヾ
          ドドドドドドドドドドドドドドドドドドド
993132人目の素数さん:03/11/26 23:50
くそ〜ゴリアテ何をしている!!
               
        .... .     プンプン!!
        r;;;;;ノヾ.      
        ヒ‐=r=;' 
   −=≡ 'ヽ二/
  −=≡ ( ヽ┐U キコキコキコキコ
 −=≡ ◎−>┘◎     
 ̄ ̄ ̄ ̄ ̄ ̄ ̄ ̄ ̄ ̄ ̄ ̄ ̄ ̄ ̄ ̄ ̄ ̄ ̄ ̄ ̄ ̄ ̄ ̄ ̄ ̄ ̄ ̄ ̄
994132人目の素数さん:03/11/26 23:50
今だ!!!ひざまづけ!!!
 ̄ ̄ ̄ ̄ ̄∨ ̄ ̄ ̄       (´´
      ノミ;;;;;r
     ';=r=‐ヒ   )      (´⌒(´
   ⊂丶▽ /⊂⌒`つ≡≡≡(´⌒;;;≡≡≡
        ̄ ̄  (´⌒(´⌒;;
      ズザーーーーーッ
995132人目の素数さん:03/11/26 23:50
  + ハッハッハッ… /ヘ;;;;;             +
  +      ∧∧  ';=r=‐リ  ∧∧   +
         (*^ー゚∩ ヽ二/∩ (*^ー゚)
      +  (つ ノ (つ ノ  ノ つつ    +
 +     〜ヽ ( ノ〜ヽ、Y〜( _つ
          し'`J  .ιヽ)  (/        +
     君も男なら埋めたまえ!
         スコココバシッスコバドドドンスコバンスコ  _∧_∧_∧_∧_∧_∧_
            从 `ヾ/゛/'  "\' /".    |                     |
        ≡≪≡ゞシ彡 ∧_∧ 〃ミ≡从≡=< 「分からない問題はここに書いてね3-A」まだぁー?       |
.          '=巛≡从ミ.(・∀・# )彡/ノ≡》〉≡ |_  _  _ _ _ _ __ _|
...        《゛=!|l|》リl⌒! I⌒I I⌒I I⌒I从=≡|l≫,
          《 l|!|!l!((つT(つ) ((つT(つ)) !|l!|l;》;
       《 l|!| ̄| ̄γ ⌒ ヽ γ ⌒ ヽ三ll≡|l》;
..        《l|!|  | ((TAMA))((TAMA))||l|||l 》;
      ≡丿-へ/人 _ 人 人 _ 人//へヾ
          ドドドドドドドドドドドドドドドドドドド
997132人目の素数さん:03/11/26 23:51
  + ハッハッハッ… /ヘ;;;;;             +
  +      ∧∧  ';=r=‐リ  ∧∧   +
         (*^ー゚∩ ヽ二/∩ (*^ー゚)
      +  (つ ノ (つ ノ  ノ つつ    +
 +     〜ヽ ( ノ〜ヽ、Y〜( _つ
          し'`J  .ιヽ)  (/        +
     君も男なら埋めたまえ!
998132人目の素数さん:03/11/26 23:51
\ 立ててしまったものは仕方ないでしょう ./
 .\                      ./
    ̄∨ ̄ ̄ ̄ ̄ ̄ ̄ ̄ ̄ ̄ ̄ ̄ ̄ ̄
 ,.:´ ̄::ヽ         /ヘ;;;;;  / ̄ ̄ ̄ ̄ ̄ ̄ ̄ ̄ ̄
 l::;.w''w;::〉 ∧_∧ネー ';=r=‐リ< 質問スレがゴミのようだ!
 |(l|゚ ヮ゚ノ| ( ・3・ )   ヽ二/  \_________
とjl([l个j])__(つ ___ )_(旦 と )_
∧ ̄ ∧_∧ ̄∧_∧ ̄ ̄ ̄./
  )  (    ) (    )y━~~~┃
 ̄ ̄ ̄ ̄ ̄ ̄ ̄ ̄\ )_/_) ┃
       .━┏  || ┃
     o   ┛  |_).┃
    O 
(クソスレに乾杯…)
         スコココバシッスコバドドドンスコバンスコ  _∧_∧_∧_∧_∧_∧_
            从 `ヾ/゛/'  "\' /".    |                     |
        ≡≪≡ゞシ彡 ∧_∧ 〃ミ≡从≡=< 「分からない問題はここに書いてね3-A」まだぁー?       |
.          '=巛≡从ミ.(・∀・# )彡/ノ≡》〉≡ |_  _  _ _ _ _ __ _|
...        《゛=!|l|》リl⌒! I⌒I I⌒I I⌒I从=≡|l≫,
          《 l|!|!l!((つT(つ) ((つT(つ)) !|l!|l;》;
       《 l|!| ̄| ̄γ ⌒ ヽ γ ⌒ ヽ三ll≡|l》;
..        《l|!|  | ((TAMA))((TAMA))||l|||l 》;
      ≡丿-へ/人 _ 人 人 _ 人//へヾ
          ドドドドドドドドドドドドドドドドドドド
1000132人目の素数さん:03/11/26 23:52
     r;;;;;ノヾ
     ヒ =r=;'
     _ヽ二/_
  / ̄  /~〉  ̄ヽ  ムスカ大佐
  / ヽ o ^~ o ノ l
 (  ヽ o   o ノ l   数学板の主人公
  ヽ  ノ    ヽノ   のはずだが、あまり出番がない
   ヽノ     ヽ
   ノ、____i
    l   l  l
    l   l  l
    l   l  l
    l   l  l
    .(__⌒)_⌒)
10011001
このスレッドは1000を超えました。
もう書けないので、新しいスレッドを立ててくださいです。。。